Rincón Matemático

Revista, Técnicas, Cursos, Problemas => La revista del foro => Artículos => Mensaje iniciado por: Carlos Ivorra en 03 Enero, 2012, 10:48 pm

Título: Número 1. (2012) - 2. NFA
Publicado por: Carlos Ivorra en 03 Enero, 2012, 10:48 pm
"Por culpa" de Sailor Starruler, hace unos meses que empecé a interesarme por NFU (aunque, como me va lo castizo, aquí diré NFA). Aunque todavía me quedan algunas cosas que estudiar al respecto, creo que ya he logrado hacerme una idea clara de cómo funciona y, como me parece curioso e interesante, voy a dedicar este hilo a exponer a un nivel lo más elemental posible, dentro de lo riguroso, en qué consiste. Mientras tenga constancia de que le interesa a alguien avanzaré hasta donde consideremos adecuado.

¿Qué es NFA? Es una teoría de conjuntos. Su historia se remonta hasta 1937, cuando Quine publicó su libro "New Foundations for Mathematical Logic", en el que presentaba una teoría de conjuntos (hoy conocida como NF, por "New Foundations" con vocación de superhéroe, pues su propósito principal era "salvar el universo", es decir, que, en ella, el universo, o el conjunto universal, o el conjunto de todos los conjuntos, no fuera un objeto inexistente desterrado del nivel formal por contradictorio, como en ZFC, ni un "conjunto de segunda categoría", como en NBG, donde es "una clase" pero "no un conjunto", sino que en NF el conjunto universal \( V \), el conjunto de todos los conjuntos, es un conjunto más, en pie de igualdad con el conjunto vacío o el conjunto de los números naturales.

La teoría de Quine no tuvo mucho impacto, en su competición con ZFC, pero sus posibilidades cayeron bajo mínimos cuando en 1953 Specker demostró que NF se vuelve contradictoria si se le añade el axioma de elección, mientras que, más de una década antes, Gödel había probado que si ZF es consistente, también lo es al añadirle el axioma de elección.

Teniendo en cuenta además que NF tiene rasgos un tanto pintorescos, el hecho de que fuera contradictoria con el axioma de elección hizo más sospechosa su propia consistencia, aun sin dicho axioma. Y hasta la fecha, nadie ha sabido probar, no ya que NF sea consistente en términos absolutos, que sería mucho pedir, sino al menos que NF es consistente bajo el supuesto de que ZFC lo sea.

De este modo, es fácil comprender que una teoría de consistencia dudosa y probadamente inconsistente con el axioma de elección, no haya merecido mucha atención por parte de los matemáticos.

Sin embargo, su posición cambió ligeramente en 1969, cuando Jensen presentó una corrección de NF libre de ambos inconvenientes. Jensen demostró que si en NF admitimos que los elementos de los conjuntos no sean necesariamente conjuntos (como sucede en ZFC), sino que también puedan contener otros objetos llamados átomos, que puedan ser elementos de los conjuntos sin ser ellos mismos conjuntos (lo que sería una formalización de las peras, manzanas, ovejas, etc., que son objetos que permiten formar conjuntos de peras, manzanas, etc. sin ser ellos mismos conjuntos), la teoría resultante (que es NFA, es decir, NF con Átomos) es consistente supuesto que lo sea ZFC y además es consistente con el axioma de elección.

A partir del trabajo de Jensen varios grupos de lógicos se interesaron por NFA (por cierto, su nombre en inglés, NFU, viene de Urelement que es el nombre que dan a los átomos). La verdad sea dicha, el interés que NFA sigue suscitando entre los matemáticos es más bien escaso, hasta el punto de que hoy yo seguiría sin saber siquiera que existe de no ser por las preguntas de Sailor en este foro. Pero lo cierto es que, como he dicho al principio, NFA es curioso e interesante.

¿Qué tiene de bueno NFA? La pregunta es muy subjetiva. Habrá quien considere que el mero hecho de que "salva el universo" ya vuelve a la teoría heroica y digna de interés. Si somos más pragmáticos y nos preguntamos si NFA es o no preferible a ZFC, mi opinión personal es que NFA no tiene nada que hacer frente a ZFC como teoría para servir de base a las matemáticas, no ya por el "hecho consumado" de que los matemáticos se han acostumbrado a ZFC y no lo cambiarían por otra teoría aunque fuera mejor, sino que mi opinión es que NFA no es mejor.

Siendo así (mi opinión), ¿qué hago escribiendo esto? Bueno, creo que, pese a lo que acabo de decir, conocer NFA es "educativo". Creo que mucha gente no acaba de tener una idea clara y apropiada de lo que es una teoría de conjuntos porque sólo conoce un ejemplo (o dos muy parecidos). Conocer NFA es como si alguien que vive muy cómodamente en su ciudad y que nunca ha salido de ella (pensemos en alguien de la época en que no había televisión ni internet) decide viajar por el mundo y, tras un largo viaje, acaba concluyendo que en ninguna parte se vive tan bien como en su ciudad. Creo que eso no es razón para que esta persona recomendara a sus conciudadanos que no salieran nunca de su ciudad, puesto que no hay nada mejor por ahí fuera. Al contrario, conocer otras costumbres y otras formas de vida cambia necesariamente la forma que tiene uno de ver las cosas, y en general las cambia para bien, es enriquecedor.

Creo que a un formalista radical, conocer NFA le debería llevar a reflexionar sobre que uno no puede decir, "trabajo en ZFC" porque sí, ya que otro podría decir "trabajo en NFA porque sí", y las consecuencias son muy distintas, así que algo tiene que haber que distinga a ZFC de NFA y no es algo que pueda expresarse en términos de un formalismo radical, pues tanto ZFC como NFA son sistemas axiomáticos formalmente aceptables. Y a alguien que piense que debajo de los lenguajes formales hay alguna clase de "verdad", conocer NFA le ayudará a concienciarse de hasta qué punto una teoría axiomática puede "traicionar" la idea más o menos vaga de "verdad" que uno pueda tener.

No me extiendo más. En los próximos mensajes de este hilo iré presentando NFA. Mi propósito es mantenerme al nivel usual semiformal de los libros de matemáticas, es decir, no entrar en cuestiones puramente lógicas. La lógica subyacente será siempre la lógica clásica de primer orden, la misma que la de ZFC y, por supuesto, si alguien plantea preguntas que involucren aspectos lógicos, entraremos en ellos, pero si es posible no entrar en esa clase de asuntos, creo que la exposición resultará más ligera sin menoscabo del rigor. No obstante, como veremos, será imprescindible describir con detalle, si no la lógica, sí el lenguaje formal de NFA, y por ahí empezaremos (si es que esto le interesa a alguien, claro).

Bibliografía
En la wikipedia http://en.wikipedia.org/wiki/New_Foundations (http://en.wikipedia.org/wiki/New_Foundations) hay bastante material, en especial ahí está disponible el libro de Holmes "Set Theory with a Universal Set", que es bastante elemental en la forma de exposición, pero que abarca los aspectos más relevantes de la teoría.
Título: Re: NFA
Publicado por: argentinator en 03 Enero, 2012, 11:15 pm
Bueno, a mí me interesa, pero no sé si al punto de hacerte gastar demasiado tiempo escribiendo.

En cuanto a la elección entre NFA y NFU, pienso que puede ser confuso, si es que en la literatura corriente se conoce como NFU.
El día que los hispanoparlantes dominemos el mundo nos podremos dar el lujo de elegir las siglas castellanizadas.

__________

Citar
Creo que mucha gente no acaba de tener una idea clara y apropiada de lo que es una teoría de conjuntos porque sólo conoce un ejemplo (o dos muy parecidos).

Comparto plenamente esta reflexión, y como es una idea que no me cuesta entender en absoluto, es por eso que no necesito el "ejemplo educativo" de NFU. Perfectamente puedo imaginarme otras teorías en las que los axiomas no sean tal cual los de ZFC.

No veo nada "sagrado" en un axioma u otro.

Por ejemplo, hay mucha gente que ve como "sagrado" el Axioma de Elección.

Pienso que un tratamiento serio y profundo de la teoría de conjuntos tiene que dejar de lado el Axioma de Elección.
Eso no significa que haya que descartar la matemática actual, que se apoya enormemente en dicho Axioma.
De lo que se trata es, en realidad, de considerar a la matemática actual junto con el Axioma de Elección como "pequeñísimas subteorías" de teorías aún mayores y más generales.

_____________

De NFU lo que me interesa es entender cómo funciona el tratamiento de los números cardinales y ordinales, y cómo se comportan respecto a las diversas jerarquías de conjuntos que allí hay.

Y también me intriga esto que has dicho de que es mejor ZFC, y que NFU no aporta nada.
¿Por qué considerás que la teoría NFU es limitada o inadecuada?
¿Qué se puede decir en ZFC o NBG que no se pueda decir o hacer en NFU?

¿No hay acaso alguna gran "clase" en NFU que sirva de simulación o modelo de ZFC?
Título: Re: NFA
Publicado por: Carlos Ivorra en 03 Enero, 2012, 11:26 pm
Bueno, a mí me interesa, pero no sé si al punto de hacerte gastar demasiado tiempo escribiendo.

Bueno, de momento tengo el morbo de escribir. En general, siempre he considerado que la mejor forma de entender algo es escribir sobre ello.

En cuanto a la elección entre NFA y NFU, pienso que puede ser confuso, si es que en la literatura corriente se conoce como NFU.
El día que los hispanoparlantes dominemos el mundo nos podremos dar el lujo de elegir las siglas castellanizadas.

Ah, se siente. Yo castellanizo todo. Queda aquí la advertencia por si alguien se puede marear que donde yo digo NFA el mundo dice NFU. Prometo no ser "no estándar" en nada más (simplemente porque no se va a dar el caso de que haya nada más que castellanizar en la teoría).

Citar
Creo que mucha gente no acaba de tener una idea clara y apropiada de lo que es una teoría de conjuntos porque sólo conoce un ejemplo (o dos muy parecidos).

Comparto plenamente esta reflexión, y como es una idea que no me cuesta entender en absoluto, es por eso que no necesito el "ejemplo educativo" de NFU. Perfectamente puedo imaginarme otras teorías en las que los axiomas no sean tal cual los de ZFC.

No veo nada "sagrado" en un axioma u otro.

Bueno, yo sólo puedo decir que a mí NFA me ha sorprendido mucho, y es uno de los motivos por los que me parece interesante presentarlo aquí. Si tú crees estar "curado de espanto", que decimos por aquí cuando alguien no se sorprende ya de nada... pues ya se verá.  ;)

Por ejemplo, hay mucha gente que ve como "sagrado" el Axioma de Elección.

Pienso que un tratamiento serio y profundo de la teoría de conjuntos tiene que dejar de lado el Axioma de Elección.
Eso no significa que haya que descartar la matemática actual, que se apoya enormemente en dicho Axioma.
De lo que se trata es, en realidad, de considerar a la matemática actual junto con el Axioma de Elección como "pequeñísimas subteorías" de teorías aún mayores y más generales.

Estoy de acuerdo, pero indudablemente es un tanto a favor de ZF que podamos trabajar con y sin el axioma de elección. NF no dejaba esa opción, mientras que NFA sí que la da.

De NFU lo que me interesa es entender cómo funciona el tratamiento de los números cardinales y ordinales, y cómo se comportan respecto a las diversas jerarquías de conjuntos que allí hay.

Ah, pues podemos entrar en todo eso, si quieres.

Y también me intriga esto que has dicho de que es mejor ZFC, y que NFU no aporta nada.
¿Por qué considerás que la teoría NFU es limitada o inadecuada?
¿Qué se puede decir en ZFC o NBG que no se pueda decir o hacer en NFU?

Bueno, puesto que pretendo presentar NFA, ¿no crees que sería mejor conocerla primero y debatir después?

¿No hay acaso alguna gran "clase" en NFU que sirva de simulación o modelo de ZFC?

Sí, la hay. En principio en NFA pudes simular ZFC sin el axioma de partes. De hecho (esto es lo que estoy estudiando justo ahora, pero creo que es así) puedes simular la teoría de Morse Kelley sin el axioma de partes.

Añadiendo axiomas más fuertes, puedes simular todo ZFC.
Título: El lenguaje de NFA
Publicado por: Carlos Ivorra en 03 Enero, 2012, 11:28 pm
Es conocido que la teoría de conjuntos usual, ZFC parte de dos nociones primitivas: la noción de conjunto y la noción de pertenencia. Esto significa que si queremos hacer matemáticas con rigor tenemos la obligación de dar una definición precisa de todos los conceptos que usemos (producto cartesiano, función continua, espacio compacto, etc.) excepto los dos anteriores. Las propiedades de los conjuntos y de la relación de pertenencia no se deducen de una inexistente definición de conjunto o de pertenencia, sino de los axiomas de la teoría de conjuntos.

La teoría original NF partía de las dos mismas nociones primitivas, pero, como hemos indicado, NFA introduce la distinción entre conjuntos y átomos, por lo que es necesaria una tercera noción primitiva. Las tres nociones primitivas de NFA son objeto, conjunto y pertenencia.

Aunque no tengan una definición "oficial", es conveniente explicar "extraoficialmente" qué pretenden significar estos objetos. La noción de objeto será la más general de todas: todos los entes de los que habla el lenguaje de NFA son objetos. Cuando escribamos \( \forall x \) o \( \exists x \) leeremos "para todo objeto \( x \)" o "existe un objeto \( x \)".

Los objetos podrán ser de dos tipos: conjuntos o átomos. Por ello, el lenguaje formal de NFA constará de los signos lógicos usuales (\( \forall, \exists, \land, \lor, \lnot, \) etc.) más otros dos signos, que representaremos por cto y \( \in \).

La expresión \( \text{cto}\,x \) la leeremos "el objeto \( x \) es un conjunto", mientras que \( x\in y \) se leerá "el objeto \( x \) pertenece al objeto \( y \)".

Aquí es esencial comprender que, ni hemos definido "objeto", ni hemos definido "conjunto", ni hemos definido "pertenencia". Simplemente acabamos de establecer un lenguaje apto para hablar de objetos, conjuntos y pertenencia sin definirlos. Daremos axiomas que especificarán qué propiedades tienen los objetos en general, los conjuntos en particular y la pertenencia y nos comprometeremos a definir con rigor cualquier concepto que no sea uno de estos tres.

Por ejemplo, podemos definir el concepto de átomo:

Definición: Diremos que un objeto es un átomo si no es un conjunto. Más formalmente, \( \text{atm}\,x\equiv \lnot\text{cto}\,x \).

Conviene destacar el uso que haremos del signo \( \equiv \). No es un signo lógico. Se lee "es una abreviatura de ". En este caso significa que cuando escribamos \( \text{atm}\,x \) habrá que entender que se trata de una abreviatura de la expresión \( \lnot\text{cto}\,x \). Oficialmente, todas las definiciones matemáticas serán abreviaturas de expresiones previamente definidas, las cuales a su vez serán abreviaturas de expresiones previamente definidas, hasta llegar a expresiones que sólo contengan signos lógicos y los signos primitivos, cto y \( \in \).

Por último, quiero advertir que este mensaje contiene una mentira piadosa: en realidad, por cuestiones técnicas, veremos luego que conviene añadir al lenguaje de NFA un último signo primitivo, un signo redundante que se podría omitir, pero a costa de hacer más complicada la exposición de la teoría. Lo introduciré dos mensajes después de éste (si sigo el plan que tengo en la cabeza).
Título: Extensionalidad y axiomas afines
Publicado por: Carlos Ivorra en 04 Enero, 2012, 12:08 am
En el mensaje anterior he dicho que convenía "explicar extraoficialmente" qué son los objetos, los conjuntos, los átomos y la pertenencia. La verdad es que no he dicho mucho al respecto, aunque también es verdad que es fácil imaginar lo que son. La idea de que los conjuntos representan colecciones de objetos (y que la pertenencia indica que tal objeto pertenece a tal conjunto) queda plasmada en el primer axioma de NFA:

Axioma de Extensionalidad  \( \forall xy(\text{cto}\,x\land \text{cto}\,y\rightarrow (\forall u(u\in x\leftrightarrow u\in y)\leftrightarrow x=y)) \)

En palabras: "Si \( x \) e \( y \) son dos conjuntos cualesquiera, entonces \( x=y \) si y sólo si ambos tienen los mismos elementos, es decir, si todo objeto \( u \) que pertenece a \( x \) también pertenece a \( y \) y viceversa."

Una nota técnica para entendidos, que puede omitirse:
Spoiler
En realidad la implicación \( \leftarrow \) se puede suprimir, pues es un teorema lógico: si dos conjuntos son iguales tienen las mismas propiedades, en particular los mismos elementos, pero la incluyo precisamente para no entrar en cuestiones lógicas.
[cerrar]

Las expresiones del lenguaje NFA se simplifican considerablemente si adoptamos un convenio de notación:

Convenio de notación: En lo sucesivo convendremos en que las letras mayúsculas representan exclusivamente conjuntos, mientras que las minúsculas representan objetos cualesquiera, átomos o conjuntos.

Con este convenio, el axioma de extensionalidad puede reescribirse así:

Axioma de Extensionalidad  \( \forall XY(\forall u(u\in X\leftrightarrow u\in Y)\leftrightarrow X=Y) \)

El axioma de extensionalidad expresa que un conjunto \( X \) es una colección de objetos, concretamente, la colección de todos los objetos \( u \) que cumplen \( u\in X \), en el sentido de que dos conjuntos \( X \) e \( Y \) que determinen de este modo la misma colección de objetos, es decir, tales que todo objeto que pertenezca a uno también pertenece al otro y viceversa, son el mismo conjunto.

Observemos ahora con atención la definición de inclusión en NFA:

Definición: \( x\subset y\equiv \text{cto}\,x\land\text{cto}\,y\land \forall u(u\in X\rightarrow u\in Y) \)

Con palabras: Un objeto \( x \) es un subconjunto de un objeto \( y \) si ambos objetos son conjuntos y todo objeto que pertenezca a \( x \) también pertenece a \( y \).

Podríamos haber usado mayúsculas para abreviar, pero hemos querido explicitar el hecho de que la expresión \( X\subset Y \) incluye en su definición que \( X \) e \( Y \) tienen que ser conjuntos. En otras palabras un átomo puede pertenecer a otro objeto, pero, por definición, nunca puede contener o estar contenido en otro objeto.

Ahora es fácil demostrar:

Teorema:

\( \forall X\ X\subset X \)

\( \forall XY (X\subset Y\land Y\subset X\rightarrow X=Y) \)

\( \forall XYZ(X\subset Y\land Y\subset Z\rightarrow X\subset Z) \).

De hecho, el axioma de extensionalidad sólo es necesario para demostrar la segunda propiedad. Las otras dos son teoremas lógicos (consecuencias lógicas de la mera definición de inclusión, que no dependen de ningún axioma conjuntista).

Con esto queda establecido que los conjuntos, aunque no tengan "definición oficial", pretenden ser colecciones de objetos, pero ¿qué pretende ser un átomo? Como anticipamos en el primer mensaje, un átomo pretende ser "material de relleno", es decir, un objeto que puede pertenecer a determinados conjuntos sin ser él mismo un conjunto. ¿Dónde consta eso en los axiomas de NFA? En el de extensionalidad no, sin duda. Podríamos considerar el axioma siguiente:

Axioma de los átomos: \( \forall xy(x\in y\rightarrow \text{cto}\,y) \)

En palabras: Si un objeto tiene elementos, entonces es un conjunto o, equivalentemente, los átomos son objetos sin elementos.

A decir verdad, no vamos a tomar el axioma de los átomos como axioma de NFA por una razón muy simple: aunque lo hiciéramos, jamás tendríamos ocasión de usarlo. La razón es que nunca vamos a hablar de hipotéticos elementos de los átomos, más objetivamente: los axiomas de NFA no dicen absolutamente nada sobre hipotéticos elementos de los átomos, y por eso es consistente suponer que los átomos no tienen elementos. Sin embargo, como nunca vamos a hablar de elementos de átomos, suponer que no existen no nos aporta nada en la práctica.

En resumen: a efectos teóricos podemos considerar el axioma de los átomos como un axioma de NFA, un axioma que precisa el concepto de átomo como el de "objeto que no tiene elementos", pero a efectos prácticos se da el hecho de que ninguno de los teoremas que demostraremos en NFA se apoyará en este axioma, luego da igual que lo contemos o que no lo contemos entre los axiomas. Decir que tal átomo pertenece a tal otro es como decir que "tal gallina pertenece a tal piedra". No molesta en nada que alguien diga eso de unos objetos (una piedra y una gallina) que no son conjuntos pero tampoco aprovecha para nada.

Para terminar con este mensaje citamos un posible axioma relacionado con lo que hemos visto aquí:

Axioma NA: \( \forall x\ \text{cto}\,x \)

En palabras: Todos los objetos son conjuntos o, equivalentemente, no existen átomos.

Si añadimos NA a los axiomas de NFA, obtenemos la teoría original de Quine, la teoría NF. Como hemos indicado en el primer mensaje, el inconveniente de prescindir de los átomos es que NA es incompatible con el axioma de elección. Más aún, se sabe que si ZFC es consistente NFA es consistente, pero no se sabe si NFA + NA es consistente.

En resumen: hemos visto que el axioma de extensionalidad y el axioma (opcional) de los átomos determinan los significados pretendidos de los objetos, conjuntos, átomos y de la pertenencia: los conjuntos son colecciones de objetos, los átomos son objetos que no tienen elementos, pero que pueden ser elementos de los conjuntos y la pertenencia es la relación que determina si un objeto es o no un elemento de un conjunto.
Título: Pares ordenados
Publicado por: Carlos Ivorra en 04 Enero, 2012, 12:25 am
En este mensaje añadiremos un signo primitivo al lenguaje de NFA, junto con un axioma que determine sus propiedades. La situación es muy delicada porque, a simple vista, tanto el signo como el axioma parecerán redundantes, cuando avancemos un poco (unos tres mensajes más) se verá que no es evidente que sean redundantes, pero lo cierto es que son redundantes, en el sentido de que el signo que introducimos como signo primitivo se puede definir y el axioma se puede demostrar.

En este punto es imposible explicar por qué (podré explicarlo dentro de tres mensajes), pero lo cierto es que introducir este signo y este axioma simplifican drásticamente la exposición de la teoría. Por otra parte, tanto el signo como el axioma tienen un aspecto claramente "inofensivo". Ahora podemos dar ya la definición definitiva del lenguaje de NFA:

Definición: Llamaremos lenguaje formal de NFA al lenguaje que consta de los signos lógicos más tres signos primitivos adicionales, el signo \( \text{cto} \), el signo \( \in \) y el signo \( (\ ,\ ) \).

La expresión \( (x,y) \) la leeremos "par ordenado de primera componente \( x \) y segunda componente \( y \)". Al decir que \( (\ ,\ ) \) es un signo primitivo estamos diciendo que no vamos a definirlo, sino que las propiedades de los pares ordenados estarán determinadas por el axioma siguiente:

Axioma de los pares ordenados: \( \forall xyuv((x,y)=(u,v)\leftrightarrow x=u\land y=v) \).

En palabras: Dos pares ordenados son iguales si y sólo si tienen la misma primera componente y la misma segunda componente.

De este modo, por el mero hecho de introducir el nuevo signo, estamos diciendo que, para cada par de objetos \( x \) e \( y \), existe un tercer objeto (no especificamos si es un átomo o un conjunto) que representamos por \( (x,y) \), y el axioma de los pares ordenados dice que el objeto \( (x,y) \) determina completamente cuál es su primera componente y cuál es su segunda componente.
Título: Formación de conjuntos: un arranque en falso
Publicado por: Carlos Ivorra en 04 Enero, 2012, 01:18 am
Sea \( \phi(x,x_1,\ldots, x_n) \) una fórmula del lenguaje de NFA (es decir, una afirmación construida a partir de los signos primitivos y los signos lógicos que dependa de una variable \( x \) y que pueda tener otras variables \( x_1,\ldots, x_n \) como parámetros). La diferencia entre "variable" y "parámetro" es puramente subjetiva. Objetivamente tenemos una fórmula con \( n+1 \) variables, sólo que a una de ellas le vamos a dar un trato diferente. A veces escribiremos simplemente \( \phi(x) \) omitiendo la mención explícita de los parámetros. Si algún lector considera esto demasiado abstracto, confío en que los ejemplos que pondremos enseguida resolverán sus dudas.

Consideremos el axioma siguiente:

Axioma de Frege: Para cada fórmula \( \phi(x,x_1,\ldots, x_n) \) se cumple: \( \forall x_1\cdots x_n\exists A\forall x(x\in A\leftrightarrow \phi(x,x_1,\ldots, x_n)) \).

En palabras: Para cada propiedad \( \phi(x) \) (que dependa tal vez de unos parámetros, que pueden ser objetos arbitrarios), existe un conjunto cuyos elementos son exactamente los objetos \( x \) que cumplen \( \phi(x) \).

Una nota técnica:
Spoiler
El axioma de Frege no es en realidad un axioma, sino un esquema axiomático, es decir, una familia de infinitos axiomas: para cada fórmula \( \phi \), tenemos un axioma distinto.
[cerrar]

El axioma de Frege es una adaptación obvia para incluir la posible existencia de átomos del axioma de formación de conjuntos que consideró Frege en la primera teoría axiomática de conjuntos de la historia. Notemos que, combinado con el axioma de extensionalidad, dice que, para cada propiedad \( \phi(x) \) existe un único conjunto \( A \) cuyos elementos son los objetos que cumplen \( \phi(x) \).

Demostración:
Spoiler
Si hubiera dos conjuntos \( A \) y \( B \) que cumplieran esta propiedad, es decir, tales que, para todo objeto \( x \), se cumpliera

\( x\in A\leftrightarrow \phi(x) \)  y  \( x\in B\leftrightarrow \phi(x) \),

entonces tendríamos que, para todo objeto \( x \), se cumpliría \( x\in A\leftrightarrow x\in B \), luego \( A=B \) por el axioma de extensionalidad.
[cerrar]

Por lo tanto, podemos hablar de el conjunto \( A \) cuyos elementos son los objetos que cumplen \( \phi(x) \), y podemos darle un nombre:

Definición: \( \{x\mid \phi(x,x_1,\ldots, x_n)\}\equiv A\mid \forall x(x\in A\leftrightarrow \phi(x,x_1,\ldots, x_n)) \).

Aquí hemos empleado por primera vez un signo lógico que en general es prescindible, pero que en el contexto de NFA resulta ser particularmente útil. Se trata del descriptor \( \mid \).

En general, una expresión de la forma \( x\mid \psi(x,x_1,\ldots, x_n) \) se leerá "el único \( x \) que cumple \( \psi(x,x_1,\ldots, x_n) \)" y sólo podremos emplearlo tras haber justificado que realmente hay un único \( x \) que cumple la propiedad \( \phi(x) \) (la cual puede a su vez depender de otros parámetros).

En nuestro contexto, hemos definido la expresión \( \{x\mid \phi(x,x_1,\ldots, x_n)\} \) como nombre para el único conjunto \( A \) cuyos elementos son los objetos que cumplen \( \phi(x) \).

Por ejemplo, si tomamos \( \phi(x)\equiv (x=x) \) obtenemos el conjunto universal:

\( V=\{x\mid x=x\} \),

cuyos elementos son todos los objetos.

Si tomamos \( \phi(x)\equiv x\neq x \) obtenemos el conjunto vacío:

\( \emptyset\equiv \{x\mid x\neq x\} \),

que no tiene elementos.

Si tomamos \( \phi(x)\equiv x\in A\lor x\in B \) (y aquí tenemos un ejemplo de fórmula con variable \( x \) y dos parámetros \( A \) y \( B \)) obtenemos la unión de conjuntos:

\( A\cup B\equiv\{x\mid x\in A\lor x\in B\} \),

cuyos elementos son los objetos que pertenecen a \( A \) o a \( B \), y de este modo podemos deducir del axioma de Frege la existencia de todos los conceptos conjuntistas básicos.

Pero ¡ay!, que una mente maliciosa puede escoger \( \phi(x)\equiv x\notin x \) y formar el conjunto de Russell:

\( R\equiv \{x\mid \text{cto}x\land x\notin x\} \)

cuyos elementos son los conjuntos que no son elementos de sí mismos. Por ejemplo, \( \emptyset \in R \) y \( V\notin R \), pero ¿\( R\in R \)? Como es bien sabido, el conjunto \( R \) es contradictorio:

Demostración:
Spoiler
Si suponemos que \( R\in R \), entonces \( R \) no cumple la condición precisa para pertenecer a \( R \), que es no pertenecerse a sí mismo, luego debería ser \( R\notin R \), y tenemos una contradicción. Esto debería probar que de hecho \( R\notin R \), pero este caso es igualmente contradictorio, ya que entonces \( R \) cumple la condición que se requiere para pertenecer a \( R \) (que es no pertenecerse a sí mismo), luego debería ser \( R\in R \). En definitiva, es contradictorio tanto suponer que \( R\in R \) como suponer que \( R\notin R \), luego concluimos que el axioma de Frege que permite demostrar la existencia de \( R \) es contradictorio.
[cerrar]

Así pues, la teoría de conjuntos de Frege (que aquí hemos adaptado para que admita átomos, pero eso es irrelevante para lo que estamos diciendo) es contradictoria. A pesar de que es muy útil y práctico, no podemos suponer que cada propiedad define un conjunto, pues al aplicar esto a la propiedad catastrófica \( x\notin x \) caemos en una contradicción irremediable.

Las distintas teorías de conjuntos son propuestas para eludir la paradoja de Russell que acabamos de recordar así como otras más que aparecen si uno postula indiscriminadamente la existencia de demasiados conjuntos.

La primera propuesta fue la de los Principia Mathematica de Russell y Whitehead, que se basa en una lógica de tipos ramificados extremadamente compleja. En su lugar, Zermelo propuso una alternativa mucho más sencilla, consistente en convertir el axioma de formación de conjuntos de Frege en un axioma de selección de subconjuntos. Esto significa que la teoría de Zermelo no postula la existencia de un conjunto

\( \{x\mid \phi(x)\} \)

para cualquier propiedad \( \phi(x) \) (con posibles parámetros), sino únicamente, dado un conjunto arbitrario \( A \) prefijado, postula la existencia del subconjunto

\( \{x\in A\mid \phi(x)\} \)

En otras palabras: las propiedades no definen conjuntos, sino únicamente subconjuntos de conjuntos dados. Este axioma de selección de subconjuntos es mucho más pobre que el axioma de formación de conjuntos de Frege. Ciertamente, no permite definir el fatídico "conjunto" de Russell, pero también nos priva del conjunto universal (la teoría de Zermelo condena al universo a la no existencia, o a una existencia "fantasmagórica", como una clase que es pero no es) y tampoco permite definir otros conjuntos "inocuos" como la unión de conjuntos, el conjunto de partes de un conjunto, el par formado por dos conjuntos dados, conceptos estos que requieren axiomas específicos que postulen su existencia para compensar la debilidad del axioma "atenuado".

En cambio, las teorías como NBG o MK optan por otra estrategia: conservan la esencia del axioma de Frege, pero lo transforman de un axioma de formación de conjuntos en un axioma de formación de clases. Estas teorías distinguen entre "conjuntos genuinos" y "clases propias" que son como conjuntos, pero con limitaciones. Así, toda propiedad define una "clase", pero otros axiomas regulan qué clases son conjuntos y cuales no. En particular, el universo queda relegado a la condición de clase propia (estas teorías "mutilan" el universo).

La idea de Quine, que da lugar a NF y, a su vez a NFA, es conservar el esquema de formación de conjuntos de Frege, sin transformarlo en un esquema de selección de subconjuntos, como Zermelo, y sin atenuar el concepto de "conjunto" para convertirlo en un concepto "light" de clase, como NBG o MK. Lo único que hace Quine es imponer una condición técnica sobre las fórmulas \( \phi(x) \) que definen conjuntos, de modo que la fórmula fatídica \( x\notin x \) no cumple la propiedad en cuestión, y por lo tanto la paradoja de Russell queda "conjurada", pero sí que la cumplen otras muchas fórmulas, como la que define la unión, o el conjunto vacío, o incluso el conjunto universal (NFA salva el universo y lo presenta como un conjunto de pleno derecho).

¿Cuál es esa condición sobre las propiedades posibles que restringe ligeramente el axioma de Frege para condenar el conjunto de Russell y salvar a la vez el universo? De eso nos ocuparemos en el mensaje siguiente.
Título: Formación de conjuntos
Publicado por: Carlos Ivorra en 04 Enero, 2012, 05:18 pm
Finalmente estamos en condiciones de presentar la teoría NFA completa, con todos sus axiomas. Para evitar que quede esparcida en varios mensajes incluyo aquí todo lo anterior:

Definición: Llamaremos NFA a la teoría axiomática cuyo lenguaje formal consta de los signos lógicos más los signos \( \text{cto} \), \( \in \) y \( (\ ,\ ) \), y cuyos axiomas son los siguientes (en los que empleamos el convenio de que las mayúsculas representan conjuntos):

Extensionalidad: \( \forall XY(\forall u(u\in X\leftrightarrow u\in Y)\leftrightarrow X=Y) \)

Pares ordenados: \( \forall xyuv((x,y)=(u,v)\rightarrow x=u\land y=v) \)

Formación de conjuntos: Para cada fórmula estratificada \( \phi(x,x_1,\ldots, x_n) \) del lenguaje de NFA, \( \forall x_1\cdots x_n\exists A\forall x(x\in A\leftrightarrow \phi(x,x_1,\ldots, x_n)) \).

Para que esto tenga sentido, sólo nos falta definir qué hemos de entender por "fórmula estratificada". La finalidad ya la hemos explicado: la estratificación es una propiedad que exigimos a las fórmulas para que definan conjuntos, de modo que así excluimos las fórmulas que dan lugar a contradicciones, como \( x\notin x \), pero permitimos muchas otras, incluso algunas que otras teorías de conjuntos prohiben.

Antes de entrar en la definición de fórmula estratificada, interpretamos el axioma de formación de conjuntos: Lo que afirma es que, para toda propiedad \( \phi(x,x_1,\ldots, x_n) \) que cumpla la propiedad técnica que tenemos pendiente definir, existe un conjunto \( A \) cuyos elementos son los objetos \( x \) (conjuntos o átomos) que cumplen la propiedad (las otras variables representan parámetros que pueden aparecer en la propiedad, como los conjuntos arbitrarios \( A \) y \( B \) cuando consideramos la propiedad \( x\in A\lor x\in B \) que define la unión de conjuntos.

El conjunto \( A \) es único por el mismo motivo explicado en el mensaje anterior, luego podemos nombrarlo con una descripción:

\( \{x\mid \phi(x,x_1,\ldots, x_n)\}\equiv A\mid \forall x(x\in A\leftrightarrow \phi(x,x_1,\ldots, x_n)) \)

En estos términos, el axioma de formación de conjuntos dice que podemos hablar del conjunto \( \{x\mid \phi(x,x_1,\ldots, x_n)\} \) siempre que la propiedad \( \phi \) que aparece en ella esté estratificada, pero en caso contrario nada nos asegura a priori que exista semejante conjunto.

En lugar de definir el concepto de fórmula estratificada definiremos algo ligeramente más general. Para ello debemos distinguir entre fórmulas, que son cadenas de signos que constituyen una afirmación, y términos, que son cadenas de signos que nombran objetos.

Por ejemplo, los axiomas de NFA son fórmulas, al igual que \( x\in Y \) o \( X\subset Y \), mientras que \( x \), \( (x,y) \), \( \{x\mid x=x\} \) son términos, pues son expresiones que nombran objetos.

Para referirnos indistintamente a términos o fórmulas usaremos la palabra expresiones. Cada expresión puede contener varias subexpresiones. Por ejemplo, la expresión (fórmula en este caso) \( (x,y)\in A\lor A\subset B \) contiene las subfórmulas \( (x,y)\in A \) y \( A\subset B \), y la primera de ellas contiene los subtérminos \( (x,y) \) y \( A \), y el primer término contiene como subtérminos a \( x \) y a \( y \), etc.

Definición de expresión estratificada Una expresión del lenguaje de NFA está estratificada si admite una estratificación, donde una estratificación es cualquier criterio que asigne un número entero a cada subtérmino de la expresión dada, al que llamaremos su tipo, de modo que se cumplan las propiedades siguientes:

1) Si un mismo término aparece repetido varias veces en la expresión, el tipo que se le asigne debe ser siempre el mismo.

2) En cada subfórmula \( t_1=t_2 \), los términos \( t_1 \) y \( t_2 \) deben tener el mismo tipo.

3) En cada subfórmula  \( t_1\in t_2 \), el tipo del término \( t_1 \) debe ser exactamente una unidad menor que el del término \( t_2 \).

4) En cada subtérmino \( (t_1,t_2) \) los dos términos \( t_1 \) y \( t_2 \) deben tener el mismo tipo, que a su vez debe coincidir con el tipo de \( (t_1,t_2) \).

5) En cada subtérmino \( x\mid \phi(x,x_1,\ldots, x_n) \) su tipo debe ser el mismo que el tipo de la variable \( x \).

Como es fácil comprender, el concepto de estratificación es crucial en la teoría, así que dedicamos el resto del mensaje a familiarizarnos con él a través de ejemplos y algunas observaciones. A partir del próximo mensaje empezaremos ¡por fin! a trabajar con (o en) NFA.

El ejemplo crucial:

La fórmula \( x\notin x \) no está estratificada.

Demostración:
Spoiler
Tenemos que \( x\notin x\equiv \lnot x\in x \). Para estratificar esta fórmula tendríamos que asignar un tipo a la variable \( x \), que tendría que ser el mismo en sus dos apariciones por la regla 1, pero tendría que ser distinto por la regla 3. Así pues, es imposible asignar tipos en las condiciones que exige la definición de estratificación.
[cerrar]

Por consiguiente, la paradoja de Russell no puede demostrarse en NFA, sus axiomas no afirman que exista el conjunto \( \{x\mid x\notin x\} \), ya que la fórmula que (supuestamente) lo define no está estratificada.

Sólo con el propósito de practicar con el concepto de estratificación, vamos a ver que los axiomas de NFA están estratificados.

Extensionalidad
Spoiler
Indicaremos los tipos mediante subíndices:

\( \forall XY(\forall u(u_0\in X_1\leftrightarrow u_0\in Y_1)\leftrightarrow X_1=Y_1) \)

Tenemos una estratificación que respeta todas las reglas: la \( u \) tiene tipo 0 en todas sus apariciones, la \( X \) tiene tipo 1 en todas sus apariciones, al igual que la \( Y \), las igualdades igualan términos del mismo tipo, mientras que las pertenencias aumentan una unidad el tipo.

Notemos que el uso de mayúsculas oculta dos subfórmulas, a saber, \( \text{cto}\,X \) y \( \text{cto}\,Y \), pero no debemos preocuparnos por ello, porque la definición de estratificación no impone ninguna condición sobre el signo \( \text{cto} \), por lo que la fórmula \( \text{cto}\,X \) está estratificada y no importa si no la escribimos explícitamente al chequear la estratificación.
[cerrar]

Pares ordenados
Spoiler
El axioma de los pares ordenados se estratifica asignando tipo cero a todas las variables:

\( \forall xyuv((x_0,y_0)_0=(u_0,v_0)_0\rightarrow x_0=u_0\land y_0=v_0) \)

Observemos que, como asignamos tipo \( 0 \) a \( x \) y a \( y \), la regla 4) exige que asignemos también tipo \( 0 \) al par \( (x,y) \). Similarmente, el par \( (u,v) \) tiene también tipo \( 0 \), por lo que la igualdad entre los dos pares cumple la condición de que sus dos términos (los dos pares) tengan el mismo tipo.
[cerrar]

Formación de conjuntos
Spoiler
Supongamos que la fórmula \( \phi(x) \) (con posibles parámetros) admite una estratificación en la que la variable \( x \) tiene tipo \( n \). Entonces una estratificación del axioma correspondiente es

\( \exists A\forall x(x_n\in A_{n+1}\leftrightarrow \phi(x_n)) \).

En realidad, para que esta fórmula sea un axioma hay que pedir que la variable \( A \) sea una variable "nueva", es decir, una variable que no aparezca ya en la fórmula \( \phi \). En caso contrario, ni la estratificación que acabamos de construir tendría por qué ser correcta, ni el "axioma" tendría el sentido que pretende tener.
[cerrar]

Un último ejemplo: Una fórmula  \( t_1\subset t_2 \) está estratificada si y sólo si los dos términos admiten una estratificación (conjunta, es decir, que los subtérminos de uno que también estén en el otro tengan el mismo tipo en ambos) en la que ambos tengan el mismo tipo.
Spoiler
Por definición

\( t_1\subset t_2\equiv \text{cto}\,t_1\land \text{cto}\,t_2\land \forall u(u\in t_1\leftrightarrow u\in t_2) \)

Si \( t_1 \) y \( t_2 \) admiten estratificaciones, entonces las fórmulas \( \text{cto}\,t_1 \) y  \( \text{cto}\,t_2 \) estarán estratificadas, porque el signo \( \text{cto} \) no impone ninguna restricción. Incluso podríamos haber omitido esta parte de la fórmula. Respecto a la parte final, si ambos términos admiten una estratificación conjunta que les asigne tipo \( n \), basta asignar tipo \( n-1 \) a la variable \( u \) y tenemos la fórmula completamente estratificada.

Recíprocamente, si la fórmula se puede estratificar, entonces \( t_1 \) y \( t_2 \) admiten una estratificación conjunta en la que ambos tienen tipo una unidad mayor que el asignado a la variable \( u \), luego ambos tienen el mismo tipo.
[cerrar]

Algunas observaciones finales:

Como vemos, NFA tiene únicamente tres axiomas, si bien ya hemos advertido que el segundo es redundante (un poco más adelante explicaré la situación) y el tercero es en realidad un esquema de infinitos axiomas (pero se puede probar que a partir de un número finito de casos particulares se pueden demostrar todos los demás, es decir, que NFA es finitamente axiomatizable). Recordemos también que, por puro perfeccionismo, podríamos añadir consistentemente el axioma de los átomos (el que afirma que los átomos no tienen elementos).

Si hemos insistido en los detalles técnicos de cómo se escriben formalmente los enunciados de NFA es por la necesidad continua que tendremos de comprobar que cada propiedad que usemos para definir un conjunto sea estratificada. Salvo por esta cuestión, a partir de aquí podremos aligerar considerablemente las cuestiones de lógica y pasar a razonar en términos ligeramente más informales.

En cuanto a la noción de estratificación, es preciso que todo aquel que quiera seguir estas explicaciones se familiarice en la práctica con el concepto. En la práctica uno tiene que ser capaz de ver una fórmula y decidir si está o no está estratificada. Siempre es posible hacerlo mediante un proceso finito, pero costaría más explicar un algoritmo general que enfrentarse directamente a cada caso particular. Estratificar es más o menos como resolver un sudoku.

No es necesario que uno adquiera la práctica ya mismo, sino que basta con que estudie atentamente los muchos ejemplos que vamos a ver y que, en cuanto uno se crea capaz de estratificar por sí mismo, que se abstenga de mirar las "soluciones" hasta que considere que ya no merece la pena pararse a hacer cuentas porque las domine sobradamente.

Algunas observaciones generales:

Notemos que todo término de NFA es una variable, un par ordenado o una descripción, por lo que, a la hora de estratificar, sólo tenemos (relativa) libertad para asignar tipos a las variables de la expresión. Una vez cada variable tiene un tipo asignado, todos los demás términos tienen asignado un único tipo por las condiciones 4 y 5.

Si en una estratificación sumamos un mismo número entero a todas las asignaciones de tipos obtenemos otra estratificación igualmente correcta. Esto es porque los términos que tenían el mismo tipo seguirán teniéndolo (luego se seguirá cumpliendo la regla 1), las igualdades que igualaban términos del mismo tipo seguirán haciéndolo (regla 2), las pertenencias seguirán manteniendo una unidad de diferencia entre sus términos, etc.

En particular, a la hora de estratificar una expresión, siempre podemos empezar por asignar un tipo arbitrario a una variable arbitraria, y a partir de ahí ir calculando tipos "obligados" de otras variables por las relaciones que hay entre ellas en la expresión estudiada.

Otra consecuencia del hecho de que podemos elevar simultáneamente todos los tipos en la misma cantidad es que podríamos haber definido el concepto de estratificación admitiendo únicamente tipos naturales en lugar de enteros, puesto que si una estratificación llega a asignar, por ejemplo, hasta el tipo \( -4 \), basta sumar \( 4 \) a todos los tipos para tener otra en la que todos los tipos son naturales. No obstante, el admitir enteros flexibilizará ciertos razonamientos.

Título: Re: NFA
Publicado por: argentinator en 04 Enero, 2012, 11:15 pm
Citar
todos los objetos de los que habla el lenguaje de NFA son objetos

La 1er palabra "objetos" es informal, mientras que la 2da palabra objetos es formal.
Y aunque la 2da aparece en cursiva lo cual la distingue de la 1era,
no deja de parecerme un abuso de lenguaje.

Me dejaría más conforme una expresión así: "todos los entes de los que habla el lenguaje de NFA son objetos".

_________

Citar
Diremos que un objeto es un átomo si no es un conjunto.

Esta definición me "asusta" porque en teorías de conjuntos más estándar como la de Morse-Kelley (MK), lo que no es un conjunto es una clase propia.
Un átomo en NFU pertenece a otros "objetos" (conjuntos) y ningún objeto pertenece a ellos,
mientras que los "no-conjuntos" de MK (clases propias) no pertenecen a conjunto alguno, sino que hay conjuntos que pertenecen a ellas.

En este mismo tema comparo las propiedades que caracterizan a los conjuntos en ambas teorias:

En MK: \( \forall xy(x\in y\rightarrow \text{cto}\,x) \)
En NFU: \( \forall xy(x\in y\rightarrow \text{cto}\,y) \)

___________

No entiendo el axioma de que todos los objetos son conjuntos.
En realidad no entiendo la notación: \( \forall x\ \text{cto}\,X \).
¿No debieran ser ambas "x" minúsculas?
O sea, ¿no son el mismo "x"?

\( \forall x\ \text{cto}\,x \)

___________

Tampoco le veo sentido a hablar de átomos y conjuntos, si es que no va a haber átomos.

Pero comprendo que el Axioma NA no es obligatorio, sino un axioma que puede agregarse luego, quién sabe para qué fines (¿preguntarse el parecido con ZFC?).

___________

Citar
¿Cuál es esa condición sobre las propiedades posibles que restringe ligeramente el axioma de Frege para condenar el conjunto de Russell y salvar a la vez el universo? De eso nos ocuparemos en el mensaje siguiente.

Hasta acá la "saga" de posts es intrigante.
A ver qué pasa en el siguiente post!!!  :o

___________

En general la exposición es muy clara, así que estoy contento pues creo haber entendido de qué se trata el asunto.

___________

En cuanto a la noción de "fórmula estratificada", tengo una duda, aunque no he reflexionado mucho sobre ella, y además es posible que no entiendo qué hacer.

En ZFC hay conjuntos (típicamente los ordinales) en que es posible decir cosas como:

\( (x\in y)\wedge (x\subset y) \)

¿Esto no atenta contra las reglas de estratificación?

O sea, si los elementos de un conjunto tienen varios grados de pertenencia e inclusión entre sí, ¿es posible definirlos adecuadamente en NFU (NFA)?

Título: Re: NFA
Publicado por: Carlos Ivorra en 04 Enero, 2012, 11:51 pm
Hola, argentinator.

Gracias por tu mensaje. En general, agradecería a los lectores que me comunicaran todo lo que les resultara oscuro o que les suscitara cualquier clase de duda, o cualquier observación, como ha hecho argentinator, para tener la ocasión de aclarar lo que resulte necesario.

Citar
todos los objetos de los que habla el lenguaje de NFA son objetos

La 1er palabra "objetos" es informal, mientras que la 2da palabra objetos es formal.
Y aunque la 2da aparece en cursiva lo cual la distingue de la 1era,
no deja de parecerme un abuso de lenguaje.

Me dejaría más conforme una expresión así: "todos los entes de los que habla el lenguaje de NFA son objetos".

Estoy de acuerdo. Ya lo he corregido.


Citar
Diremos que un objeto es un átomo si no es un conjunto.

Esta definición me "asusta" porque en teorías de conjuntos más estándar como la de Morse-Kelley (MK), lo que no es un conjunto es una clase propia.
Un átomo en NFU pertenece a otros "objetos" (conjuntos) y ningún objeto pertenece a ellos,
mientras que los "no-conjuntos" de MK (clases propias) no pertenecen a conjunto alguno, sino que hay conjuntos que pertenecen a ellas.

En este mismo tema comparo las propiedades que caracterizan a los conjuntos en ambas teorias:

En MK: \( \forall xy(x\in y\rightarrow \text{cto}\,x) \)
En NFU: \( \forall xy(x\in y\rightarrow \text{cto}\,y) \)

Yo creo que esta analogía es muy superficial (no digo que no sea natural planteársela, sino que, cuando uno se pone a pensar sobre ella, se da cuenta de que en el fondo son como sardinas y delfines, que se parecen, pero en el fondo no tienen nada que ver).

Fíjate que también podrías plantear una teoría MK con átomos, con lo que tendrías conjuntos, átomos y clases propias. Un átomo es un "objeto primitivo" (como las manzanas con las que puedes formar conjuntos de manzanas sin ser ellas mismas conjuntos), un conjunto es una colección de objetos, y una clase propia es un conjunto demasiado grande como para meterlo en otros conjuntos. Claro: en una teoría en la que sólo hay conjuntos y átomos, puedes definir un átomo como algo que no es un conjunto y no te complicas más la vida; en una teoría en la que sólo hay conjuntos y clases propias puedes definir una clase propia como una clase que no es un conjunto y no te complicas más la vida, pero eso es como si ves a un negro al lado de un blanco y dices que el negro es el que no es blanco, y luego ves a un cobrizo al lado de un blanco y dices que el cobrizo es el que no es blanco. Eso no significa que haya ninguna relación de identidad entre el negro y el cobrizo.

Si quisieras una teoría que incluyera átomos, conjuntos y clases, tendrías que introducir al menos dos signos básicos, digamos cto y atm y definir una clase como un objeto que no es un conjunto ni un átomo. Y nadie vería entonces ninguna relación entre clases y átomos.

No entiendo el axioma de que todos los objetos son conjuntos.
En realidad no entiendo la notación: \( \forall x\ \text{cto}\,X \).
¿No debieran ser ambas "x" minúsculas?
O sea, ¿no son el mismo "x"?

\( \forall x\ \text{cto}\,x \)

La X mayúscula era una errata. Ya la he corregido. Gracias. Si era eso lo que no entendías ya está, pero si no era eso, el axioma NA afirma que no hay átomos, que todos los objetos son conjuntos, es decir, que la situación es la que se da en ZFC o en MK, donde todos los objetos son conjuntos, y los elementos de los conjuntos son más conjuntos. Pero fíjate que NA no es un axioma de NFA, al contrario, si añades NA a NFA obtienes la teoría NF, teoría que podrías introducir más fácilmente eliminando el signo cto del lenguaje y tomando el mismo axioma de extensionalidad de ZFC (dos objetos=conjuntos con los mismos elementos son iguales).


Tampoco le veo sentido a hablar de átomos y conjuntos, si es que no va a haber átomos.

Claro, claro, eso es lo que acabo de decir, que, si quisiéramos añadir el axioma NA, sería más fácil no haber hablado de átomos desde un principio. Nosotros vamos a trabajar en NFA, donde sí que hay átomos (o, por lo menos, no se afirma que no los haya, no se sabe si es consistente que no los haya, pero sí que se pueden construir modelos de NFA en los que de hecho los hay). El axioma NA sólo sirve para relacionar lo que digamos con lo que puede decirse en la teoría NF. La verdad es que si no lo hubiera nombrado no se habría perdido nada importante.

Pero eso que quede claro: en NFA nadie dice que no haya átomos. De hecho, si suponemos el axioma de elección, se puede demostrar que sí que hay átomos.

Pero comprendo que el Axioma NA no es obligatorio, sino un axioma que puede agregarse luego, quién sabe para qué fines (¿preguntarse el parecido con ZFC?).

Casi. Más bien comparar con la teoría original de Quine, NF. Pero, como ya digo, es más una curiosidad secundaria. No hay ninguna razón que yo conozca por la que sea interesante añadir el axioma NA a la teoría.

En general la exposición es muy clara, así que estoy contento pues creo haber entendido de qué se trata el asunto.

Gracias. De todos modos, creo que ésta es la parte más desagradable, porque la necesidad de definir la estratificación nos obliga a no alejarnos mucho del lenguaje formal. En cuanto sepamos manejar la estratificación con soltura, la exposición podrá hacerse menos técnica y más fluida (espero).

La impresión que tengo por tus observaciones es que has entendido muy certeramente todos los matices.

En cuanto a la noción de "fórmula estratificada", tengo una duda, aunque no he reflexionado mucho sobre ella, y además es posible que no entiendo qué hacer.

En ZFC hay conjuntos (típicamente los ordinales) en que es posible decir cosas como:

\( (x\in y)\wedge (x\subset y) \)

¿Esto no atenta contra las reglas de estratificación?

O sea, si los elementos de un conjunto tienen varios grados de pertenencia e inclusión entre sí, ¿es posible definirlos adecuadamente en NFU (NFA)?

Esto confirma lo que acabo de decir, pues es una observación muy perspicaz. En efecto, la definición de ordinal (de ZF o MK) no está estratificada, la teoría de ordinales de von Neumann no funciona bien en NFA, pero en su lugar tenemos una definición alternativa de ordinal que funciona perfectamente y es mucho más fiel a la idea original: un ordinal se define como una clase de equivalencia de conjuntos bien ordenados respecto a la relación de isomorfismo. Eso en ZFC o en MK no funciona bien porque se trataría de una relación de equivalencia sobre una clase propia, y cada clase de equivalencia sería una clase propia, pero en NFA el conjunto de todos los conjuntos bien ordenados (entendidos como pares \( (A, \leq) \)) es un conjunto y no hay ninguna pega en definir los ordinales como clases de equivalencia, igual que definiremos el número natural 1 como el conjunto de todos los conjuntos de la forma \( \{x\} \), y el 2 como el conjunto de todos los conjuntos de la forma \( \{x,y\} \), con \( x\neq y \), etc.
Título: El álgebra de conjuntos
Publicado por: Carlos Ivorra en 05 Enero, 2012, 01:32 am
Ha llegado el momento de empezar a usar NFA:

La fórmula \( x=x \) es trivialmente estratificada (no creo que merezca poner la prueba en spoiler), luego podemos definir el conjunto universal:

\( V\equiv \{x\mid x=x\} \).

Así, \( V \) es, no el conjunto de todos los conjuntos, sino el conjunto de todos los objetos, pues contiene a todos los conjuntos y a todos los átomos. Nada nos impide definir el conjunto de todos los conjuntos y el conjunto de todos los átomos:

\( C\equiv \{x\mid \text{cto}\,x\}\qquad A\equiv \{x\mid \lnot \text{cto}\,x\} \)

Esto es correcto porque las fórmulas correspondientes están estratificadas (recordemos que la definición de estratificación no impone ninguna restricción al signo \( \text{cto} \)).

Igualmente podemos definir el conjunto vacío:

\( \emptyset\equiv \{x\mid x\neq x\} \)

Una nota técnica:
Spoiler
Alguien podría pensar que podríamos haber incorporado el axioma de los átomos a la teoría a la vez que eliminábamos el signo primitivo cto definiendo un átomo como un objeto sin elementos y un conjunto como un objeto con elementos. Eso no es conveniente porque nos interesa que la teoría tenga un conjunto vacío, es decir, un objeto que no tiene elementos y, a pesar de ello, no es un átomo, sino un conjunto. El axioma de extensionalidad implica que el conjunto vacío es el único conjunto sin elementos, pues dos conjuntos vacíos serían dos conjuntos con los mismos elementos (ninguno) y no puede haber dos conjuntos distintos con los mismos elementos. En cambio, el axioma de extensionalidad no afecta a los átomos, por lo que nada impide que haya muchos átomos distintos y que ninguno tenga elementos.
[cerrar]

Ahora definimos las operaciones conjuntistas básicas (no explico su significado porque son de sobra conocidas):

\( A\cup B \equiv \{x\mid x\in A\lor x\in B\},\qquad A\cap B \equiv \{x\mid x\in A\land x\in B\}, \)

\( A\setminus B\equiv \{x\mid x\in A\land x\notin B\},\qquad \overline A=\{x\mid x\notin A\} \).

Observemos en particular la última definición, que no es posible en ZFC (o, al menos, da lugar a una clase propia): el conjunto complementario de un conjunto \( A \) es el conjunto de todos los objetos que no están en \( A \). Una definición alternativa sería: \( \overline A =V\setminus A \), aunque también podríamos haber definido \( A\setminus B = A\cap \overline B \).

El lector no debería tener dificultad en probar que todas las fórmulas que hemos usado están estratificadas. Por si acaso pongo un spoiler con las estratificaciones:

Spoiler
\( x_0\in A_1\lor x_0\in B_1 \)

\( x_0\in A_1\land x_0\in B_1 \)

\( x_0\in A_1\land \lnot x_0\in B_1 \)

\( \lnot x_0\in A_1 \)
[cerrar]

A partir de estas definiciones se pueden probar todas las relaciones usuales entre estos conceptos exactamente igual a como se hace en ZFC, con la ventaja incluso de que disponemos de un conjunto universal (hemos salvado el universo), de modo que los conjuntos forman una auténtica álgebra de Boole.

Cabe destacar algunas relaciones obvias que pueden "chirriar" a una mente clásica: \( V\in C,\quad C\in V, \quad V\in V,\quad C\in C \).

También podemos definir las grandes uniones e intersecciones:

\( \bigcup X\equiv \{x\mid \exists Y\in X\ x\in Y\},\qquad \bigcap X\equiv \{x\mid \forall Y\in X\ x\in Y\} \).

Así, \( \bigcup X \) es el conjunto cuyos elementos son todos los elementos de los conjuntos que pertenecen al conjunto \( X \) (si \( X \) contiene átomos, éstos no afectan a la definición de \( \bigcup X \), en el sentido de que el conjunto \( X\setminus A \) tiene la misma gran unión). Similarmente, \( \bigcap X \) es el conjunto cuyos elementos son todos los objetos que pertenecen a todos los conjuntos que pertenecen a \( X \). En otras palabras, la gran unión es la unión de todos los conjuntos que pertenecen a \( X \) y la gran intersección es la intersección de todos los conjuntos que pertenecen a \( X \).

Unas cuantas preguntas sobre estos dos conceptos:

1) Probar que las fórmulas que los definen están estratificadas.

2) Probar que los términos \( \bigcup X \) y \( \bigcap X \) están estratificados, ¿cuál es su tipo respecto del tipo de \( X \)?

3) ¿Quiénes son \( \bigcup \emptyset \) y \( \bigcap \emptyset \)?

Respuestas:
Spoiler
1) Unas estratificaciones para las fórmulas que los definen son:

\(  \exists Y_1\in X_2\ x_0\in Y_1 \)

\( \forall Y_1\in X_2\ x_0\in Y_1 \)

2) Unas estratificaciones para los términos son:

\( \bigcup X\equiv \{x_0\mid \exists Y_1\in X_2\ x_0\in Y_1\}_1,\qquad \bigcap X\equiv \{x_0\mid \forall Y_1\in X_2\ x_0\in Y_1\}_1 \)

Aquí hemos usado que, en general,

\( \{x\mid \phi(x)\}\equiv A\mid\forall x(x\in A\leftrightarrow \phi(x)) \)

y, si la fórmula \( \phi(x) \) está estratificada con una estratificación que asigna tipo \( n \) a la variable \( x \), entonces el término está estratificado asignando a la variable \( A \) el tipo \( n+1 \) (por la propiedad 3 de la definición de estratificación) y asignando a todo el término (es decir, a la descripción) el mismo tipo que a la variable \( A \) (por la propiedad 5), luego, en general:

El tipo de un término \( \{x\mid \phi(x)\} \), donde \( \phi(x) \) es una fórmula estratificada, es una unidad mayor que el tipo de la variable \( x \) en la estratificación de \( \phi \).

En nuestros ejemplos, como la variable \( x \) tiene tipo \( 0 \), la gran unión y la gran intersección tienen ambas tipo \( 1 \), y así lo expresa el subíndice de más a la derecha.

La respuesta a la segunda pregunta es que el tipo de \( \bigcup X \) y \( \bigcap X \) es siempre una unidad inferior al tipo de \( X \). En la estratificación concreta que hemos considerado \( X \) tiene tipo \( 2 \) y la unión y la intersección tienen tipo \( 1 \), pero podríamos haber asignado arbitrariamente tipo \( n \) a la variable \( X \) y el tipo de la unión y la intersección habría sido \( n-1 \), como es fácil comprobar. En otras palabras: la gran unión y la gran intersección rebajan el tipo una unidad.

3) Esto es un ejercicio de la "lógica del vacío":

\( \bigcup\emptyset = \emptyset,\quad \bigcap\emptyset =V \).

En efecto, para que un objeto perteneciera a \( \bigcup \emptyset \) tendría que pertenecer a un conjunto perteneciente a \( \emptyset \). Como no hay tal conjunto, la unión es vacía.

Para que un objeto no perteneciera a \( \bigcap\emptyset \), debería haber un conjunto en \( \emptyset \) al cual no perteneciera, como eso es imposible, todo objeto está en la intersección.
[cerrar]

Otro concepto conjuntista básico es

\( \mathcal PA\equiv \{X\mid X\subset A\} \)

Claramente, se trata del conjunto de todos los subconjuntos de \( A \). Ya vimos en el mensaje anterior que la fórmula \( X\subset A \) está estratificada. ¿Cuál es el tipo de \( \mathcal PA \) en función del tipo de \( A \)?

Respuesta:
Spoiler
Si asignamos a \( A \) el tipo \( 0 \), entonces \( X \) tiene también tipo \( 0 \) y, por la observación general hecha en el spoiler anterior, las llaves \( \{X\mid \cdots\} \) aumentan en una unidad el tipo de \( X \), luego el tipo de \( \mathcal PA \) es \( 1 \) o, más en general, es una unidad superior al tipo de \( A \).
[cerrar]

Naturalmente, todas las propiedades básicas sobre los conceptos que acabamos de introducir (por ejemplo, \( X\subset Y\rightarrow \mathcal PX\subset \mathcal PY \)) se demuestran exactamente igual que en ZFC con los mismos argumentos.

Una relación curiosa es que \( C=\mathcal PV \). ¿Cuál es la interpretación de \( F\equiv\mathcal P\mathcal PV \)?

Respuesta:
Spoiler
\( F \) es el conjunto de todas las familias de conjuntos, es decir, el conjunto de todos los conjuntos cuyos elementos son conjuntos (el conjunto de todos los conjuntos que no contienen átomos).
[cerrar]

En el próximo mensaje hablaremos de pares desordenados, pares ordenados y productos cartesianos, y trataré de explicar por qué ha sido conveniente tomar el concepto de par ordenado como concepto primitivo.
Título: Re: NFA
Publicado por: Raúl Aparicio Bustillo en 05 Enero, 2012, 11:31 am
Bueno, primero, decir que  "todo un honor"  haber contribuido a liar a Ivorra en NFU (o  NFA). Lo de rebautizar la teoría como NFA  me parece prudente,  simplemente por ser conciso: estamos prescindiendo del axioma de átomos. La apuesta de Ivorra es bastante fuerte, en principio pensaba que el axioma era eliminado de la teoría porque era demostrable a partir de los otros 3. Al releer los post me he dado cuenta de que no es eso lo que quiere decir. Lo destierra simplemente porque considera que los teoremas utiles se pueden deducir sin el. En cualquier caso, tampoco hay mucho  que discutir: Ivorra nos habla de una teoría que es NFA, deja claro su lenguaje, deja claro que trabajamos en logica de primer orden, y  deja claros sus 3  axiomas, el resto ya es demostrar y demostrar,y si llegasemos a una contradicción en NFA no se salvan ni NFU ni ZFC.

Una  puntualización (no necesariamente una corrección)  a raiz de un comentario de Argentinator:

 El axioma de estratificación dice que las formulas estratificadas necesariamente definen conjuntos, pero eso no implica que algunos objetos sólo definibles mediante formulas no estratificadas no puedan ser conjuntos. No es demasiado claro Holmes en su libro, y es aquí donde mi puntualización ya se transforma en  una duda: Cuando se refiere a los ordinales de Von Neumann Holmes afirma que podemos considerar que existen o que no existen. Creo que lo que trata de decir (evitandose usar la palabra "modelo") es que NFU tiene modelos con  ordinales de Von Neumann, y otros sin ellos;y  que por tanto es conveniente definir los ordinales de otra forma, que es la que comenta Ivorra, para asegurarnos que existen en la teoría (o sea, que existen en todos los modelos de la teoría). Lo que pasa es que eso de modelos sin ordinales de Von Neumann no es exactamente así: es trivial que el ordina de Von Neumann 0 (el conjunto vacio) ha de existir en todos los modelos de NFU. Me aventuraría a decir que los ordinales finitos también existen en todos los modelos de NFU. Pero de ser así ya me pierdo totalmente y no tengo nada claro que es eso de que "pueden existir pero también pueden no existir" en NFU.
Título: Re: NFA
Publicado por: Carlos Ivorra en 05 Enero, 2012, 11:54 am
Bueno, primero, decir que  "todo un honor"  haber contribuido a liar a Ivorra en NFU (o  NFA).

La verdad es que me alegro de haber conocido NFA y eso te lo debo a ti.

Lo de rebautizar la teoría como NFA  me parece prudente,  simplemente por ser conciso: estamos prescindiendo del axioma de átomos. La apuesta de Ivorra es bastante fuerte, en principio pensaba que el axioma era eliminado de la teoría porque era demostrable a partir de los otros 3. Al releer los post me he dado cuenta de que no es eso lo que quiere decir. Lo destierra simplemente porque considera que los teoremas utiles se pueden deducir sin el.

Sí, más o menos, pero, para ser más exactos: todos los teoremas que demuestra Holmes en su libro se demuestran sin el axioma de los átomos, y sin tener que hacer ningún esfuerzo para evitarlo, es que simplemente en ningún momento te hace falta para nada. No es que yo diga: "fíjate que gracias a este truco y a este otro podemos arreglárnoslas para prescindir del axioma de los átomos", sino que nunca tienes la necesidad de recurrir a él.

En cualquier caso, tampoco hay nada que discutir: Ivorra nos habla de una teoría que es NFA, da explicitamente los axiomas, y desde luego, es una teoría que es más debil que ZFC y (quizás) más debil tambien que NFU.

Añadir el axioma de los átomos no vuelve a la teoría "más fuerte", en el sentido de que todo modelo de NFA sin el axioma de los átomos se convierte en un modelo de NFA más el axioma de los átomos sin más que modificar la relación de pertenencia para "borrar" los posibles elementos de los átomos.

Una  puntualización (no necesariamente una corrección)  a raiz de un comentario de Argentinator:

El axioma de estratificación dice que las formulas estratificadas necesariamente definen conjuntos, pero eso no implica que algunos objetos sólo definibles mediante formulas no estratificadas no puedan ser conjuntos. No es demasiado claro Holmes en su libro, y es aquí donde mi puntualización ya se transforma en  una duda: Cuando se refiere a los ordinales de Von Neumann Holmes afirma que podemos considerar que existen o que no existen. Creo que lo que trata de decir (evitandose usar la palabra "modelo") es que NFU tiene modelos con  ordinales de Von Neumann, y otros sin ellos;y  que por tanto es conveniente definir los ordinales de otra forma, que es la que comenta Ivorra, para asegurarnos que existen en la teoría (o sea, que existen en todos los modelos de la teoría). Lo que pasa es que esto no es exactamente así: es trivial que el ordinal 0 (el conjunto vacio) ha de existir en todos los modelos de NFU. Me aventuraría a decir que los ordinales finitos también existen en todos los modelos de NFU. Pero de ser así ya me pierdo totalmente y no tengo nada claro que es eso de que "pueden existir pero también pueden no existir" en NFU.

El concepto de ordinal de von Neumann se puede definir en toda teoría de conjuntos entre cuyos teoremas se encuentren el axioma de extensionalidad, la existencia de la unión de dos conjuntos, del complemento de dos conjuntos, del par desordenado de dos conjuntos y del conjunto vacío. No hace falta nada más, en particular no hace falta ningún axioma de formación de conjuntos o subconjuntos a partir de fórmulas.

Todos estos resultados son teoremas de NFA, luego en NFA puedes definir el concepto de ordinal (de von Neumann) e incluso el de número natural (de von Neumann, de modo que \( 0=\emptyset, 1 = \{0\}, 2=\{0,1\} \), etc.)

Ahora bien, las fórmulas "x es un ordinal" o "x es un número natural" (en este sentido de von Neumann) no están estratificadas y, por ello dan muy poco juego. En NFA no es posible demostrar que exista el conjunto de los números naturales en este sentido (pero sí que se pueden definir de otro modo, estratificado, y todo va bien).

Observemos que esto no es una desventaja de NFA frente a ZFC, sino que la situación es simétrica: hay una posible forma de definir los números naturales que en ZFC va perfectamente y en NFA va muy mal, y otra forma de definir los números naturales que en NFA va perfectamente y en ZFC va muy mal (haría que cada número natural fuera una clase propia).

En definitiva, en NFA no puedes demostrar la existencia del ordinal de von Neumann \( \omega \) (aunque puedes definir un ordinal \( \omega \) equivalente de otro modo), pero, por otra parte, puedes tomar como axioma adicional que existe \( \omega \) (es decir, tomar como axioma que exista un conjunto cuyos elementos sean los números naturales de von Neumann), y la teoría sigue siendo consistente (supuesto que ZFC lo sea).

Por otra parte, la verdad es que en ZFC tampoco puedes demostrar la existencia de \( \omega \), sino que esto es un axioma de ZFC. En NFA puedes construir los números naturales sin necesidad de postular la existencia de \( \omega \). (Bueno, ahí hay ciertas sutilezas, pero es pronto para entrar en ellas.)
Título: Pares y productos cartesianos
Publicado por: Carlos Ivorra en 05 Enero, 2012, 01:05 pm
Otros conceptos elementales que en NFA se definen sin dificultad son:

\( \{x\}\equiv \{u\mid u=x\},\qquad \{x,y\}\equiv \{u\mid u=x\lor u=y\} \).

Es evidente que las fórmulas que los definen están estratificadas, con todas las variables, por ejemplo, de tipo \( 0 \).

Notemos que el criterio de que "las llaves \( \{\cdots \} \) aumentan el tipo una unidad" se cumple también en este caso:

Si \( t \) es cualquier término estratificado, entonces \( \{t\} \) también está estratificado, y su tipo es una unidad más que el tipo de \( t \).

(Pues \( \{t\}\equiv \{u\mid u=t\} \), luego, partiendo de la estratificación de \( t \), hemos de asignar a la variable \( u \) el mismo tipo que a \( t \) y a \( \{t\} \) una unidad más que a \( u \).)

Similarmente:

Un término de la forma \( \{t_1,t_2\} \) está estratificado si y sólo si los términos \( t_1 \) y \( t_2 \) admiten una estratificación conjunta (es decir, tal que los subtérminos comunes tengan siempre el mismo tipo) que les asigne a los dos el mismo tipo, y entonces el tipo de \( \{t_1,t_2\} \) es una unidad mayor.

Ahora es cuando todo aquel familiarizado con ZFC señala que podemos definir:

\( (x,y)^*\equiv \{\{x\},\{x,y\}\} \).

Pongo una estrella para distinguir esta definición del par ordenado \( (x,y) \) que hemos tomado como signo primitivo. Es muy fácil probar que

\( \forall xyuv\,((x,y)^*=(u,v)^*\leftrightarrow x=u\land y=v) \)

Y entonces ¿para qué hemos introducido el par ordenado como signo primitivo y para qué hemos incluido el axioma de los pares ordenados entre los axiomas de NFA cuando teníamos la posibilidad de definir así el par ordenado y demostrar el axioma asociado?

La respuesta es que el par ordenado \( (x,y)^* \) tiene una característica que lo distingue de \( (x,y) \) y lo hace mucho menos conveniente que éste. Observemos que, por las observaciones precedentes sobre la estratificación de los pares desordenados, una estratificación del par que acabamos de definir es

\( (x,y)^*\equiv \{\{x_0\}_1,\{x_0,y_0\}_1\}_2 \)

de modo que el tipo de \( (x,y)^* \) es dos unidades superior al de \( x \) e \( y \). Más en general, un término \( (t_1,t_2)^* \) está estratificado si y sólo si los dos términos \( t_1 \) y \( t_2 \) admiten una estratificación conjunta que les asigne el mismo tipo, y entonces el tipo del par es dos unidades mayor que el tipo de sus componentes.

En pocas palabras: el par ordenado \( (x,y)^* \) aumenta dos unidades el tipo de sus componentes, mientras que, por la propia definición de estratificación, el par ordenado \( (x,y) \) tiene el mismo tipo que sus componentes. Esto se expresa diciendo que el término \( (x,y) \) es un par ordenado nivelado. Así pues:

Lo que afirma el axioma de los pares ordenados es que existe un par ordenado nivelado, es decir, un par ordenado que no aumenta el tipo de sus componentes. El par ordenado que se define en ZFC es un par ordenado no nivelado, luego no es equivalente en NFA al que hemos tomado como signo primitivo (no se puede hacer lo mismo con ambos).

Para ver las ventajas de contar con un par ordenado nivelado tendremos que esperar a hablar de funciones. Ahora bien, sucede que, pese a todo, aunque no hubiéramos tomado el par ordenado como signo primitivo, en NFA habríamos podido demostrar la existencia de pares ordenados nivelados (aquí hay ciertos matices que explicaré más adelante). El proceso habría sido el siguiente:

1) No suponemos la existencia de pares ordenados nivelados.

2) Definimos \( (x,y)\equiv \{\{x\},\{x,y\}\} \) y con este concepto de par ordenado (estratificado, pero no nivelado) definimos los conceptos de relación, aplicación, etc., desarrollamos la teoría de cardinales, números naturales, etc. (para lo cual hay que hacer varios malabarismos por culpa de la estratificación)

3) Cuando tenemos suficiente teoría a nuestra disposición, demostramos que existen pares ordenados nivelados.

4) Entonces volvemos atrás y decimos: todo lo que hemos hecho hasta ahora con pares ordenados no nivelados, se puede hacer también con pares ordenados nivelados, pero más fácil, así que redefinimos los conceptos de relación, aplicación, etc. para considerar que son los definidos a partir de pares nivelados, volvemos hasta el punto al que habíamos llegado y a partir de aquí continuamos la teoría con pares ordenados nivelados.

Así, el postular desde un principio la existencia de pares nivelados nos ahorra las estrecheces de trabajar con pares no nivelados y la necesidad de rehacer todo lo hecho una vez disponemos de los pares nivelados.

Una última observación a este respecto es que en NF (es decir, si suponemos que no hay átomos, se puede demostrar la existencia de pares nivelados sin usar el axioma de elección (a la fuerza, porque el axioma de elección es contradictorio en NF), mientras que en NFA se necesita el axioma de elección, pero esto es lógico que sea así: en NFA tenemos un conjunto de átomos del que no sabemos nada, así que, ¿cómo vamos a emparejar "finamente" unos objetos de los que no sabemos nada sin realizar elecciones carentes de criterio explícito? No podemos hacer elecciones explícitas sobre unos objetos de los que no sabemos absolutamente nada, por lo que el axioma de elección se vuelve imprescindible. En este sentido se podría decir que al postular la existencia de pares nivelados estamos postulando un caso particular del axioma de elección, pero eso es una verdad a medias. También se podría pensar que estamos diciendo algo sobre los átomos: no sabemos nada sobre ellos salvo que se pueden emparejar bien, lo cual es un requisito razonable.

Definimos

\( A\times B\equiv\{x\mid \exists ab(a\in A\land b\in B\land x=(a,b))\} \)

El lector que quiera seguir esto en serio debería plantearse si es capaz de justificar que la fórmula que define el producto está estratificada, así como comprobar que el producto es un término estratificado. ¿Cuál es su tipo en relación al de \( A \) y \( B \)?

Solución:
Spoiler
Una estratificación para la fórmula que define el producto es

\( \exists ab(a_0\in A_1\land b_0\in B_1\land x_0=(a_0,b_0)_0) \).

A su vez, una estratificación de producto es:

\( A\times B\equiv\{x_0\mid \exists ab(a_0\in A_1\land b_0\in B_1\land x_0=(a_0,b_0)_0)\}_1 \)

luego vemos que \( A\times B \) tiene el mismo tipo que \( A \) y \( B \). Más en general, si \( A \) y \( B \) son términos cualesquiera, no necesariamente meras variables, entonces el término \( A\times B \) está estratificado si y sólo si \( A \) y \( B \) admiten una estratificación conjunta que les asigne el mismo tipo, y entonces ese mismo tipo es también el tipo de \( A\times B \).
[cerrar]

Sólo a modo de ejercicio (sin que nos vaya a aprovechar para nada más adelante): Si definimos el producto cartesiano \( A\times B \) con pares ordenados no nivelados \( (a,b)^* \), ¿cuál sería ahora la relación entre el tipo de \( A\times B \) y el tipo de cada factor?

Respuesta:
Spoiler
El tipo de \( A\times B \) sería dos unidades superior a los tipos de \( A \) y \( B \)
[cerrar]

A veces, a la hora de definir subconjuntos de productos cartesianos, conviene emplear expresiones de la forma:

\( \{(a,b)\mid \phi(a,b,x_1,\ldots, x_n)\}\equiv \{x\mid \exists ab(x=(a,b)\land \phi(a,b,x_1,\ldots, x_n))\} \)

Observemos que esto es correcto siempre y cuando la fórmula \( \phi \) admita una estratificación que asigne el mismo tipo a las variables \( a \) y \( b \), pues esa es la condición necesaria y suficiente para que la fórmula \( \exists ab(x=(a,b)\land \phi(a,b,x_1,\ldots, x_n)) \) esté estratificada. En tal caso, el término que acabamos de definir está estratificado y su tipo es una unidad mayor que el de las variables \( a \) y \( b \).

Para terminar una cuestión "para sobresaliente":

Problema: La fórmula \( x\in A\lor x\in \{A\} \) no está estratificada. Entonces, dado un conjunto \( A \), ¿podemos afirmar que existe el conjunto \( A\cup\{A\} \)?

Solución:
Spoiler
Por definición:

\( A\cup \{A\}\equiv \{x\mid x\in A\lor x\in \{A\}\}\equiv Z\mid \forall x(x\in Z\leftrightarrow x\in A\lor x\in \{A\}) \).

Como la fórmula no está estratificada, no podemos usar el axioma de formación de conjuntos para asegurar que exista el conjunto \( A\cup \{A\} \). Sin embargo, esto es un ejemplo de que, aunque el axioma de formación de conjuntos sólo afirma que las fórmulas estratificadas definen conjuntos, eso no impide que una fórmula no estratificada pueda definir igualmente un conjunto. Para probar que así sucede en nuestro ejemplo consideramos la fórmula estratificada

\( x\in A\lor x\in B \)

y le aplicamos el axioma de formación de conjuntos, según el cual:

\( \forall AB\exists Z\forall x(x\in Z\leftrightarrow x\in A\lor x\in B) \).

Ahora, dado un conjunto \( A \), tenemos que \( \{A\} \) es otro conjunto, luego la afirmación anterior, que vale para todo \( A \) y para todo \( B \), vale en particular para \( A \) y \( \{A\} \), es decir:

\( \exists Z\forall x(x\in Z\leftrightarrow x\in A\lor x\in \{A\}) \),

luego, al fin y al cabo, sí que existe el conjunto \( A\cup \{A\} \).

Ahora bien, esto no significa que, cuando en el mensaje anterior hemos afirmado que existe la unión de conjuntos, dicha afirmación no fuera totalmente general, y que en realidad haga falta considerar aparte casos como éste. He hecho estos razonamientos para mostrar explícitamente un caso de fórmula no estratificada que define un conjunto, pero no hacía falta razonar así. El razonamiento directo es el siguiente:

1) Definimos \( A\cup B \equiv \{x\mid x\in A\lor x\in B\} \).

2) Puesto que la fórmula \( x\in A\lor x\in B \) (donde \( x, A, B \) son variables) está estratificada, el axioma de formación de conjuntos (junto con el de extensionalidad, que da la unicidad) nos asegura que la definición anterior es correcta, en el sentido de que

\( \forall AB\forall x(x\in A\cup B\leftrightarrow x\in A\lor x\in B) \).

Esto, así de general, es un teorema de NFA, y la respuesta al problema planteado se sigue inmediatamente de aquí: la respuesta es afirmativa porque basta hacer \( B=\{A\} \) en este teorema.

En resumen: cuando demostramos que existe la unión de conjuntos (o cualquier otro concepto a partir del axioma de formación de conjuntos) demostramos que existe para cualquier valor de los parámetros \( A \) y \( B \), luego en particular existe cuando sustituimos \( A \) y \( B \) por términos que no admiten una estratificación conjunta, como \( A \) y \( \{A\} \), de modo que eso sólo se traduce en que el término \( A\cup\{A\} \) no está estratificado, pero no en que no exista la unión, cuya existencia tenemos garantizada para todo par de conjuntos.
[cerrar]
Título: Funciones
Publicado por: Carlos Ivorra en 05 Enero, 2012, 09:40 pm
En este mensaje revisaremos los conceptos básicos relacionados con funciones. Supongo que todo el mundo estará familiarizado con ellos. La cuestión es simplemente comprobar que están estratificados. Empezamos por la definición misma de aplicación entre dos conjuntos:

\( f:A\longrightarrow B\equiv f\subset A\times B\land \forall abb'((a,b)\in f\land (a,b')\in f\rightarrow b=b') \)

\( \land \forall a\in A\exists b\in B\ (a,b)\in f \)

Notemos que aunque la \( f \) sea minúscula, es un conjunto, como se deduce, por ejemplo, de la inclusión \( f\subset A\times B \).

Estratificación de la definición
\( f_1\subset A_1\times B_1\land \forall abb'((a_0,b_0)_0\in f_1\land (a_0,b'_0)\in f_1\rightarrow b_0=b'_0) \)

\( \land \forall a_0\in A_1\exists b_0\in B_1\ (a_0,b_0)_0\in f_1 \)
[cerrar]

Al estratificar \( f: A\longrightarrow B \) vemos que las tres variables \( f, A, B \) tienen que tener asignado el mismo tipo. Más en general, si en vez de variables son términos cualesquiera, la fórmula estará estratificada si y sólo si los tres términos admiten una estratificación conjunta que asigne a los tres el mismo tipo.

Ahora hemos de fijarnos en la expresión \( f(x) \). En principio tiene sentido cuando \( f \) es una función y \( x \) está en su dominio, pero para estratificarla hemos de preguntarnos: ¿cómo se define exactamente? La respuesta es que se trata de una descripción:

\( f(x)\equiv y\mid (x,y)\in f \).

Para estratificarla tenemos en cuenta que el tipo de la descripción debe coincidir con el de la variable \( y \), luego es: \( (y_0\mid (x_0,y_0)_0\in f_1)_0 \). Más en general:

El término \( f(x) \) (cuando tiene sentido) está estratificado y tiene el mismo tipo que \( x \) y una unidad menos que el tipo de \( f \).

A menudo es frecuente definir aplicaciones diciendo: "Sea \( f:A\longrightarrow B \) la aplicación dada por \( f(x)=t(x,x_1,\ldots, x_n) \)", donde \( t \) es un término que depende de \( x \) y tal vez de otros parámetros. Esto equivale a que

\( f\equiv \{(x,y)\mid x\in A\land y\in B\land y=t(x,x_1,\ldots, x_n)\} \)

Y vemos que la definición está estratificada si y sólo si el término \( t(x,x_1,\ldots, x_n) \) admite una estratificación que le asigna el mismo tipo que a la variable \( x \).

El principio general es:

Para que la definición de una aplicación \( f \) esté estratificada su definición debe asignar a cada elemento de su dominio el mismo tipo que a su imagen, y éstos han de ser una unidad inferior al de \( f \).

Se trata de un principio general que podríamos concretar en muchos resultados generales concretos para definir aplicaciones, pero no merece la pena. Por ejemplo, si definimos

\( f: A\times B\longrightarrow B\times A \) mediante \( f(a,b)=(b,a) \), podemos decir que la definición es correcta porque podemos estratificar \( (a_0,b_0)_0 \) y \( (b_0,a_0)_0 \) ambos con tipo \( 0 \) (y entonces \( f \) tiene tipo \( 1 \)). Esta situación no se ajusta exactamente al caso general precedente (porque la función tiene dos variables), pero la comprobación es similar. En la práctica, si alguien quiere comprobar la estratificación de una función con detalle, lo mejor es que la escriba como conjunto, por ejemplo:

\( f\equiv \{x\mid \exists ab(a\in A\land b\in B\land x = ((a,b),(b,a)))\} \)

y es fácil ver que la fórmula que define a \( f \) está estratificada (en esencia por que se cumple el principio general: cada par tiene el mismo tipo que su imagen).

Dejamos al lector la comprobación de que si escribe explícitamente las definiciones de

\( f:A\longrightarrow B \) inyectiva, \( f:A\longrightarrow B \) suprayectiva y \( f:A\longrightarrow B \) biyectiva,

observará que todas ellas están estratificadas y que las tres variables tienen el mismo tipo.

Los conceptos de dominio y rango los usaremos normalmente con funciones, pero no hay razón para complicar la definición exigiendo esto, sino que pueden definirse para conjuntos arbitrarios (y, en realidad, también es útil tener las definiciones aplicables a relaciones cualesquiera):

\( \mathcal DR\equiv \{x\mid \exists y\ (x,y)\in R\},\qquad \mathcal RR\equiv \{y\mid \exists x\ (x,y)\in R\} \).

Igualmente definimos el conjunto (o relación o función) inverso: \( R^{-1}\equiv \{(a,b)\mid (b,a)\in R\} \).

Finalmente:

\( f\circ g\equiv\{(a,c)\mid \exists b\ ((a,b)\in g\land (b,c)\in f)\} \)

\( f[X]\equiv \{y\mid \exists x\in X\ (x,y)\in f\},\qquad f^{-1}[Y]\equiv \{x\mid \exists y\in Y\ (x,y)\in f\} \)

\( f|_X\equiv \{(a,b)\mid a\in X\land (a,b)\in f\} \).

El lector que quiera cerciorarse de que domina "el arte de la estratificación" debería comprobar que todos estos conceptos están estratificados y calcular la relación entre los tipos de sus variables. La tabla siguiente recapitula todos los conceptos que hemos introducido hasta ahora con los datos relevantes respecto a estratificación:

\( \begin{array}{|ccc||ccc|}
\text{Expresión}&\text{Condición}&\text{Variación}&\text{Expresión}&\text{Condición}&\text{Variación}\\
\hline
A\subset B&t(A)=t(B)&&A\times B&t(A)=t(B)&0\\
A\cup B&t(A)=t(B)&0&f:A\longrightarrow B&t(f)=t(A)=t(B)&\\
A\cap B&t(A)=t(B)&0&\mathcal DR&&0\\
\overline A&&0&\mathcal RR&&0\\
A\setminus B&t(A)=t(B)&0&R^{-1}&&0\\
\bigcup A&&-1&f\circ g&t(f)=t(g)&0\\
\bigcap A&&-1&f(x)&t(f)=t(x)+1&-1/0\\
\mathcal PA&&1&f[X]&t(f)=t(X)&0\\
\{x,y\}&t(x)=t(y)&1&f^{-1}[Y]&t(f)=t(Y)&0\\
(x,y)&t(x)=t(y)&0&f|_X&t(f)=t(X)&0\\
\hline
\end{array} \)

Por ejemplo, la segunda línea de la izquierda dice que la condición para que \( A\cup B \) esté estratificado es que el tipo de \( A \) sea igual al tipo de \( B \), y que en tal caso la unión tiene el mismo tipo que \( A \) y \( B \) (la unión no varía el tipo).

En cambio, la sexta línea dice que el tipo de \( \bigcup A \) es una unidad inferior al tipo de \( A \) (la gran unión disminuye el tipo una unidad).

La línea de \( f(x) \) dice que, para que el término esté estratificado el tipo de \( f \) debe ser una unidad mayor que el de \( x \) y que, en tal caso, el término \( f(x) \) tiene tipo una unidad inferior al de \( f \) pero igual al de \( x \) (variación \( -1 \) respecto de \( f \) y \( 0 \) respecto de \( x \)).



Terminamos con una observación sobre qué pasaría si quisiéramos trabajar con aplicaciones definidas con pares no nivelados.

Para definir la suma de cardinales hay que probar que, dados dos conjuntos \( A \) y \( B \), existen siempre dos conjuntos disjuntos del mismo cardinal. La forma típica de asegurar que obtenemos conjuntos disjuntos es pasar a los conjuntos \( A\times\{0\} \) y \( B\times\{1\} \). Dejando de lado la definición de \( 0 \) y \( 1 \), ¿cómo probamos que \( A \) tiene el mismo cardinal que \( A\times\{u\} \), para cualquier objeto \( u \)?. Lo natural es considerar la biyección \( f: A\longrightarrow A\times \{u\} \) dada por \( f(a)=(a,u) \). Esta definición es correcta según lo que hemos visto porque podemos estratificar el par \( (a_0,u_0)_0 \) de modo que tenga el mismo tipo que \( a \) (es decir, de modo que \( a \) y \( f(a)=(a,0) \) tengan el mismo tipo). Así pues, todo va bien.

En cambio, si quisiéramos considerar el par ordenado \( (a,u) \) usual en ZFC, que tiene tipo dos unidades mayor que el tipo de \( a \), resulta que la aplicación \( a\mapsto (a,u) \) no está bien definida (la definición no está estratificada y, por consiguiente, no podemos asegurar que exista el conjunto (la función) \( f \) que pretendemos definir). Eso tampoco significa que no exista, pero el hecho es que tenemos un problema. Esta clase de problemas es la que nos evitamos al trabajar con pares nivelados.
Título: Re: NFA
Publicado por: Óscar Matzerath en 06 Enero, 2012, 04:56 pm
Hola,

Solo quiero felicitarte por el trabajo realizado y animarte a seguir, ya que a mí me interesa bastante el tema y lo voy a seguir con atención. Si ya te animaras a hacer una exposición no solo de la teoría de conjuntos en sí, sino de aspectos de la teoría de modelos de NFU (o NFA), sería genial.

Saludos
Título: Re: NFA
Publicado por: Carlos Ivorra en 06 Enero, 2012, 05:02 pm
Hola,

Solo quiero felicitarte por el trabajo realizado y animarte a seguir, ya que a mí me interesa bastante el tema y lo voy a seguir con atención. Si ya te animaras a hacer una exposición no solo de la teoría de conjuntos en sí, sino de aspectos de la teoría de modelos de NFU (o NFA), sería genial.

Saludos

Gracias, Óscar. Cualquier comentario será bienvenido. En cuanto a lo de los modelos, hay un "robot" pululando por ahí que ha escrito un abstract para mi exposición y ya ha puesto que voy a hablar de la consistencia de NFA, así que ya me había hecho a la idea de hablar de eso. Con mayor razón si hay interés.
Título: Cardinales en NFA
Publicado por: Carlos Ivorra en 06 Enero, 2012, 06:38 pm
En este mensaje expondré los elementos básicos de la teoría de cardinales en NFA como base para construir los números naturales.

La definición básica es la siguiente:

Definición: Diremos que dos conjuntos \( X \) e \( Y \) son equipotentes si existe una aplicación biyectiva \( f:X\longrightarrow Y \). En signos:

\( X\sim Y\equiv \exists f\ f: X\longrightarrow Y \) biyectiva.

Observemos que esta fórmula está estratificada y en ella las variables \( X \) e \( Y \) tienen el mismo tipo.

Las propiedades básicas de la equipotencia son:

1) \( \forall X\ X\sim X \)

2) \( \forall XY(X\sim Y\rightarrow Y\sim X) \)

3) \( \forall XYZ(X\sim Y\land Y\sim Z\rightarrow X\sim Z) \).

Las tres se demuestran sin dificultad. No pongo las pruebas, pero si algún lector tiene dificultades en demostrarlas por sí mismo que me lo diga y las incluiré. En definitiva, estas propiedades afirman que la equipotencia es una relación de equivalencia sobre el conjunto \( C \) de todos los conjuntos.

Definición: Llamaremos cardinal de un conjunto \( X \) al conjunto de todos los conjuntos equipotentes con él, es decir:

\( |X|\equiv \{Y\mid Y\sim X\} \).

Observamos que el término \( |X| \) está estratificado y que su tipo es una unidad superior al de la variable \( X \). Ahora podemos probar la relación fundamental:

Teorema: Dos conjuntos son equipotentes si y sólo si tienen el mismo cardinal.

Demostración
Supongamos que \( X\sim Y \), y hemos de probar que \( |X|=|Y| \). Para ello tomamos un objeto \( u\in |X| \), que será, de hecho, un conjunto \( u\sim X \). Por la transitividad de la equipotencia, \( u\sim Y \), luego \( u\in |Y| \). Igualmente se prueba que si \( u\in |Y| \) entonces \( u\in |X| \), luego \( |X|=|Y| \).

Recíprocamente, si \( |X|=|Y| \), entonces, como \( X\sim X \), tenemos que \( X\in |X| \), luego \( X\in |Y| \), luego \( X\sim Y \).
[cerrar]

Definición: Llamaremos \( K \) al conjunto de todos los cardinales, es decir:

\( K\equiv \{\kappa\mid \exists X\ \kappa = |X|\} \)

Observemos que la fórmula que define a \( K \) está estratificada, luego \( K \) es un conjunto bien definido.

Vamos a definir una relación de orden en \( K \).

Definición: Diremos que un conjunto \( X \) es minuspotente a un conjunto \( Y \) si existe una aplicación \( f:X\longrightarrow Y \) inyectiva. Equivalentemente:

\( X\preceq Y\equiv \exists f\ f:X\longrightarrow Y \) inyectiva.

Así vemos que esta fórmula está estratificada con las dos variables del mismo tipo.

Definición: Diremos que un cardinal \( \kappa \) es menor o igual que un cardinal \( \mu \) si existen conjuntos \( X \) e \( Y \) tales que \( |X|=\kappa, |Y|=\mu \) y \( X\preceq Y \).

Más precisamente, podemos definir \( \leq \) como el siguiente subconjunto de \( K\times K \):

\( \leq \ \equiv \{(\kappa,\mu)\mid \exists XY(|X|=\kappa\land |Y|=\mu\land X\preceq Y)\} \)

La definición es correcta porque la fórmula está estratificada y asigna el mismo tipo a las variables \( \kappa \) y \( \mu \).

De este modo, si escribimos \( \kappa\leq \mu\equiv (\kappa,\mu)\in\ \leq \), tenemos que

\( \kappa\leq \mu\leftrightarrow \exists XY(|X|=\kappa\land |Y|=\mu\land X\preceq Y) \).

Teorema: Para todo par de conjuntos \( X \), \( Y \) se cumple que \( X\preceq Y\leftrightarrow |X|\leq |Y| \).

Demostración
Si \( X\preceq Y \) es inmediato que \( |X|\leq |Y| \), por la propia definición. El recíproco requiere cierto cálculo. En efecto, si suponemos que \( |X|\leq |Y| \), esto significa que existen conjuntos \( X' \) e \( Y' \), no necesariamente los que teníamos, tales que \( X'\preceq Y' \).

Ahora bien, entonces tenemos aplicaciones \( f: X\longrightarrow X' \) biyectiva, \( f': Y\longrightarrow Y' \) biyectiva y \( g': X'\longrightarrow Y' \) inyectiva. Consideramos entonces \( g=f'^{-1}\circ g'\circ f: X\longrightarrow Y \), y es fácil ver que es inyectiva. Por lo tanto \( X\preceq Y \).
[cerrar]

En particular tenemos que si \( X\subset Y \) entonces \( |X|\leq |Y| \), pues podemos considerar la aplicación inclusión \( X\longrightarrow Y \) dada por \( x\mapsto x \).

La relación que acabamos de definir es una relación de orden, es decir, cumple las tres propiedades siguientes:

Teorema:

1) \( \forall \kappa\in K\ \kappa\leq \kappa \)

2) \( \forall \kappa\mu\in K\ (\kappa\leq \mu\land \mu\leq \kappa\rightarrow \kappa=\mu \)

3) \( \forall\kappa\mu\nu\in K(\kappa\leq \mu\land \mu\leq \nu\rightarrow \kappa\leq \nu) \).

Las propiedades 1) y 3) son muy sencillas y no pondré la demostración. (Pero insisto en que si alguien quiere que demuestre cualquier cosa que me salto sólo tiene que pedirlo. Me salto cosas por aligerar un poco, pero lo que haga falta probar, se prueba. Incluso si alguien quiere completar el hilo añadiendo pruebas de cosas no demostradas, yo las revisaré con mucho gusto.)

La propiedad 2) es el teorema de Cantor-Bernstein, y se prueba exactamente igual que en ZFC, pero incluiré la prueba por si alguien tiene curiosidad y no conoce la demostración. Necesita un teorema previo:

Teorema de punto fijo: Sea \( X \) un conjunto y \( F:\mathcal PX\longrightarrow \mathcal PX \) una aplicación con la propiedad de que si \( A\subset B\subset X \) entonces \( F(A)\subset F(B) \).  Entonces existe un \( Z\in \mathcal PX \) tal que \( F(Z)=Z \).

Demostración
Sea \( S=\{A\mid A\in \mathcal PX\land F(A)\subset A\} \). Como siempre, es crucial comprobar que las definiciones son correctas, es decir, que las propiedades están estratificadas, pero en este caso está claro, pues \( F(A) \) tiene el mismo tipo que \( A \), como requiere la incusión.

Ahora observamos que \( S\neq \emptyset \), porque claramente \( X\in S \). Definimos \( Z=\bigcap S \), que está contenido en todos los elementos de \( S \), en particular en \( X \), luego \( Z\in \mathcal PX \). (Si \( S \) fuera vacío, sería \( Z=V \).) Vamos a probar que \( Z \) cumple lo pedido.

Si \( A\in S \), entonces, como \( Z \) está contenido en todos los elementos de \( S \), tenemos que \( Z\subset A \), luego, por la hipótesis sobre \( F \) y por definición de \( S \), tenemos que \( F(Z)\subset F(A)\subset A \).

Con esto hemos probado que \( F(Z) \) está contenido en todos los elementos de \( S \), luego está contenido en su intersección, es decir, \( F(Z)\subset Z \).

De nuevo por la hipótesis sobre \( F \) tenemos que \( F(F(Z))\subset F(Z) \), y por definición de \( S \) tenemos que \( F(Z)\in S \), luego \( Z\subset F(Z) \), ya que \( Z \) (la intersección de \( S \)) está contenida en todos los elementos de \( S \).

La doble inclusión prueba que \( F(Z)=Z \).
[cerrar]

Teorema de Cantor Bernstein Dados dos conjuntos \( X \) e \( Y \), si \( X\preceq Y \) e \( Y\preceq X \), entonces \( X\sim Y \).

Demostración
La hipótesis es que tenemos aplicaciones \( f:X\longrightarrow Y,\ g: Y\longrightarrow X \) inyectivas. Definimos
\( F:\mathcal PX\longrightarrow \mathcal PX \) mediante \( F(A)=X\setminus g[Y\setminus f[A]] \).

Lo único que hemos de hacer de más por trabajar en NFA en lugar de en ZFC es comprobar que la definición de \( F \) es correcta, lo que en este caso significa que el término \( X\setminus g[Y\setminus f[A]] \) está estratificado con el mismo tipo que \( A \), lo cual es cierto, pues hemos visto que todos los conceptos involucrados conservan el tipo.

Ahora comprobamos que \( F \) cumple las hipótesis del teorema anterior: Si \( A\subset B\subset X \), entonces \( f[A]\subset f[B]\rightarrow  Y\setminus f[B]\subset Y\setminus f[A]\rightarrow g[Y\setminus f[B]]\subset g[Y\setminus f[A]] \)

\( \rightarrow X\setminus g[Y\setminus f[A]]\subset X\setminus g[Y\setminus f[B]]\rightarrow F(A)\subset F(B) \).

Así pues, podemos aplicar el teorema anterior para concluir que existe un \( Z\in \mathcal PX \) tal que \( Z=X\setminus g[Y\setminus f[Z]] \) o, equivalentemente, \( X\setminus Z=g[Y\setminus f[Z]] \).

Así tenemos que \( f|_Z: Z\longrightarrow f[Z] \) biyectiva y \( g|_{Y\setminus f[Z]}:Y\setminus f[Z]\longrightarrow X\setminus Z \) biyectiva. Si llamamos \( h=(g|_{Y\setminus f[Z]})^{-1}: X\setminus Z\longrightarrow Y\setminus f[Z] \) biyectiva, concluimos que

\( f|_Z\cup h: X\longrightarrow Y \) biyectiva.

(Notemos que \( f|_Z\cup h \) es la aplicación que actúa como \( f \) sobre \( Z \) y como \( h \) sobre \( X\setminus Z \)).

Esto prueba que \( X\sim Y \).
[cerrar]

La propiedad antisimétrica de la relación de orden entre cardinales (propiedad 2) es simplemente el teorema de Cantor-Bernstein en términos de cardinales en lugar de en términos de conjuntos.

En estos momentos no podemos demostrar que \( \forall \kappa\mu\in K(\kappa\leq \mu\lor \mu\leq \kappa) \) porque esta propiedad requiere el axioma de elección, y todavía no lo hemos incorporado a la teoría. Por otra parte, tampoco necesitamos esto para nada, de momento.

Por ahora sólo podemos añadir lo siguiente a las propiedades de la relación de orden:

Teorema: Existe un mínimo cardinal \( 0 = |\emptyset| \) y un máximo cardinal \( \kappa_0=|V| \).

Demostración
Que \( 0=|\emptyset| \) sea el mínimo cardinal significa que si \( \kappa \) es cualquier cardinal entonces \( 0\leq \kappa \), lo que a su vez significa que si \( |X|=\kappa \) entonces existe una aplicación inyectiva \( f: \emptyset\longrightarrow X \).

Sucede que \( f=\emptyset \) cumple lo pedido, y probarlo es el típico ejercicio de "cinismo" que podemos llamar "lógica del conjunto vacío": \( \emptyset \) es una aplicación de \( \emptyset \) en \( X \), porque no hay ningún elemento de \( \emptyset \) que no sea un par ordenado con primera componente en \( \emptyset \) y segunda componente en \( X \) (¿cómo lo iba a haber, si en \( \emtyset \) no hay nada?) y no hay ningún elemento en \( \emptyset \) que no tenga imagen en \( X \) (¿cómo lo iba a haber?).

Además \( \emptyset \) es una aplicación inyectiva, pues no hay dos elementos en \( \emptyset \) con la misma imagen en \( X \) ¿cómo los iba a haber?. Por dudoso que esto le pueda parecer a alguien: esto es lógica.  ;D

Observemos que en particular \( \emptyset:\emptyset \longrightarrow \emptyset \) biyectiva, pues no hay ningún elemento en \( \emptyset \) que carezca de antiimagen (¿cómo podría haberlo?). Más aún,  si \( f:\emptyset \longrightarrow X \) biyectiva, necesariamente \( X=\emptyset \). En efecto, si hubiera un elemento \( u\in X \), debería tener una antiimagen en \( \emptyset \), lo cual es imposible, luego \( X=\emptyset \).

Esto significa que \( 0=\{\emptyset\} \) (el conjunto de todos los conjuntos equipotentes a \( \emptyset \) se reduce al conjunto vacío).

Por otro lado, dado un cardinal \( \kappa \) cualquiera, tomamos un conjunto tal que \( |X|=\kappa \). Puesto que \( X\subset V \), la inclusión \( i: X\longrightarrow V \) dada por \( x\mapsto x \) es claramente inyectiva, luego \( \kappa = |X|\leq |V|=\kappa_0 \).

Esto prueba que \( \kappa_0 \) es el mayor de todos los cardinales.
[cerrar]

Para definir la suma de cardinales observamos primero lo siguiente:

Teorema: Dados dos cardinales \( \kappa \) y \( \mu \) existen conjuntos tales que \( |X|=\kappa, |Y|=\mu \) y \( X\cap Y=\emptyset \).

Demostración
Tomemos conjuntos tales que \( |X'|=\kappa \) y \( |Y'|=\mu \). Existen por la propia definición de cardinal, pero no podemos asegurar que sean disjuntos. Para obtener conjuntos disjuntos tomamos dos objetos cualesquiera tales que \( u\neq v \). Por ejemplo, \( u=\emptyset \), \( v=\{\emptyset\} \) y definimos

\( X=X'\times\{u\} \), \( Y=Y'\times \{v\} \).

Así es inmediato que \( X\cap Y=\emptyset \), pues un elemento de la intersección debería ser un par ordenado con segunda componente igual a \( u \) y a \( v \) a la vez. Sólo hemos de probar que \( |X|=|X'| \) y que \( |Y|=|Y'| \). Lo probamos para \( X \), para lo cual basta construir la aplicación \( f: X\longrightarrow X\times \{u\} \) mediante \( f(x)=(x,u) \).

Es crucial comprobar que la definición es correcta porque la imagen \( (x,u) \) tiene el mismo tipo que \( x \) (y aquí es crucial que contamos con un par ordenado nivelado). No cuesta nada probar que \( f \) es biyectiva, luego en efecto, \( |X|=|X'| \), e igualmente se razona con \( Y \).
[cerrar]

Definición: Llamaremos suma de cardinales a la aplicación \( +:K\times K\longrightarrow K \) definida como sigue: dados dos cardinales \( \kappa \) y \( \mu \), tomamos conjuntos disjuntos tales que \( |X|=\kappa \) y \( |Y|=\mu \) y llamamos \( \kappa+\mu=|X\cup Y| \).

Más técnicamente:

\( +\equiv \{((\kappa,\mu),\nu))\mid \exists XY(\kappa=|X|\land \mu=|Y|\land X\cap Y=\emptyset\land \nu=|X\cup Y|)\} \).

Es evidente que existe el conjunto \( + \) (la definición está estratificada), pero no es evidente que se trata realmente de una aplicación \( +: K\times K\longrightarrow K \). Ciertamente, es un conjunto de pares ordenados con primera componente en \( K\times K \) y segunda componente en \( K \), pero hemos de probar que cada par de cardinales \( (\kappa, \mu) \) tiene una única imagen \( \nu \).

Demostración
Si \( ((\kappa, \mu),\nu)\in + \) y también \( ((\kappa, \mu), \nu')\in + \), esto significa que, por una parte, tenemos conjuntos  tales que \( |X|=\kappa, |Y|=\mu, X\cap Y=\emptyset, |X\cup Y|=\nu \) y por otra parte tenemos conjuntos tales que \( |X'|=\kappa, |Y'|=\mu, X'\cap Y'=\emptyset, |X'\cup Y'|=\nu' \).

Hemos de probar que \( \nu=\nu' \). Tenemos biyecciones \( f: X\longrightarrow X' \) y \( g: Y\longrightarrow Y' \), y el hecho de que los conjuntos sean disjuntos implica que \( (f\cup g): X\cup Y\longrightarrow (X'\cup Y') \) biyectiva. Por lo tanto \( \mu = |X\cup Y|=|X'\cup Y'|=\nu \).
[cerrar]

En definitiva, tenemos el teorema siguiente:

Teorema: Si \( X \) e \( Y \) son conjuntos disjuntos, entonces \( |X\cup Y|=|X|+|Y| \).

Ahora es fácil probar que la suma de cardinales es asociativa, conmutativa y que tiene como elemento neutro al \( 0 \). Dejamos todos estos hechos como ejercicios.

No estamos en condiciones de calcular sumas explícitamente, pero hay dos muy sencillas: \( 0+0=0 \) y \( \kappa_0+\kappa_0=\kappa_0 \).

El producto de cardinales se define sin dificultad:

Definición: Definimos el producto de cardinales como la aplicación \( \cdot: K\times K\longrightarrow K \) que se calcula como sigue: tomamos conjuntos tales que \( |X|=\kappa \) y \( |Y|=\mu \) y entonces \( \kappa\cdot \mu=|X\times Y| \).

Explícitamente:

\( \cdot \equiv \{((\kappa,\mu),\nu)\mid \exists XY(\kappa=|X|\land \mu=|Y|\land \nu=|X\times Y|)\} \)

Como en el caso de la suma, es inmediato que la definición está estratificada, pero hay que probar que este producto es realmente una aplicación de \( K\times K \) en \( K \). El argumento sigue exactamente el mismo esquema que para la suma, así que lo omitimos.

De este modo tenemos:

Teorema: Para todo par de conjuntos, se cumple que \( |X\times Y| = |X|\cdot |Y| \).

Dejamos también como ejercicio las propiedades obvias: el producto de cardinales es asociativo y conmutativo, además \( \forall \kappa\in K\ \kappa\cdot 0=0 \) y el producto tiene como elemento neutro al cardinal

\( 1= \{X\mid \exists u\ X= \{u\}\} \).

Ejercicio: Probar que \( 1 \) es el menor cardinal mayor que \( 0 \).

Solución
Sea \( \kappa>0 \). Hemos de probar que \( 1\leq \kappa \). Sea \( |X|=\kappa \). No puede ser \( X=\emptyset \), pues entonces \( \kappa =|X|=0 \). Por lo tanto, existe un objeto \( u\in X \), luego \( \{u\}\subset X \), luego \( 1=|\{u\}|\leq |X|=\kappa \).
[cerrar]

En el próximo mensaje construiremos los números naturales.
Título: Los números naturales en NFA
Publicado por: Carlos Ivorra en 07 Enero, 2012, 01:03 pm
Con lo visto hasta ahora es fácil construir los números naturales en NFA:

Tenemos definido el conjunto \( K \) de todos los cardinales y, en él, la suma y el producto de cardinales, así como los cardinales \( 0=|\emptyset| \) y \( 1=\{X\mid \exists u\ X=\{u\}\} \).

Así podemos definir el conjunto de los conjuntos inductivos como:

\( \text{Ind}\equiv \{A\mid A\subset K\land 0\in A\land \forall n\in A\ n+1\in A\} \)

Es claro que la definición está estratificada, pero, no obstante, haremos algunos comentarios al respecto:

Spoiler
Una estratificación es:

\( A_1\subset K_1\land 0_0\in A_1\land \forall n_0\in A_1\ (n_0+_11_0)_0\in A_1 \)

Aquí hemos tenido en cuenta varias cosas:

1) Los términos que no tienen variables libres, como \( K, 0, 1,+ \), si están estratificados, admiten estratificaciones en las que el tipo asignado sea cualquiera que nos convenga, porque siempre es posible sumar un mismo número entero a todos los tipos y la estratificación se conserva.

2) Notemos que \( + \) es una aplicación (en particular es un conjunto), de modo que \( n+1\equiv +(n,1)\equiv (n,1)\in + \) y sabemos que una expresión de este tipo está estratificada de modo que la imagen \( n+1 \) tiene el mismo tipo que el argumento \( (n,1) \), que a su vez tiene el mismo tipo que las dos componentes.
[cerrar]

Definición: Llamaremos conjunto de los números naturales a \( \mathbb{N}=\bigcap\text{Ind} \), es decir, a la intersección de todos los conjuntos inductivos.

Observamos que trivialmente \( K\in \text{Ind} \) y, como \( \mathbb{N} \) está contenido en todos los elementos de \( \text{Ind} \), concluimos que \( \mathbb{N}\subset K \), es decir, que los números naturales son cardinales.

Teorema (Axiomas de Peano) Se cumple:

1) \( 0\in \mathbb{N} \).

2) \( \forall n\in \mathbb{N}\ n+1\in \mathbb{N} \).

3) \( \forall mn\in \mathbb{N}(m+1=n+1\rightarrow m=n). \)

4) \( \forall n\in \mathbb{N}\ n+1\neq 0 \).

5) \( \forall X(X\subset \mathbb{N}\land 0\in X\land \forall n\in X\ n+1\in X\rightarrow X=\mathbb{N}) \).

Demostración
1) Como \( 0 \) pertenece a todo conjunto inductivo, también pertenece a la intersección de todos los conjuntos inductivos, es decir, a \( \mathbb{N} \).

2) Si \( n\in \mathbb{N} \) y \( A\in \text{Ind} \), entonces \( n\in A \) por definición de \( \mathbb{R} \), luego \( n+1\in A \) por definición de \( \text{Ind} \), luego \( n+1\in \mathbb{N} \) por definición de \( \mathbb{N} \).

3) Por definición de suma podemos tomar conjuntos \( A, B, C, D \) tales que \( |A|=m, |B|=n, |C|=|D|=1 \) y \( A\cap C=B\cap D=\emptyset \). Por definición de \( 1 \) tenemos que \( C=\{u\} \), \( D=\{v\} \), para ciertos objetos \( u \) y \( v \).

Entonces \( m+1 = |A\cup\{u\} \) y \( n+1=|B\cup\{v\} \) y por hipótesis existe una biyección \( f:A\cup \{u\}\longrightarrow B\cup \{v\} \). Si \( f(u)=v \), entonces \( f \) se restringe a una biyección \( A\longrightarrow B \) que prueba que \( m=|A|=|B|=n \).

En caso contrario sea \( f(u)=u'\in B \) y sea \( f^{-1}(v)=v'\in A \). Es claro que \( g=(f\setminus\{(u,u'),(v',v)\})\cup \{(v',u')\} \) es una biyección \( g:A\longrightarrow B \) y concluimos igualmente.

4) Como antes, podemos tomar \( |A|=n \) y un objeto \( u \) de modo que \( n+1=|A\cup\{u\}| \). Como \( A\cup\{u\}\neq\emptyset \), tenemos que \( n+1\neq 0 \).

5) La hipótesis es que \( X\subset \mathbb{N} \) y \( X\in \text{Ind} \), pero lo segundo implica que \( \mathbb{N}\subset X \) por definición de \( \mathbb{N} \).
[cerrar]

El quinto axioma de Peano es el principio de inducción, pero hay que ser cauto a la hora de aplicarlo, pues demostrar por inducción que todos los números naturales cumplen una propiedad \( \phi(n) \) consiste en aplicar el quinto axioma de Peano al conjunto \( X=\{n\mid n\in \mathbb{N}\land \phi(n)\} \), y para que esto sea correcto la fórmula \( \phi \) tiene que estar estratificada.

Como ejemplo de prueba por inducción veamos que la función siguiente \( S:\mathbb{N}\longrightarrow \mathbb{N}\setminus\{0\} \) es biyectiva. Sabemos que es inyectiva por el tercer axioma de Peano. Para probar la suprayectividad (es decir, que todo natural no nulo es el siguiente de otro número natural) consideramos el conjunto (bien definido)

\( X=\{n\mid n\in \mathbb{N}\land (n=0\lor \exists m\in \mathbb{N}\ n=m+1\} \)

y se prueba trivialmente que cumple las condiciones del principio de inducción, luego \( X=\mathbb{N} \) y esto implica lo dicho.

Veamos algunas propiedades más:

Teorema: Si \( n\in \mathbb{N} \), entonces \( n+1 \) es el menor cardinal mayor que \( \mathbb{N} \).

Demostración
Obviamente \( n\leq n+1 \). Que la desigualdad es estricta se prueba por inducción sobre el conjunto

\( X=\{n\mid n\in \mathbb{N}\land n<n+1\} \)

Tenemos que \( 0\in X \) por el cuarto axioma de Peano y, si \( n\in X \), entonces \( n<n+1 \) y no puede ser \( n+1=n+1+1 \) por el tercer axioma de Peano, luego \( n+1\in X \).

Esto prueba que \( n+1 \) es un cardinal mayor que \( n \). Para probar que es el mínimo tomamos un cardinal \( \kappa \) tal que \( n<\kappa \) y probamos que \( n+1\leq \kappa \).

Por definición de suma existe un conjunto \( |A|=n \) y un objeto \( u\notin A \) de modo que \( n+1=|A\cup\{u\}| \). Sea \( \kappa=|B| \). Por hipótesis existe una aplicación \( f:A\longrightarrow B \) inyectiva que no puede ser biyectiva, pues en tal caso sería \( n=\kappa \). Sea \( u'\in B\setminus f[A] \). Entonces \( g=f\cup\{(u,u')\} \) cumple que \( f':A\cup\{u\}\longrightarrow B \) inyectiva, luego \( n+1\leq \kappa \).
[cerrar]

Más precisamente, los números naturales son los primeros cardinales en el sentido siguiente:

Teorema: Si \( \kappa\in K \) y \( n\in \mathbb{N} \), entonces \( n\leq \kappa \) o \( \kappa\leq n \), y en el segundo caso \( \kappa\in \mathbb{N} \).

Demostración
Por inducción sobre \( n \) (dejo como ejercicio comprobar que el conjunto correspondiente está bien definido).

Si \( n=0 \) sabemos que \( 0\leq \kappa \).

Supongamos que el teorema vale para \( n \), es decir, que \( n\leq \kappa \) o bien \( \kappa\leq n\land \kappa\in \mathbb{N} \).

En el segundo caso, obviamente \( \kappa\leq n+1\land \kappa\in \mathbb{N} \).

En el primer caso, o bien \( n=\kappa \), en cuyo caso \( \kappa\leq n+1\land \kappa\in \mathbb{N} \), o bien \( n<\kappa \), en cuyo caso \( n+1\leq \kappa \) por el teorema anterior.
[cerrar]

Por último:

Teorema: La suma y el producto de números naturales es un número natural.

Demostración
Fijamos un número natural \( m \) y consideramos el conjunto (bien definido)

\( X=\{n\mid n\in \mathbb{N}\land m+n\in \mathbb{N}\} \).

Como \( m+0=m \), tenemos que \( 0\in X \).

Si \( n\in X \), entonces \( m+n\in \mathbb{N} \), luego \( m+(n+1)=(m+n)+1\in \mathbb{N} \) por el segundo axioma de Peano, luego \( n+1\in X \).

(Aquí hemos empleado que la suma de cardinales es asociativa, cosa que ya establecimos en el mensaje anterior.)

Para el producto se razona análogamente, usando esta vez que \( m(n+1)=mn+n \), de modo que \( mn\in \mathbb{N} \) por hipótesis de inducción y la suma está en \( \mathbb{N} \) porque ya hemos probado que la suma de números naturales es un número natural.
[cerrar]

Al tener definida la suma y el producto de los números naturales con sus propiedades usuales (que hemos demostrado, de hecho, para cardinales arbitrarios) tenemos probado que NFA es una teoría aritmética (obviamente recursiva), de modo que, por una parte, le son aplicables los teoremas de incompletitud de Gödel y, por otra parte, ahora sabemos que en NFA se pueden definir todos los conceptos que en general se pueden definir en cualquier teoría aritmética, como la exponenciación de cardinales, los conceptos de número par, impar, primo, exponente de un primo en un número, etc.

Podemos definir el conjunto de todos los conjuntos finitos como

\( \mathcal F=\bigcup \mathbb{N} \).

Dejamos al lector que desarrolle las propiedades básicas de los conjuntos finitos en NFA.
Título: Ordinales en NFA
Publicado por: Carlos Ivorra en 08 Enero, 2012, 07:03 pm
Describiremos aquí los elementos básicos de la teoría de ordinales. Empezamos recordando la definición de conjunto bien ordenado, principalmente para constatar que está estratificada.

Definición: \( (A,\leq) \) es un conjunto bien ordenado si \( \leq\ \subset A\times A \) y además

1) \( \forall x\in A\ x\leq x \),

2) \( \forall xy\in A\ (x\leq y\land y\leq x\rightarrow x=y \),

3) \( \forall xyz\in A\ (x\leq y\land y\leq z\rightarrow x\leq z \),

4) \( \forall X (X\subset A\land X\neq \emptyset\rightarrow \exists x\in X\forall y\in X\ x\leq y) \).

Es facil ver que todas estas condiciones están estratificadas. La última dice que todo subconjunto de \( A \) no vacío tiene un mínimo elemento. Este mínimo es único, pues si \( x,x'\in X \) cumplen la condición, tendríamos que \( x\leq x' \) y \( x'\leq x \), luego \( x=x' \) por 2).

Esto nos permite definir

\( \min X\equiv x\mid (x\in X\land \forall y\in X\ x\leq y) \),

y así observamos que el término \( \min X \) está estratificado y que su tipo es una unidad menor que el tipo de \( X \).

Observemos que todo conjunto bien ordenado está totalmente ordenado, es decir, \( \forall xy\in A(x\leq y\lor y\leq x) \). Basta tener en cuenta que \( \{x,y\} \) tiene mínimo.

En general, si \( (A,\leq) \) es un conjunto ordenado (no necesariamente bien ordenado) y \( a\in A \), podemos considerar el segmento inicial determinado por \( a \):

\( A_a^<\equiv \{x\mid x\in A\land x<a\} \).

El término está estratificado y su tipo es el mismo de \( A \) y una unidad mayor que el de \( a \).

A menudo abusaremos de la notación escribiendo \( (A_a^<,\leq) \), cuando en lugar de \( \leq \) habría que escribir \( \leq\ \cap\ (A_a^<\times A_a^<) \).

Para completar la construcción de los números naturales del mensaje anterior faltaría probar lo siguiente:

Teorema: \( (\mathbb{N},\leq) \) es un conjunto bien ordenado.

Demostración
Sea \( X\subset \mathbb{N} \) un conjunto no vacío, y consideramos el conjunto

\( A=\{n\mid n\in \mathbb{N}\land \mathbb{N}_n^<\cap X=\emptyset\lor \exists m\in \mathbb{N}(m\leq n\land m \text{ es m\'{\i}nimo de }X)\} \).

Vemos que la definición de \( A \) es correcta (la propiedad que lo define está estratificada) y razonamos por inducción que \( A=\mathbb{N} \). En efecto, \( 0\in A \) porque \( \mathbb{N}_0^<=\emptyset \) y si \( n\in A \), o bien hay un mínimo de \( X \) menor o igual que \( n \), en cuyo caso lo mismo vale para \( n+1 \), luego \( n+1\in A \), o bien \( \mathbb{N}_n^<\cap X=\emptyset \). En este caso, o bien \( n\notin X \), con lo que \( \mathbb{N}_{n+1}^<\cap X=\emptyset \) y \( n+1\in A \), o bien \( n\in X \), en cuyo caso \( n \) es el mínimo de \( X \) y \( n\leq n+1 \), luego \( n+1\in A \).

Como sabemos que \( X \) no es vacío, tomamos \( n\in X \). Como \( n+1\in A \) y no cumple la primera condición, debe cumplir la segunda, es decir, hay un mínimo de \( X \) por debajo de \( n+1 \).
[cerrar]

Definición: Diremos que \( f:(A,\leq_A)\longrightarrow (B,\leq_B) \) es una semejanza si \( (A,\leq_A) \) y \( (B,\leq_B) \) son conjuntos bien ordenados, \( f: A\longrightarrow B \) biyectiva y \( \forall xy\in A(x\leq_A y\leftrightarrow f(x)\leq_B f(y)) \). Dos conjuntos bien ordenados son semejantes si existe una semejanza entre ellos.

Observemos que la definición de semejanza está estratificada con todas las variables del mismo tipo. Naturalmente se puede definir el concepto de semejanza para conjuntos no necesariamente bien ordenados, pero si no están totalmente ordenados hay que pedir que la inversa conserve también el orden.

Es fácil probar que la identidad de un conjunto bien ordenado en sí mismo es una semejanza, que la inversa de una semejanza es una semejanza y que la composición de semejanzas es una semejanza. En general, las semejanzas conservan todos los conceptos definidos en términos de la relación de orden (por ejemplo, si \( m \) es el mínimo de \( X \) entonces \( f(m) \) es el mínimo de \( f[X] \), o también, \( f[A_a^<]=B_{f(a)}^< \), etc.)

El siguiente hecho elemental tiene consecuencias importantes:

Teorema: Sea \( (A,\leq) \) un conjunto bien ordenado y \( f: A\longrightarrow A \) inyectiva tal que \( \forall xy\in A(x\leq y\rightarrow f(x)\leq f(y)) \). Entonces \( \forall x\in A\ x\leq f(x) \).

Demostración
Consideramos el conjunto \( X=\{x\in A\mid f(x)<x\} \). Basta probar que es vacío. Si no lo fuera, tendría un mínimo \( x \). Tenemos que \( f(x)<x \), luego \( f(f(x))<f(x) \), luego \( f(x)\in X \) y es menor que su mínimo, contradicción.
[cerrar]

De aquí deducimos:

Teorema: Un conjunto bien ordenado no es semejante a ninguno de sus segmentos iniciales.

Demostración
Si existe \( f:(A,\leq)\longrightarrow (A_a^<,\leq) \) semejanza, entonces \( f \) está en las condiciones del teorema anterior, pero \( f(a)\in A_a^< \), es decir, \( f(a)<a \), contradicción.
[cerrar]

El hecho básico no trivial sobre conjuntos bien ordenados es el siguiente:

Teorema: Dados dos conjuntos bien ordenados, o bien son semejantes, o bien uno es semejante a un (único) segmento inicial del otro.

Demostración
Sean \( (A,\leq_A) \) y \( (B,\leq_B) \) dos conjuntos bien ordenados.

En esta prueba vamos a considerar segmentos iniciales no estrictos, es decir, para cada \( a\in A \), definimos \( A_a^\leq\equiv \{x\mid x\in A\land x\leq a\} \), e igualmente para \( B \)

Observamos que, dado \( a\in A \), si existe un \( b\in B \) tal que \( (A_a^\leq,\leq_A) \) es semejante a \( (B_b^\leq,\leq_B) \), entonces \( b \) es único, pues si hubiera dos, digamos \( b<b' \) y \( b<c\leq b' \) es el menor elemento de \( B \) mayor que \( b \), entonces \( B_b^\leq \) y \( B_{b'}^\leq \) serían semejantes, pues ambos son semejantes a \( A_a^\leq \), pero \( B_b^\leq = (B_{b'}^\leq)_c^< \), y tenemos un conjunto bien ordenado semejante a uno de sus segmentos iniciales (estrictos), lo cual es imposible.

Similarmente, para cada \( b\in B \) hay a lo sumo un \( a\in A \) tal que \( (A_a^\leq,\leq) \) es semejante a \( (B_b^\leq,\leq) \).

Esto se traduce en que el conjunto

\( f\equiv\{(a,b)\mid a\in A\land b\in B\land \exists g\ g:(A_a^\leq,\leq_A)\longrightarrow (B_b^\leq,\leq_B)\mbox{ semejanza}\} \)

es una biyección de un subconjunto de \( A \) en un subconjunto de \( B \). (Recordamos que siempre que se define un conjunto se sobreentiende que hay que comprobar que la fórmula está estratificada.)

Veamos que \( f:(\mathcal Df,\leq_A)\longrightarrow (\mathcal Rf,\leq_B) \) es una semejanza.

En efecto, ya sabemos que \( f \) es biyectiva y, si \( a\leq a' \) están en \( \mathcal Df \), entonces existe una semejanza \( g:(A_{a'}^\leq,\leq_A)\longrightarrow (B_{f(a')}^{\leq},\leq_B) \). Sea \( b=g(a)\leq f(a') \), de modo que \( g \) se restringe a una semejanza

\( (A_a^\leq,\leq_A)\longrightarrow (B_b^\leq,\leq_B) \),

luego el par \( (a,b) \) cumple la definición de \( f \), es decir, \( f(a)=b\leq f(a') \).


Vamos a probar que o bien \( \mathcal Df=A \), o bien existe un \( a\in A \) tal que \( \mathcal Df=A_a^< \).

Si no se da el primer caso, sea \( a = \min(A\setminus \mathcal Df) \) y vamos a probar que \( \mathcal Df = A_a^< \). Trivialmente \( A_a^<\subset \mathcal Df \).

Tomamos \( a'\in \mathcal Df \) y hemos de ver que \( a'<a \). En caso contrario \( a<a' \), ya que la igualdad está descartada, pues \( a\notin \mathcal Df \). Sea \( b'=f(a') \). Esto significa que existe una semejanza \( g: (A_{a'}^{\leq},\leq)\longrightarrow (B_{b'}^{\leq},\leq) \). Sea \( b=g(a) \). Entonces, es claro que \( g|_{A_a^\leq}: (A_{a}^{\leq},\leq)\longrightarrow (B_{b}^{\leq},\leq) \), luego \( a\in \mathcal Df \), contradicción.

Similarmente se prueba que, o bien \( \mathcal Rf=B \) o bien existe un \( b\in B \) tal que \( \mathcal Rf=B_b^< \).

Esto nos da cuatro posibilidades, de las cuales las tres primeras son las que hemos de probar y sólo nos falta descartar la cuarta:

1) \( \mathcal Df=A\land \mathcal Rf=B \), con lo que los conjuntos dados son semejantes.

2) \( \mathcal Df=A\land \mathcal Rf=B_b^< \), con lo que \( A \) es semejante a un segmento de \( B \).

3) \( \mathcal Df=A_a^<\land \mathcal Rf=B \), con lo que \( B \) es semejante a un segmento de \( A \).

4) \( f: (A_a^<,\leq)\longrightarrow (B_b^<,\leq) \) semejanza.

Este caso no puede darse, pues esto significa que el par \( (a,b) \) cumple la definición de \( f \), luego \( a\in \mathcal Df \), contradicción.
[cerrar]



Con esto tenemos suficiente para definir los ordinales:

Definición: Si \( (A,\leq) \) es un conjunto bien ordenado, definimos su ordinal como el conjunto de todos los conjuntos bien ordenados semejantes a \( (A,\leq) \), es decir,

\( \mbox{ord}(A,\leq)\equiv\{x\mid \exists B\leq_B f\ f: (A,\leq)\longrightarrow (B,\leq_B)\text{ semejanza}\} \).

Observamos que el término \( \mbox{ord}(A,\leq) \) está estratificado y tiene tipo una unidad mayor que el de \( A \) y el de \( \leq \).

Llamaremos \( \text{Ord} \) al conjunto de todos los ordinales, es decir:

\( \text{Ord}\equiv \{\alpha\mid \exists A\leq((A,\leq)\text{ es un c.b.o}\land \alpha=\text{ord}(A,\leq))\} \)

Razonando exactamente igual que con los cardinales (cambiando biyección por semejanza) se prueba que:

Teorema: Si \( (A,\leq_A) \) y \( (B,\leq_B) \) son dos conjuntos bien ordenados, entonces

\( \text{ord}(A,\leq_A)=\text{ord}(B,\leq_B)\leftrightarrow \exists f\ f: (A,\leq_A)\longrightarrow (B,\leq_B)\text{ semejanza} \).

Como en el caso de los cardinales, podemos definir una relación de orden en el conjunto de los ordinales:

\( \leq\ \equiv\ \{(\alpha,\beta)\mid \exists A\leq_AB\leq_B\ (\alpha=\text{ord}(A,\leq_A)\land \beta=\text{ord}(B,\leq_B)\land
 \)

\(
(\alpha=\beta\lor \exists bg(b\in B\land g: (A,\leq_A)\longrightarrow (B_b^<,\leq_B)\text{ semejanza})))\} \)

La demostración del teorema siguiente es también análoga a la correspondiente para cardinales (y en ella está implícito que  la definición de la relación de orden no depende de la elección del conjunto ordenado que representa a cada ordinal):

Teorema: Si \( (A,\leq_A) \) y \( (B,\leq_B) \) son conjuntos bien ordenados, entonces

\( \text{ord}(A,\leq_A)\leq \text{ord}(B,\leq_B)\leftrightarrow \exists f\ f:(A,\leq_A)\longrightarrow (B,\leq_B)\text{ semejanza}\lor
 \)

\(
\exists fb\ (b\in B\land f:(A,\leq_A)\longrightarrow (B_b^<,\leq_B)\text{ semejanza}) \).

En definitiva: un conjunto bien ordenado tiene ordinal menor o igual que otro si ambos son semejantes (y entonces los ordinales son iguales) o bien el primero es semejante a un segmento inicial del segundo.

Observemos que si un conjunto bien ordenado es semejante a un segmento inicial de otro, entonces no pueden ser semejantes entre sí, pues entonces el segundo sería semejante a un segmento inicial propio, y ya hemos visto que eso es imposible. Por lo tanto, podemos precisar:

Teorema: Si \( (A,\leq_A) \) y \( (B,\leq_B) \) son conjuntos bien ordenados, entonces

\( \text{ord}(A,\leq_A)< \text{ord}(B,\leq_B)\leftrightarrow \exists fb\ (b\in B\land f:(A,\leq_A)\longrightarrow (B_b^<,\leq_B)\text{ semejanza}) \).

Más informalmente: el ordinal de \( A \) es menor que el ordinal de \( B \) si para ordenar un conjunto como \( B \) hay que empezar ordenando parte de sus elementos como \( A \) y luego añadir unos cuantos más detrás.

Hemos probado que, dados dos conjuntos bien ordenados, o bien son semejantes, o bien uno es semejante a un segmento del otro. Esto tiene la traducción inmediata según la cual dados dos ordinales \( \alpha \) y \( \beta \), o bien \( \alpha=\beta \), o bien \( \beta<\alpha \), o bien \( \alpha=\beta \). En definitiva, la relación de orden en \( \text{Ord} \) es total. Más aún:

Teorema: \( (\text{Ord},\leq) \) es un conjunto bien ordenado.

Demostración
Sea \( X\subset \text{Ord} \) no vacío. Sea \( \alpha\in X \), sea \( (A,\leq) \) un conjunto bien ordenado tal que \( \alpha=(A,\leq) \). Consideramos el conjunto (bien definido)

\( Y=\{a\mid a\in A\land \text{ord}(A_a^<,\leq)\in X\} \).

Si \( Y=\emptyset \), entonces, para cada \( \beta\in X \), tenemos que \( \beta=\text{ord}(B,\leq_B) \) para un cierto conjunto bien ordenado \( (B,\leq_B) \) que no es semejante a ninguna sección inicial de \( A \), ya que en tal caso \( \beta\in Y \). Por lo tanto \( \alpha\leq \beta \) y concluimos que \( \alpha \) es el mínimo de \( X \).

Si \( Y\neq \emptyset \) podemos tomar \( a=\min Y \), así como el ordinal \( \alpha_0=\text{ord}(A_a^<,\leq)\in X \). Se cumple que \( \alpha_0 \) es el mínimo de \( X \), pues para todo \( \beta=\text{ord}(B,\leq_B)\in X \), se cumple que \( (B,\leq_B) \) no es semejante a ninguna sección inicial de \( (A_a^<,\leq) \), es decir, a un conjunto de la forma \( (A_a^<)_x^<=A_x^< \) para un \( x\in A \) con \( x<a \), pues en tal caso \( x\in Y \) sería un elemento menor que el mínimo. Por consiguiente, \( \alpha_0\leq \beta \).
[cerrar]

En particular es fácil ver que el menor ordinal es el ordinal cero, definido como

\( 0=\text{ord}(\emptyset,\emptyset) \),

es decir el ordinal del conjunto vacío con la relación vacía, que es trivialmente un buen orden en el conjunto vacío, por la lógica "cínica" del vacío.

Teorema: No existe un máximo ordinal o, equivalentemente, todo ordinal \( \alpha \) tiene un ordinal inmediatamente mayor al que llamaremos \( \alpha+1 \).

Demostración
Sea \( \alpha=\text{ord}(A',\leq_{A'}) \). Sea \( A=A'\times\{0\} \) con el orden dado por

\( (x,0)\leq_A (y,0)\leftrightarrow x\leq_{A'}y \).

Es claro que \( A \) con este orden es semejante a \( A' \), por lo que también \( \alpha=\text{ord}(A,\leq_A) \), pero ahora podemos tomar \( u=(0,1) \) con la garantía de que \( u\notin A \), y podemos formar el conjunto \( B=A\cup\{u\} \). Si extendemos el orden de \( A \) de modo que \( u \) sea el máximo de \( B \), es claro que tenemos un buen orden y que \( A=B^<_u \), luego \( \alpha=\text{ord}(A,\leq_A)<\text{ord}(B,\leq_B) \).

Con esto ya hemos probado que \( \alpha \) tiene un ordinal posterior, y la buena ordenación da que hay un mínimo ordinal mayor que \( \alpha \), pero vamos a ver que dicho mínimo es precisamente \( \alpha+1=\text{ord}(B,\leq_B) \).

Para ello tomamos un ordinal \( \gamma>\alpha \). Esto significa que \( \gamma=\text{ord}(C,\leq_C) \) y que existe una semejanza \( f:(A,\leq_A)\longrightarrow (C_c^<;\leq_C) \). Extendemos \( f \) a \( g:(B,\leq_B)\longrightarrow (C,\leq_C) \) haciendo \( g(u)=c \).

Hay dos posibilidades: o bien \( c \) es el máximo de \( C \) y así \( C=C_c^<\cup\{c\} \), con lo que \( g \) es una semejanza y \( \alpha+1=\gamma \), o bien \( c \) tiene un inmediato posterior \( c' \), de modo que \( g:(B,\leq_B)\longrightarrow (C_{c'}^<,\leq_C) \) semejanza, con lo que \( \alpha+1<\gamma \).

En ambos casos \( \alpha+1\leq \gamma \), luego \( \alpha+1 \) es el mínimo ordinal mayor que \( \alpha \).
[cerrar]

Notemos que el término \( \alpha+1 \) está estratificado con el mismo tipo que \( \alpha \), pues puede definirse como \( \alpha+1\equiv \min\{\beta\mid \beta\in \text{Ord}\land \beta>\alpha\} \). (Las llaves suben el tipo una unidad, pero el mínimo lo vuelve a bajar.)

Notemos que hay una diferencia notable entre cardinales y ordinales: mientras que hay un máximo cardinal \( \kappa_0 = |V| \), no hay un máximo ordinal.

Añado un último teorema elemental que hará falta luego:

Teorema: Si \( (A,\leq) \) es un conjunto bien ordenado y \( B\subset A \), entonces \( \text{ord}(B,\leq)\leq \text{ord}(A,\leq) \)

Demostración
Si fuera \( \text{ord}(A,\leq)< \text{ord}(B,\leq) \), existiría un \( b\in B \) y una semejanza \( f: (A\leq)\longrightarrow (B_b^<,\leq) \), que es una aplicación \( f:A\longrightarrow A \) inyectiva tal que conserva el orden y \( f(b)<b \). Hemos visto antes que esto es imposible.
[cerrar]



Esbozamos a continuación las definiciones de suma y producto de ordinales sin entrar en detalles, porque no nos van a ser necesarias posteriormente.

Definición: Si \( (A,\leq_A) \) y \( (B,\leq_B) \) son dos conjuntos bien ordenados definimos su suma como el conjunto bien ordenado \( ((A\times\{0\})\cup (B\times\{1\}),\leq) \), donde la relación viene dada por

\( (x,u)\leq (y,v)\leftrightarrow (u=0\land v=1)\lor (u=v\land x\leq y) \),

Es decir, la suma es un conjunto que contiene un subconjunto ordenado como \( A \) y otro ordenado como \( B \), de modo que todos los elementos de \( A \) son anteriores a los de \( B \). Es fácil probar que si \( A \) es semejante a \( A' \) y \( B \) es semejante a \( B' \) entonces la suma de \( A \) y \( B \) es semejante a la suma de \( A' \) y \( B' \), lo cual permite definir una operación

\( +:\text{Ord}\times \text{Ord}\longrightarrow \text{Ord} \)

de modo que \( \alpha+\beta \) se calcula tomando un conjunto de ordinal \( \alpha \), otro de ordinal \( \beta \) y haciendo \( \alpha+\beta \) igual al ordinal de la suma.

Si llamamos

\( 1\equiv \text{Ord}(\{\emptyset\},\{(\emptyset,\emptyset)\}) \),

es decir, el ordinal del conjunto \( \{\emptyset\} \) con la única relación de orden que podemos definir en él, es fácil ver que el ordinal \( \alpha+1 \) que hemos construido algo más arriba como el mínimo ordinal mayor que \( \alpha \) no es sino la suma de \( \alpha \) y \( 1 \) según la definición general.

También podemos definir una aplicación \( \cdot:\text{Ord}\times \text{Ord}\longrightarrow \text{Ord} \) a partir del producto lexicográfico, de dos conjuntos bien ordenados \( (A,\leq_A) \) y \( (B,\leq_B) \), que es el buen orden en \( (A\times B) \) dado por

\( (a,b)\leq (a',b')\leftrightarrow b<b'\lor (b=b'\land a\leq a') \)

Es decir, para comparar dos pares comparamos su componente de la derecha y, en caso de empate, miramos la otra. Así, el ordinal \( \alpha\beta \) se define como el ordinal del producto lexicográfico de un conjunto de ordinal \( \alpha \) por otro de ordinal \( \beta \).

A partir de estas definiciones se pueden demostrar las propiedades usuales de la suma y el producto de ordinales (la asociatividad, la no conmutatividad, la distributividad, etc.) Como ya hemos indicado, no necesitaremos nada de esto, y por ello lo pasamos por encima.



Para terminar definimos una aplicación que conecta los ordinales y los cardinales: Es evidente que si dos conjuntos bien ordenados \( (A,\leq_A) \) y \( (B,\leq_B) \) son semejantes, en particular los conjuntos \( A \) y \( B \) son equipotentes, porque las semejanzas son biyecciones, luego podemos definir la aplicación

\( \text{card}: \text{Ord}\longrightarrow K \)

mediante

\( \text{card}\equiv\{(\alpha,\kappa)\mid \exists A\leq\ ((A,\leq)\text{ es un c.b.o.}\land \alpha=\text{ord}(A,\leq)\land \kappa = |A|)\} \)

Así \( \text{card}(\alpha) \) es el cardinal de cualquier conjunto bien ordenado cuyo ordinal sea \( \alpha \).

Observemos que si \( \alpha<\beta \), entonces \( \alpha=\text{ord}(A,\leq_A) \), \( \beta=\text{ord}(B,\leq_B) \), y existe un \( b\in B \) y una semejanza \( f: (A,\leq_A)\longrightarrow (B_b^<,\leq_B) \), que en particular es una aplicación inyectiva \( f:A\longrightarrow B \). Por lo tanto \( |A|\leq |B| \), y esto es trivialmente cierto si \( \alpha=\beta \), luego hemos probado:

Teorema: \( \forall \alpha\beta\in\text{Ord}\ (\alpha\leq\beta\rightarrow \text{card}(\alpha)\leq \text{card}(\beta)) \).

De las definiciones de suma y producto de ordinales se deduce fácilmente que

\( \text{card}(\alpha+\beta)=\text{card}(\alpha)+\text{card}(\beta),\qquad \text{card}(\alpha\beta)=\text{card}(\alpha)\text{card}(\beta) \)

(donde a la izquierda del igual tenemos las operaciones con ordinales y a la derecha las operaciones con cardinales).

No necesitaremos estas dos últimas fórmulas. A lo sumo el caso de \( \alpha+1 \) que sí que hemos descrito con detalle. En la construcción de \( \alpha+1 \) se ve que si \( \alpha=(A,\leq) \), entonces \( \alpha+1=\text{ord}(A\cup \{u\},\leq) \), con \( A\cap \{u\}=\emptyset \), luego \( |A\cup\{u\}|=|A|+1 \), y así:

\( \text{card}(\alpha+1)=\text{card}(\alpha)+1 \),

donde el primer \( 1 \) es ordinal y el segundo cardinal.

Título: Re: NFA
Publicado por: Carlos Ivorra en 10 Enero, 2012, 12:42 am
He añadido un par de cosas en el mensaje anterior que me hacían falta luego: un teorema sencillo justo antes de la definición de la aritmética ordinal y una sección (separada por una raya) justo al final.
Título: Conjuntos finitos y numerables
Publicado por: Carlos Ivorra en 10 Enero, 2012, 12:54 am
Antes de introducir en la teoría el axioma de elección conviene exponer los resultados básicos sobre conjuntos finitos y numerables, pues no dependen de él.

Habíamos definido el conjunto de los conjuntos finitos como:

\( \mathcal F=\bigcup \mathbb{N} \).

En otras palabras, un conjunto es finito si su cardinal es un número natural. Explícitamente:

\( X\text{ es finito}\equiv \exists n\in \mathbb{N}\ |X|=n \).

Así vemos que la fórmula está estratificada, luego la podemos usar para definir conjuntos.

Hemos probado que todo cardinal menor o igual que un número natural es un número natural, lo que se traduce en que todo subconjunto de un conjunto finito es finito.

Hemos probado que la suma de dos números naturales es un número natural, lo que se traduce en que la unión de dos conjuntos finitos disjuntos es finita, pero si no son disjuntos da igual, porque

\( A\cup B = A\cup (B\setminus A) \)

y la unión es disjunta (y el segundo conjunto es finito porque está contenido en \( B \)).

Similarmente, el hecho de que el producto de números naturales sea un número natural se traduce en que el producto cartesiano de dos conjuntos finitos es un conjunto finito.

Otra propiedad relevante de los conjuntos finitos es la siguiente:

Teorema: Si \( f: A\longrightarrow A \) es una aplicación inyectiva y el conjunto \( A \) es finito, entonces \( f \) es suprayectiva.

Demostración
Sea \( B=A\setminus f[A] \). Entonces \( A=f[A]\cup B \) y la unión es disjunta, luego \( |A|=|f[A]|+|B| = |A|+|B| \). Como los sumandos son números naturales y contamos con las propiedades básicas de la aritmética (que se deducen todas de los axiomas de Peano), podemos simplificar, de modo que \( |B|=0 \), es decir, \( f[A]=A \) y \( f \) es suprayectiva.
[cerrar]

Como consecuencia:

Teorema: \( \mathbb{N} \) es infinito.

Pues la aplicación sucesor \( S:\mathbb{N}\longrightarrow \mathbb{N} \) es inyectiva y no suprayectiva.

Definición: Llamaremos \( \aleph_0\equiv |\mathbb{N}| \) y \( \omega\equiv \text{ord}(\mathbb{N},\leq) \).

La relación obvia entre ambos es: \( \text{card}(\omega)=\aleph_0 \).

Los números naturales (es decir, los cardinales de los conjuntos finitos), se llaman también cardinales finitos (aunque como conjuntos son infinitos, pues, por ejemplo, \( 1 \) es el conjunto (infinito) de todos los conjuntos con un elemento). Por lo tanto, tenemos que  \( \aleph_0 \) es un cardinal infinito. Hemos probado en su momento que todo cardinal finito es menor que todo cardinal infinito. Ahora vamos a probar que los únicos cardinales menores que \( \aleph_0 \) son los números naturales.

Para ello observamos antes lo siguiente:

Teorema: \( \forall n\in \mathbb{N}\ \mathbb{N}_n^< \) es finito.

Spoiler
Consideramos el conjunto \( X=\{n\mid n\in \mathbb{N}\land \mathbb{N}_n^<\text{ es finito}\} \), que está bien definido, porque la definición está estratificada. Podemos, pues, aplicar el principio de inducción. Claramente \( 0\in X \), pues \( \mathbb{N}_0^<=\emptyset \) y, si \( n\in X \), entonces

\( \mathbb{N}_{n+1}^<=\mathbb{N}_n^<\cup\{n\} \) es finito porque es unión de dos conjuntos finitos, el primero por hipótesis de inducción. Esto prueba el teorema.
[cerrar]

Observemos que los ordinales \( \text{ord}(\mathbb{N}_n^<,\leq) \) son por definición los ordinales menores que \( \omega \), luego el teorema anterior se puede reescribir así:

\( \forall \alpha\in \text{Ord}(\alpha<\omega\rightarrow \text{card}(\alpha)\in \mathbb{N}) \).

Como consecuencia:

Teorema: Los cardinales \( \kappa< \aleph_0 \) son los números naturales.

Spoiler
Si \( \kappa<\aleph_0 \), existe un \( A\subset \mathbb{N} \) tal que \( |A|=\kappa \). En el mensaje anterior hemos visto que \( \alpha=\text{ord}(A,\leq)\leq \omega \). No puede darse la igualdad, pues entonces \( \kappa = |A|=\text{card}(\alpha)=\text{card}(\omega)=\aleph_0 \), luego \( \kappa=\text{card}(\alpha)<\omega \), luego \( \kappa\in \mathbb{N} \), según hemos probado.
[cerrar]

Teorema: La restricción \( \text{card}: \text{Ord}_\omega^<\longrightarrow \mathbb{N} \) es biyectiva.

Spoiler
Ya hemos visto que los cardinales de los ordinales menore que \( \omega \) son números naturales. La aplicación es suprayectiva pues si \( n\in \mathbb{N} \) sabemos que \( n<\aleph_0 \), luego existe un \( A\subset \mathbb{N} \) tal que \( |A|=n \), luego \( \alpha=\text{ord}(A,\leq) \) es un ordinal cuyo cardinal es \( n \), y tiene que ser \( \alpha<\omega \), porque si fuera \( \omega\leq \alpha \) tendríamos que \( A \) contendría un segmento inicial semejante a \( \mathbb{N} \) (o bien sería él mismo semejante a \( \mathbb{N} \)) y sería infinito. Así pues, \( n=\text{card}(\alpha) \), con \( \alpha<\omega \).

Para probar la inyectividad basta ver que si \( \alpha<\beta<\omega \), entonces \( \text{card}(\alpha)<\text{card}(\beta) \). En efecto, sea \( \beta=\text{ord}(B,\leq) \) y sea \( b\in B \) tal que \( \alpha=\text{ord}(B_b^<,\leq) \). Tenemos entonces que \( \text{card}(\alpha)=|B_b^<| \) y \( \text{card}(\beta)=|B| \), ambos conjuntos son finitos y \( B_b^<\subsetneq B \). Como un conjunto finito no puede biyectarse con un subconjunto propio (pues tendríamos una aplicación inyectiva y no suprayectiva en sí mismo), es \( |B_b^<|<|B| \) y el teorema queda probado.
[cerrar]

Así pues, si llamamos ordinales finitos a los ordinales cuyos cardinales asociados son números naturales, tenemos una correspondencia biunívoca entre ordinales y cardinales finitos que conserva el orden. Además, los ordinales finitos son exactamente los ordinales menores que \( \omega \). La biyección también hace equivalentes la suma y el producto de ordinales y cardinales en el caso finito (aunque, según hemos dicho, no vamos a necesitar la aritmética ordinal).

En la práctica no distinguiremos entre un número natural y su correspondiente ordinal asociado. Ahora es evidente, por ejemplo, que todo conjunto finito puede ser bien ordenado (pues su cardinal es el cardinal de un ordinal).

Para terminar, y recordando que podemos contar con los hechos básicos de la aritmética de los números naturales, de la descomposición en unión disjunta

\( \mathbb{N}=\{n\mid n\in \mathbb{N}\land \exists k\in \mathbb{N}\ n=2k\}\cup \{n\mid n\in \mathbb{N}\land \exists k\in \mathbb{N}\ n=2k+1\} \)

Se deduce inmediatamente que \( \aleph_0+\aleph_0=\aleph_0 \), y de aquí que \( \forall n\in \mathbb{N}\ n+\aleph_0=\aleph_0 \),

pues, por propiedades generales de la suma de cardinales: \( \aleph_0\leq n+\aleph_0\leq \aleph_0+\aleph_0=\aleph_0 \).

Dejamos al lector la comprobación de que cualquiera de los argumentos usuales que prueban que \( \aleph_0\aleph_0=\aleph_0 \) es válido en este contexto.

No hemos definido todavía la exponenciación de cardinales porque requiere algunos tecnicismos, así que damos una demostración un tanto "artificial" de otro hecho básico:

Teorema: Si \( A \) es un conjunto finito, entonces \( \mathcal PA \) también lo es.

Spoiler
Podemos considerar el conjunto
\( X=\{n\mid n\in \mathbb{N}\land \forall A(|A|=n\rightarrow \mathcal PA\text{ es finito})\} \), claramente bien definido por una condición estratificada. Basta probar por inducción que \( X=\mathbb{N} \).

Si \( n=0 \), entonces \( A=\emptyset \) y \( \mathcal PA=\{\emptyset\} \), claramente finito.

Si es cierto para \( n \), sea \( |A|=n+1 \), de modo que \( A=B\cup\{u\} \), con \( |B|=n \) y \( u\notin B \). Podemos definir \( f: \mathcal PA\longrightarrow (\mathcal PB\times\{0\})\cup (\mathcal PB\times \{1\}) \) mediante

\( f(X)=\left\{\begin{array}{ll}(X,0)&\text{ si }u\notin X,\\ (X\setminus\{u\},1)&\text{ si }u\in X\end{array}\right. \)

Dejamos al lector la comprobación de que la definición es correcta, así como que \( f \) es biyectiva, por lo que

\( |\mathcal PA| = |\mathcal PB|\cdot 1+|\mathcal PB|\cdot 1 = 2|\mathcal PB| \)

es un cardinal finito, pues \( |\mathcal PB| \) lo es por hipótesis de inducción.
[cerrar]
Título: El axioma de elección
Publicado por: Carlos Ivorra en 11 Enero, 2012, 12:22 pm
Aunque todavía hay muchos resultados importantes de NFA que tenemos que exponer y que no dependen del axioma de elección, lo cierto es que seguir evitando su uso llevaría a una exposición un tanto artificial, así que vamos a introducirlo y en lo sucesivo señalaremos explícitamente los teoremas que lo usan.

La versión más adecuada en NFA para el axioma de elección es la siguiente:

Axioma de elección (AE): \( \exists F(F: \mathcal PV\setminus\{\emptyset\}\longrightarrow V\land \forall A\in \mathcal PV\setminus \{\emptyset\}\exists u\in A\ F(A)=\{u\}) \)

En palabras, existe una función que a cada conjunto no vacío \( A \) le asigna un conjunto de la forma \( \{u\} \) con \( u\in A \).

Sin duda hubiera sido más natural pedir que \( F(A)\in A \), pero esta fórmula no está estratificada ya que \( F(A) \) tiene que tener el mismo tipo que \( A \), mientras que su tipo debería ser una unidad inferior al de \( A \) para que la pertenencia estuviera estratificada.

Vamos a probar que el axioma de elección equivale al lema de Zorn, y de paso probamos una equivalencia sencilla que es idéntica a una versión de AE en ZFC. Todos los argumentos son igualmente válidos en ZFC.

Teorema: Las afirmaciones siguientes son equivalentes:

1) El axioma de elección.

2) Si \( A \) es una familia de conjuntos no vacíos disjuntos dos a dos, entonces existe un conjunto que contiene exactamente un elemento de cada conjunto de \( A \).

3) Lema de Zorn: Todo conjunto parcialmente ordenado en el que toda cadena tiene una cota superior tiene un elemento maximal.


1-> 2)
Si \( F \) es la función dada por AE, basta tomar \( \bigcup F[A] \).

(La idea es que si \( A=\{X, Y, Z,\ldots\} \) entonces \( F[A]=\{\{x\},\{y\},\{z\},\ldots\} \) con \( x\in X \), etc., y \( \bigcup F[A]=\{x, y, z, \ldots \} \).)
[cerrar]

2) -> 3)
Sea \( (A,\leq) \) un conjunto parcialmente ordenado que cumpla las hipótesis (omitimos las definiciones de conjunto parcialmente ordenado, cadena, maximal, etc.). Para cada cadena \( C\subset A \) sea

\( X_C\equiv\{X\mid X\subset A\land \exists u\in A(\forall v\in C\ v< u\land X=C\cup\{u\})
 \)

\(
\lor ((\lnot\exists u\in A\forall v\in C\ v<u)\land X=C))
\} \)

Así, si existe un \( u\in A \) que es mayor que todos los elementos de \( C \), y en tal caso \( X_C \) contiene todas las extensiones \( C\cup\{u\} \) en dichas condiciones, o no existe un \( u \) así y entonces \( X_C=\{C\} \).

Como siempre, hay que comprobar que las fórmulas que usamos al definir conjuntos están estratificadas, lo cual no ofrece dificultad. En particular vemos que \( X_C \) está estratificado y que su tipo es una unidad mayor que el de \( C \).

Sea \( S=\{X\mid \exists C(C\text{ es una cadena en } (A,\leq)\land X=\{C\}\times X_C)\} \).

Nuevamente no hay dificultad en comprobar que la definición de \( S \) está estratificada. Explícitamente:

\( \exists C(C_1\subset A_1\land \forall u_0v_0\in C_1(u_0\leq_1 v_0\lor v_0\leq_1 u_0)\land X_2=\{C_1\}_2\times (X_{C_1})_2) \).

Claramente, \( S \) es una familia de conjuntos no vacíos disjuntos dos a dos, luego por 2) existe un conjunto \( G \) que contiene exactamente un elemento de cada elemento de \( S \). Es claro entonces que \( G \) es una función definida sobre el conjunto de todas las cadenas de \( (A,\leq) \) y que \( G(C)\in X_C \).

En definitiva, hemos construido una función que extiende cada cadena cuando es posible extenderla y la deja igual cuando no es posible.

Diremos que \( C \) es una buena cadena en \( (A,\leq) \) si \( C \) está bien ordenada por \( \leq \) y, para cada \( a\in C \) se cumple que

\( G(\{u\mid u\in C\land u<a\})=\{u\mid u\in C\land u\leq a\} \).

Es decir, se trata de una cadena que "se ha ido construyendo" añadiendo cada vez el elemento establecido por \( G \). Es fácil comprobar lo siguiente:

a) Si \( C \) es una buena cadena y \( a\in C \), entonces los conjuntos \( \{u\mid uin C\land u<a\} \) y \( \{u\mid u\in C\land u\leq a\} \) son también buenas cadenas.

b) \( \emtyset \) es una buena cadena.

c) Si \( C \) es una buena cadena, \( G(C) \) también lo es.



Veamos lo siguiente:

Si \( C \) y \( C' \) son buenas cadenas y \( a\in C \) cumple que \( \{u\mid u\in C\land u<a\}=\{u\mid u\in C'\land u<a\} \), entonces \( a\in C' \) o bien \( C'=\{u\mid u\in C\land u<a\} \).

En efecto, suponemos que \( C'\neq \{u\mid u\in C\land u<a\} \) y vamos a probar que \( a\in C' \).

Como \( C \) y \( C' \) tienen los mismos elementos bajo \( a \), tiene que haber un \( a'\in C' \) mayor que todos los \( u\in C' \) tales que \( u<a \). Como \( C' \) está bien ordenado, podemos suponer que \( a' \) es el mínimo elemento de \( C' \) mayor que todos los \( u\in C' \) tales que \( u<a \), es decir,

\( \{u\mid u\in C'\land u<a'\}=\{u\mid u\in C'\land u<a\}=\{u\mid u\in C\land u<a\} \).

Como \( C \) y \( C' \) son buenas cadenas, aplicando \( G \) obtenemos un conjunto cuyo máximo tiene que ser tanto \( a' \) como \( a \), luego \( a=a'\in C' \).



Veamos ahora que dos buenas cadenas \( C \) y \( C' \) están necesariamente contenidas una en la otra. Si \( C \) no está contenida en \( C' \), podemos tomar el mínimo \( a\in C\setminus C' \), de modo que \( \{u\mid u\in C\land u<a\}\subset \{u\mid u\in C'\land u<a\} \). Pero tiene que darse la igualdad, pues en caso contrario existiría un \( a'\in C' \) tal que \( a'<a \), pero \( a'\notin C \). Podríamos tomar el mínimo posible. Así \( \{u\mid u\in C\land u<a'\}=\{u\mid u\in C'\land u<a'\} \) y, según hemos visto, esto implica que \( C=\{u\mid u\in C'\land u<a'\} \), pero entonces \( a<a' \), contradicción. Esto prueba que \( \{u\mid u\in C\land u<a\}=\{u\mid u\in C'\land u<a\} \) y, aplicando de nuevo la propiedad que hemos probado más arriba, \( C'\subset C \).

Ahora es fácil ver que la unión de todas las buenas cadenas es una buena cadena. Llamémosla \( C \). Entonces, \( G(C) \) también es una buena cadena, pero necesariamente ha de ser \( G(C)=C \) (pues no puede ser estrictamente mayor, ya que \( C \) es la mayor de todas las buenas cadenas), y esto significa que cualquier cota superior \( m \) de \( C \) (que existe por hipótesis) tiene que cumplir \( m\in C \), ya que de lo contrario permitiría extender \( C \). Así pues, \( m \) es el máximo de \( C \), y tiene que ser un maximal de \( A \), ya que cualquier elemento por encima de \( m \) sería una cota superior de \( C \) fuera de \( C \) que, por consiguiente, permitiría extender \( C \).
[cerrar]

 3) -> 1)

Llamamos \( A \) al conjunto de todas las funciones de elección (como la que postula el axioma de elección) definidas sobre un subconjunto de \( \mathcal PV \), es decir:

\( A\equiv\{f\mid \exists X(X\subset \mathcal PV\setminus\{\emptyset\}\land f: X\longrightarrow V\land \forall Y\in X\exists u\in Y\ f(Y)=\{u\})\} \)

Podría pensarse que la definición no es correcta, porque la variable \( X \) debe tener el mismo tipo que \( \mathcal PV\setminus \{\emptyset\} \), que es una unidad mayor que el de \( V \), mientras que la fórmula \( f:X\longrightarrow V \) requiere que \( X \) tenga el mismo tipo que \( V \), pero es que podemos llamar \( P=\mathcal PV\setminus \{\emptyset\} \) y \( Q=V \) y considerarlos como parámetros, es decir, aplicar el axioma de formación de conjuntos a la fórmula estratificada

\(  \exists X(X\subset P\land f: X\longrightarrow Q\land \forall Y\in X\exists u\in Y\ f(Y)=\{u\}) \)

lo cual es correcto para todo valor de los parámetros \( P \) y \( Q \), y luego particularizar a los conjuntos que hemos indicado.

Es inmediato que \( A \) satisface las hipótesis del lema de Zorn con el orden dado por la inclusión (una cota superior de cada cadena \( C \) es \( \bigcup C \)), por lo que existe una función de elección maximal \( F: X\longrightarrow V \). Basta probar que \( X=\mathcal PV\setminus\{\emptyset\} \), pero es que si existiera un \( Y\in \mathcal PV\setminus\{\emptyset\} \) que no estuviera en \( X \), podríamos tomar \( u\in X \), y entonces \( F\cup\{(Y,\{u\})\} \) sería un elemento de \( A \) mayor que la función maximal \( F \), contradicción.
[cerrar]

La forma equivalente de AE que usaremos casi exclusivamente es ésta:

Teorema: Las afirmaciones siguientes son equivalentes:

1) El axioma de elección.

2) \( V \) admite un buen orden.

3) Todo conjunto admite un buen orden.


 1) -> 2)
Sea \( X \) el conjunto de todos los buenos órdenes, es decir, cada elemento de \( X \) es un conjunto \( R \) tal que existe un conjunto \( A \) tal que \( R\subset A\times A \) es un buen orden en \( A \). El conjunto \( A \) es necesariamente \( A = \mathcal DR \).

Definimos en \( X \) el orden parcial dado por \( R\leq S \) si \( R\subset S \) y, para todo \( r\in \mathcal DR \) y todo \( s\in \mathcal DS\setminus \mathcal DR \) se cumple \( r\,S\,s \). En otras palabras, todos los elementos de \( \mathcal DS \) que no están en \( \mathcal DR \) son posteriores a los de \( \mathcal DR \).

Es fácil ver que \( (X,\leq) \) satisface las hipótesis del lema de Zorn y un elemento maximal es necesariamente un buen orden sobre todo \( V \).
[cerrar]

La equivalencia 2) \( \leftrightarrow \) 3) es trivial, y para probar que 2) \( \rightarrow \) 1) fijamos un buen orden en \( V \) y definimos una función de elección mediante \( F(A)=\{\min A\} \).

En el mensaje siguiente veremos las consecuencias del axioma de elección sobre los cardinales infinitos.
Título: Cardinales infinitos en NFA
Publicado por: Carlos Ivorra en 13 Enero, 2012, 07:49 pm
En este mensaje mostramos las consecuencias del axioma de elección sobre los cardinales infinitos en NFA. Todas las demostraciones son variaciones mínimas de argumentos válidos en ZFC.

Empezamos con una consecuencia elemental:

Teorema: Sean \( A \) y \( B \) dos conjuntos tales que \( A\neq \emptyset \). Entonces \( |A|\leq |B| \) si y sólo si existe \( f: B\longrightarrow A \) suprayectiva.

Demostración
Si \( |A|\leq |B| \) existe \( g: A\longrightarrow B \) inyectiva. Tomamos \( a\in A \) y definimos \( f= g^{-1}\cup ((B\setminus g[A])\times \{a\}) \). Es claro que \( f: B\longrightarrow A \) suprayectiva.

Recíprocamente, si existe \( f \), tomamos un buen orden \( \leq \) en \( B \) y definimos \( g: A\longrightarrow B \) mediante \( g(a)=\min f^{-1}(\{a\}) \).

Notemos que la definición es correcta porque \( \{a\} \) tiene tipo una unidad mayor que \( a \), luego \( f^{-1}[\{a\}] \) tiene este mismo tipo (una unidad mayor) y el mínimo reduce el tipo una unidad, luego \( a \) y \( g(a) \) tienen el mismo tipo, como requiere la estratificación.

Es fácil ver que \( g \) es inyectiva, luego \( |A|\leq |B| \).
[cerrar]

Observemos que sólo la implicación \( \Leftarrow \) requiere AE y, más concretamente, requiere que \( B \) tenga un buen orden. En particular, el resultado se prueba sin AE cuando \( B \) es finito o numerable.

Teorema: La aplicación \( \text{card}: \text{Ord}\longrightarrow K \) es suprayectiva.

Demostración
Dado cualquier cardinal \( \kappa \), tomamos un conjunto tal que \( |A|=\kappa \), tomamos un buen orden \( \leq \) en \( A \) y consideramos \( \alpha = \text{ord}(A,\leq) \). Es claro entonces que \( \text{card}(\alpha)=\kappa \).
[cerrar]

Definición: Llamaremos \( \text{In}: K\longrightarrow \text{Ord} \) a la aplicación (inyectiva) dada por \( \text{In}(\kappa)=\min \text{card}^{-1}[\{\kappa\}] \)

El ordinal \( \text{In}(\kappa) \) se llama ordinal inicial de \( \kappa \), y es el mínimo ordinal tal que los conjuntos ordenados con el tipo de orden que representa tienen cardinal \( \kappa \). Equivalentemente, si \( (A,\leq) \) es un conjunto bien ordenado tal que \( |A|=\kappa \) y para todo \( a\in A \) se cumple que \( |A_a^<|<\kappa \), entonces \( \text{ord}(A,\leq)=\text{In}(\kappa) \).

Es claro que la aplicación \( \text{In} \) conserva el orden, es decir:

\( \kappa<\mu\leftrightarrow \text{In}(\kappa)<\text{In}(\mu) \),

luego el orden de \( K \) es semejante al de \( \text{In}[K]\subset \text{Ord} \), luego:

Teorema: El conjunto \( K \) de todos los cardinales está bien ordenado.

Habíamos probado que los únicos cardinales menores que \( \aleph_0 \) son los números naturales, luego, ahora que sabemos que \( K \) está totalmente ordenado, podemos afirmar algo más directo:

Teorema: \( \aleph_0 \) es el menor cardinal infinito.

Claramente \( \text{In}(\aleph_0)=\omega \).

Teorema: Si \( \kappa \) es un cardinal infinito, entonces \( \text{In}(\kappa) \) es un ordinal límite, es decir, no tiene un inmediato anterior.

Demostración
Supongamos que \( \text{In}(\kappa)=\alpha+1 \) y vamos a llegar a una contradicción. Concretamente, esto supone que \( \text{card}(\alpha)<\text{card}(\alpha+1) \) y vamos a probar que en realidad se da la igualdad.

Sabemos que \( \text{card}(\alpha+1)=\text{card}(\alpha)+1 \), donde \( \text{card}(\alpha) \) es un cardinal infinito. Por lo tanto, basta probar que si \( \mu \) es un cardinal infinito, entonces \( \mu=\mu+1 \).

Ahora bien, como todo conjunto de cardinal \( \mu \) contiene un subconjunto numerable, existe un cardinal \( \nu \) tal que \( \mu=\nu+\aleph_0 \), luego \( \mu+1=\nu+\aleph_0+1=\nu+\aleph_0=\mu \), donde hemos usado que \( \aleph_0+1=\aleph_0 \), como ya habíamos probado.
[cerrar]

Definición: Sea \( (A,\leq) \) un conjunto bien ordenado. Definimos el orden canónico en \( A\times A \) como el dado por

\( (a,b)\leq (c,d)\leftrightarrow \max\{a,b\}<max\{c,d\}\lor (\max\{a,b\}=\max\{c,d\}\land a<c) \)

\( \lor (\max\{a,b\}=\max\{c,d\}\land a=c\land b\leq d) \)

Es decir, para comparar dos pares empezamos comparando el máximo de cada par, en caso de empate comparamos las primeras componentes y en caso de empate comparamos las segundas.

Una comprobación rutinaria muestra que esta relación es de hecho un buen orden. Para encontrar el mínimo de un conjunto de pares nos quedamos con los pares cuya máxima componente sea la mínima posible, de entre todos ellos nos quedamos con los que tengan la menor primera componente, y de entre todos ellos nos quedamos con el que tenga la menor segunda componente. El único par que nos quede es el mínimo del conjunto dado.

Teorema (AE) Si \( \kappa \) es un cardinal infinito, entonces \( \kappa\kappa=\kappa \).

Demostración
Podemos considerar el conjunto

\( X=\{\kappa\mid \kappa\in K\land \aleph_0\leq \kappa\land \kappa\kappa\neq \kappa\} \).

Queremos probar que es vacío. Si suponemos que no lo es, entonces tendrá un mínimo elemento \( \kappa \). Puesto que todo cardinal cumple \( \kappa\leq\kappa\kappa \) tenemos que \( \kappa \) es un cardinal infinito para el cual \( \kappa<\kappa\kappa \), pero todo cardinal \( \mu<\kappa \) cumple \( \mu<\aleph_0 \) o bien \( \mu\mu=\mu \). En ambos casos \( \mu\mu<\kappa \).

Sea \( \alpha=\text{In}(\kappa) \) y sea \( \text{ord}(A,\leq)=\alpha \). Hemos visto que \( \alpha \) es un ordinal límite, lo cual se traduce en que \( (A,\leq) \) no tiene un máximo elemento. (Si tuviera un máximo \( m \), entonces sería \( \alpha = \text{ord}(A_m^<,\leq)+1 \).)

Consideramos en \( A\times A \) el buen orden canónico y observamos que si \( (c,d)\in A\times A \) y \( e>\max\{c,d\} \) (que existe, porque no hay máximo), entonces

\( (A\times A)^<_{(c,d)}\subset A^<_e\times A_e^< \).

En efecto, si \( (a,b)\in (A\times A)^<_{(c,d)} \), entonces \( \max\{a,b\}\leq\max\{c,d\}<e \), luego \( a,b\in A_e^< \).

Por consiguiente, \( |(A\times A)^<_{(c,d)}|\leq |A^<_e\times A_e^<|=|A_e^<||A_e^<|<\kappa \), donde hemos usado que, como \( \text{ord}(A,\leq) \) es el ordinal inicial de \( \kappa \), todos sus segmentos iniciales tienen cardinal \( <\kappa \), junto con la minimalidad de \( \kappa \).

Así pues, \( (A\times A,\leq) \) tiene todos sus segmentos de cardinal \( <\kappa \), pero por otra parte, el ordinal \( \beta=\text{ord}(A\times A,\leq) \) tiene cardinal \( \kappa\kappa>\kappa \), luego, \( \beta>\alpha \), luego \( (A\times A,\leq) \) debería tener un segmento de ordinal \( \alpha \) y, por consiguiente, de cardinal \( \kappa \), contradicción.
[cerrar]

A partir de aquí obtenemos fácilmente el criterio para calcular cualquier suma o producto de cardinales:

Teorema: Si \( \kappa \) y \( \mu \) son cardinales no nulos y uno es infinito, entonces \( \kappa\mu=\max\{\kappa,\mu\} \).

Demostración
Pongamos que \( \kappa\leq\mu \). Entonces \( \mu\leq\kappa\mu\leq \mu\mu=\mu \).
[cerrar]

Teorema: Si \( \kappa \) y \( \mu \) son cardinales y uno es infinito, entonces \( \kappa+\mu=\max\{\kappa,\mu\} \).

Spoiler
Pongamos que \( \kappa\leq \mu \). Entonces \( \mu\leq \kappa+\mu\leq\mu+\mu=2\mu\leq \mu\mu=\mu \).
[cerrar]

Hasta aquí hemos demostrado (sin ánimo de ser exhaustivos) los resultados básicos de NFA que son esencialmente análogos a resultados de ZFC (aunque las definiciones de los conceptos involucrados difieran sustancialmente), y que se demuestran con argumentos similares, cuando no idénticos. A partir del próximo mensaje empezaremos a exponer los resultados de NFA que son esencialmente diferentes a lo que sucede en ZFC. Empezaremos analizando cómo resuelve NFA las paradojas de Cantor y de Burali-Forti.
Título: Las paradojas de Cantor y Burali-Forti
Publicado por: Carlos Ivorra en 15 Enero, 2012, 04:52 pm
Ya hemos visto que la paradoja de Russell no se aplica a NFA porque el conjunto paradójico

\( R=\{X\mid X\notin X\} \)

no es tal conjunto. Por una parte, no puede probarse que existe por el esquema de formación de conjuntos, ya que la fórmula que lo define no está estratificada y, más aún, el hecho de que \( R \) sea contradictorio pasa de ser una paradoja a ser una demostración de que \( R \) no existe.

Ahora vamos a ver qué sucede en NFA con otras dos paradojas clásicas de la teoría de conjuntos (ninguno de los resultados de este mensaje usa el axioma de elección):



La paradoja de Cantor

La paradoja de Cantor es consecuencia del llamado teorema de Cantor. Recordamos su enunciado y su demostración:

Teorema de Cantor: Para todo conjunto \( X \) se cumple que \( |X|<|\mathcal PX| \).

Demostración : Para probar que \( |X|\leq |\mathcal PX| \) basta observar que la aplicación \( f:X\longrightarrow \mathcal PX \) dada por \( x\mapsto \{x\} \) es inyectiva. Si se diera la igualdad, tendríamos una biyección \( g: X\longrightarrow \mathcal PX \) y podríamos definir el conjunto

\( C=\{x\mid x\in X\land x\notin g(x)\} \).

Puesto que \( C\in \mathcal PX \), existe un \( x_0\in X \) tal que \( C=g(x_0) \), pero entonces tenemos una contradicción, pues si \( x_0\in C \) por definición de \( C \) resulta que \( x_0\notin g(x_0)=C \), mientras que si \( x_0\notin C=g(x_0) \), por definición de \( C \) debería ser \( x_0\in C \). Esto prueba que la desigualdad es estricta.

Aparentemente, este hecho lleva a una contradicción en NFA cuando se aplica al conjunto \( X=V \), pues nos dice que \( |V|<|\mathcal PV| \), cuando, por otra parte, es obvio que \( \mathcal PV\subset V \) y por lo tanto debería ser \( |\mathcal PV|\leq |V| \). Ésta es la paradoja de Cantor, pero no es una paradoja en NFA, porque el argumento no es formalizable en NFA. ¿Por qué?

Solución
Las dos partes de la demostración del teorema de Cantor son incorrectas como demostraciones en NFA. La primera parte supone que podemos definir la aplicación

\( f=\{u\mid \exists x\in X\ u=(x,\{x\})\} \),

pero la definición de \( f \) no está estratificada, ya que \( \{x\} \) tiene tipo una unidad superior a \( x \) y las dos componentes del par ordenado \( (x,\{x\}) \) deberían tener el mismo tipo.

Por otra parte, la definición del conjunto \( C \) tampoco está estratificada, pues los dos términos de \( x\notin g(x) \) tienen el mismo tipo.

Así pues, el contraejemplo que ofrece \( V \) prueba que el teorema de Cantor, al menos con su enunciado válido en ZFC no es válido en NFA.
[cerrar]



La antinomia de Burali-Forti

Tenemos definido un buen orden canónico en el conjunto \( \text{Ord} \) de todos los ordinales, luego podemos considerar el ordinal \( \Omega=\text{ord}(\text{Ord},\leq) \). La conocida como antinomia de Burali-Forti es una paradoja que se desprende del teorema siguiente:

Teorema: Si \( \alpha \) es un ordinal, entonces \( \text{ord}(\text{Ord}^<_\alpha,\leq)=\alpha \).

Demostración: Sea \( \alpha=\text{Ord}(A,\leq_A) \) y consideremos la aplicación \( f: A\longrightarrow \text{Ord}_\alpha^< \) dada por \( f(a)=\text{ord}(A_a^<,\leq) \). Notemos que, por la definición de la relación de orden en \( \text{Ord} \), se cumple que \( f(a) \) es ciertamente un ordinal menor que \( \alpha \). Además es claro que si \( a<a' \), entonces \( A_a^< \) es un segmento inicial de \( A_{a'}^< \), luego \( f(a)<f(a') \).

Por otro lado, todo \( \beta<\alpha \) es el ordinal de un segmento inicial de \( (A,\leq) \), luego \( f \) es suprayectiva y, por consiguiente, una semejanza. Esto prueba que \( \alpha=\text{ord}(A,\leq)=\text{ord}(\text{Ord}_\alpha^<,\leq) \).

La paradoja consiste en que \( \text{ord}(\text{Ord}_\alpha^<,\leq) \) es el ordinal de un segmento de \( \text{Ord} \), luego, por definición de la relación de orden en los ordinales, es un ordinal menor que \( \Omega \). En resumen, lo que prueba el teorema anterior es que todo ordinal cumple \( \alpha<\Omega \), y esto es absurdo, pues no puede ser \( \Omega<\Omega \).

Nuevamente, este argumento no es formalizable en NFA. ¿Por qué?

Solución:
El problema es el mismo que en la demostración del teorema de Cantor: la aplicación \( f \) no está bien definida, ya que el término \( \text{ord}((A_{a_0}^<)_1,\leq_1)_2 \) tiene tipo dos unidades mayor que \( a \), mientras que para definir así \( f(a) \) debería tener el mismo tipo que \( a \). Por lo tanto, el "teorema" no es realmente un teorema de NFA y no hay contradicción.
[cerrar]



Los argumentos que hemos dado bastan para rebatir los argumentos paradójicos que afectan a la teoría intuitiva de conjuntos, pero son puramente negativos y, por consiguiente, no nos permiten entender qué sucede realmente en el universo de NFA para que no se cumplan los dos "teoremas" que acabamos de "demostrar". Empezaremos demostrando una versión del teorema de Cantor válida en NFA, para lo cual necesitamos la definición siguiente:

Definición: Para todo conjunto \( X \), definimos \( \mathcal P_1X= \{\{x\}\mid x\in X\} \).

Notemos que la definición es correcta y que \( \mathcal P_1X \) tiene tipo una unidad mayor que \( X \): \( \{\{x_0\}_1\mid x_0\in X_1\}_2 \). El teorema siguiente es "de verdad":

Teorema de Cantor: Para todo conjunto \( X \) se cumple que \( |\mathcal P_1X|<|\mathcal PX| \).

Demostración:
Puesto que \( \mathcal P_1X\subset \mathcal PX \) tenemos que \( |\mathcal P_1X|\leq |\mathcal PX| \). Si se diera la igualdad existiría una biyección \( g: \mathcal P_1X\longrightarrow \mathcal PX \), y podríamos definir el conjunto

\( C=\{x\mid x\in X\land x\notin f(\{x\})\} \).

La definición es correcta, pues el tipo de \( f(\{x\}) \) es el mismo que el de \( \{x\} \), que a su vez es una unidad mayor que el de \( x \), tal y como se requiere para que la fórmula \( \lnot x\in f(\{x\}) \) esté estratificada. Como \( C\in \mathcal PX \), existe un \( x_0\in X \) tal que \( f(\{x_0\})=C \), y a partir de aquí llegamos a la misma contradicción que en la prueba del teorema de Cantor en ZFC: \( x_0\in C\leftrightarrow x_0\notin C \).
[cerrar]

En particular, \( |\mathcal P_1V|<|\mathcal PV|\leq |V| \), luego \( |\mathcal P_1V|< |V| \).

Explícitamente: ¡el conjunto \( \{\{x\}\mid x\in V\} \) tiene menos elementos que \( V \)! (Aquí es donde empieza a verse que NFA es... digamos... peculiar). En particular:

Teorema: No existe ninguna aplicación \( f:V\longrightarrow V \) que cumpla \( f(x)=\{x\} \).

Pues si existiera probaría que \( |V|=|\mathcal P_1V| \).

Naturalmente, esto plantea una cuestión: dado un conjunto \( X \), ¿qué cardinal tiene \( \mathcal P_1X \)? Lo razonable sería que la respuesta fuera \( |X| \), pero acabamos de encontrar un contraejemplo. Ésta es la típica pregunta que se "responde" con una definición, pero antes conviene hacer una observación muy simple:

Teorema: Si \( |X|=|Y| \), entonces \( |\mathcal P_1X|=|\mathcal P_1Y| \).

Demostración:
Sea \( f: X\longrightarrow Y \) biyectiva. Entonces definimos

\( \bar f=\{(\{x\},\{y\})\mid x\in X\land y = f(x)\} \),

que está bien definida y es claro que \( \bar f: \mathcal P_1X\longrightarrow \mathcal P_1Y \) biyectiva, luego \( |\mathcal P_1X|=|\mathcal P_1Y| \).
[cerrar]

Así pues, el cardinal de \( \mathcal P_1X \) no depende del conjunto \( X \) en particular, sino únicamente de su cardinal.

Definición: Para cada cardinal \( \kappa \) definimos \( T(\kappa)\equiv \mu\mid \exists X(\kappa=|X|\land \mu=|\mathcal P_1X|) \).

En estos términos, para todo conjunto \( X \), podemos decir que \( |\mathcal P_1X|=T(|X|) \).

Notemos que \( T(\kappa) \) es un término estratificado cuyo tipo es una unidad mayor que el de \( \kappa \):

\( \mu_2\mid \exists X(\kappa_1=|X_0|_1\land \mu_2=|(\mathcal P_1X_0)_1|_2) \).

En particular, no podemos ver a \( T \) como una función \( T:K\longrightarrow K \), pues el conjunto \( \{(\kappa, T(\kappa)\mid \kappa\in K\} \) no estaría bien definido. Por el contrario, \( T(\kappa) \) es un término definido exactamente en las mismas condiciones que los términos \( \mathcal PX \) o \( \mathcal P_1X \), para los que sucede lo mismo: no existe ninguna aplicación \( \mathcal PV\longrightarrow \mathcal PV \) dada por \( X\mapsto \mathcal PX \), porque la definición no estaría estratificada.

Algunas propiedades sencillas de la operación \( T \):

Teorema: Se cumple:

1) \( \forall \kappa\mu\in K(\kappa\leq \mu\leftrightarrow T(\kappa)\leq T(\mu)) \)

2) \( \forall \kappa\mu\in K(\kappa= \mu\leftrightarrow T(\kappa)= T(\mu)) \)

3) \( \forall \kappa\mu\in K\ T(\kappa+\mu)=T(\kappa)+T(\mu) \)

4) \( \forall \kappa\mu\in K\ T(\kappa\mu)=T(\kappa)T(\mu) \).

Demostración:
1) Tomemos conjuntos tales que \( \kappa=|X| \) y \( \mu=|Y| \). Si \( \kappa\leq \mu \) existe \( f: X\longrightarrow Y \) inyectiva, y entonces \( \bar f = \{(\{x\},\{y\})\mid x\in X\land y=f(x)\} \) cumple que \( \bar f: \mathcal P_1X\longrightarrow \mathcal P_1Y \), luego \( T(\kappa)=|\mathcal P_1X|\leq |\mathcal P_1Y|=T(\mu) \).

Recíprocamente, si \( T(\kappa)\leq T(\mu) \), existe \( \bar f: \mathcal P_1X\longrightarrow \mathcal P_1Y \) inyectiva, con lo que \( f=\{(x,y)\mid x\in X\land f(\{x\})=\{y\}\} \) cumple que \( f: X\longrightarrow Y \) inyectiva, luego \( \kappa\leq \mu \).

2) Se sigue inmediatamente de 1).

3) Podemos suponer que \( X\cap Y=\emptyset \). Entonces \( \mathcal P_1(X\cup Y)=\mathcal P_1X\cup \mathcal P_1Y \), y la unión es disjunta, con lo que

\( T(\kappa+\mu)=|\mathcal P_1(X\cup Y)|=|\mathcal P_1X\cup \mathcal P_1Y|=|\mathcal P_1X|+|\mathcal P_1Y|=T(\kappa)+T(\mu) \).

4) Basta razonar con la biyección \( f: \mathcal P_1(X\times Y)\longrightarrow \mathcal P_1X\times \mathcal P_1Y \) dada por \( f(\{(x,y)\})=(\{x\},\{y\}) \).
[cerrar]

Escribiremos \( T^2(\kappa)\equiv T(T(\kappa)), T^3(\kappa)\equiv T(T(T(\kappa))) \), etc. Recordemos que habíamos definido \( \kappa_0=|V| \). Llamaremos \( \kappa_1=T(\kappa_0)=|\mathcal P_1V| \), \( \kappa_2=T^2(\kappa_0)=T(\kappa_1), \kappa_3 = T^3(\kappa_0)=T(\kappa_2) \), etc.

En estos términos, el teorema de Cantor puede expresarse diciendo que, para todo conjunto \( X \) se cumple la relación:

\( T(|X|)<|\mathcal PX| \).

Hemos visto que \( |\mathcal P_1V|<|V| \), lo que ahora se traduce en la relación \( T(\kappa_0)<\kappa_0 \) o, más simplemente, en que \( \kappa_1<\kappa_0 \). Si aplicamos \( T \) a esta desigualdad obtenemos que \( T(\kappa_1)<T(\kappa_0) \), es decir, que \( \kappa_2<\kappa_1 \) y, en general, vemos que los cardinales que acabamos de definir cumplen:

\( \cdots < \kappa_3<\kappa_2<\kappa_1<\kappa_0 \).

Aunque no hemos usado en ningún momento el axioma de elección, esto es compatible con dicho axioma, el cual implica que el conjunto \( K \) de todos los cardinales está bien ordenado. Esto es un teorema de NFA + AE y, no obstante, no contradice a la existencia de esta sucesión decreciente de cardinales.

El lector ingenuo podría alegar que \( \{\kappa_0,\kappa_1,\kappa_2,\ldots \} \) es un subconjunto de \( K \) no vacío y sin mínimo elemento, y tendría razón excepto en lo de que "es un conjunto". No se puede demostrar la existencia de un conjunto cuyos elementos sean precisamente los cardinales \( \kappa_n \). No hay contradicción, aunque resulta patente que, en cierto sentido que nada tiene que ver con la inconsistencia, NFA "no es muy digno de crédito".

El teorema siguiente determina la imagen del operador \( T \):

Teorema: Un cardinal \( \kappa \) es de la forma \( \kappa=T(\mu) \), para cierto cardinal \( \mu \) si y sólo si \( \kappa\leq \kappa_1 \).

Demostración:
Si \( \kappa=T(\mu) \), entonces \( \mu\leq \kappa_0 \) (pues \( \kappa_0 \) es el mayor cardinal), luego \( \kappa=T(\mu)\leq T(\kappa_0)=\kappa_1 \).

Recíprocamente, si \( \kappa\leq \kappa_1=|\mathcal P_1V| \), tomamos un conjunto \( X \) tal que \( \kappa=|X| \) y entonces existe \( f: X\longrightarrow \mathcal P_1V \) inyectiva. Como \( |X|=|f[X]| \), podemos suponer que \( X\subset \mathcal P_1V \). Así, podemos definir \( Y=\{x\mid \{x\}\in X\} \), de modo que \( X=\mathcal P_1Y \), luego \( \kappa = |X|=T(|Y|) \) y basta tomar \( \mu=|Y| \).
[cerrar]

Observemos que si \( \kappa=T(\mu) \), entonces \( \mu \) es único, por las propiedades de \( T \), luego, para cada cardinal \( \kappa\leq \kappa_1 \) podemos definir

\( T^{-1}(\kappa)\equiv \mu\mid T(\mu)=\kappa \),

que es un término estratificado cuyo tipo es una unidad inferior al de \( \kappa \).

Terminamos el estudio del operador \( T \) con su acción sobre los cardinales finitos:

Teorema: Un conjunto \( X \) es finito si y sólo si \( \mathcal P_1X \) es finito.

Demostración
Ya hemos probado que si \( X \) es finito entonces \( \mathcal PX \) es finito y, como \( \mathcal P_1X\subset \mathcal PX \), concluimos que \( \mathcal P_1X \) también es finito.

Para probar el recíproco consideramos el conjunto

\( A=\{n\mid n\in \mathbb{N}\land \forall A(|\mathcal P_1A|=n\rightarrow A\text{ es finito})\} \),

que está bien definido y basta probar por inducción que \( A=\mathbb{N} \).

Se cumple que \( 0\in A \), pues si \( |\mathcal P_1A|=0 \) entonces \( \mathcal P_1A=\emptyset \), lo cual sólo es posible si \( A=\emptyset \), luego \( A \) es finito.

Si \( n\in A \) y \( |\mathcal P_1A|=n+1 \), entonces \( A\neq\emptyset \), luego podemos tomar \( u\in A \) y llamar \( B=A\setminus\{u\} \), con lo que \( A=B\cup\{u\} \) y \( \mathcal P_1A = \mathcal P_1B\cup\{\{u\}\} \) y las uniones son disjuntas, luego \( |\mathcal P_1B|=n \), luego por hipótesis de inducción \( B \) es finito y, por consiguiente, \( A \) también lo es (por ser unión de dos conjuntos finitos).
[cerrar]

Esto se traduce en que \( \kappa \) es un cardinal finito si y sólo si lo es \( T(\kappa) \). En particular, como \( \kappa_0 \) es infinito, lo mismo vale para todos los cardinales \( \kappa_0,\kappa_1,\kappa_2,\ldots \)

A su vez, esto implica que si \( n\in \mathbb{N} \) entonces \( n\leq\kappa_1 \), luego está definido \( T^{-1}(n)\in \mathbb{N} \).

Nota: Observemos que es fácil probar, por ejemplo, que \( T(3)=3 \). En efecto, es fácil ver que un conjunto de cardinal \( 3 \) es de la forma \( X=\{a,b,c\} \), donde los tres elementos son distintos dos a dos. Entonces podemos definir

\( f=\{(a,\{a\}),(b,\{b\}),(c,\{c\})\} \),

que es unconjunto bien definido. No es un término estratificado, pero no hace falta. Para todo objeto \( a \) existe el conjunto \( \{a\} \). Para todo par de conjuntos existe su par ordenado, luego existe \( (a,\{a\}) \) (aunque sea un término no estratificado). Por el mismo motivo anterior, existe \( \{(a,\{a\})\} \) y \( f \) está definido como la unión de tres conjuntos de esta forma. Claramente, \( f: X\longrightarrow \mathcal P_1X \) biyectiva.

Esto prueba que \( 3=|X|=|\mathcal P_1X|=T(3) \), e igualmente podemos probar \( T(0)=0, T(1)=1, T(2)=2, T(3)=3, T(4)=4, \) etc. Sin embargo, no podemos probar que \( \forall n\in \mathbb{N}\ T(n)=n \).



La "conjuración" de la antinomia de Burali-Forti se interpreta también en términos de un operador \( T \) análogo al que acabamos de exponer, pero definido sobre ordinales. En efecto:

Definición: Si \( (A,\leq) \) es un conjunto bien ordenado, definimos \( (A,\leq)_1\equiv (\mathcal P_1A, \leq_1) \), donde \( \leq_1\equiv\{(\{a\},\{b\})\mid a\in A\land b\in A\land a\leq b\} \).

Equivalentemente, \( \leq_1 \) es la relación en \( \mathcal P_1A \) dada por

\( \{a\}\leq_1\{b\}\leftrightarrow a\leq b \).

El término \( (A,\leq)_1 \) está estratificado y tiene tipo una unidad mayor que el de \( (A,\leq) \). Es fácil ver que \( (A,\leq)_1 \) es un conjunto bien ordenado, así como que dos conjuntos bien ordenados  \( (A,\leq_A) \) y \( (B,\leq_B) \) son semejantes si y sólo si lo son \( (A,\leq_A)_1 \) y \( (B,\leq_B)_1 \).

Esto nos permite definir, para cada ordinal \( \alpha \):

\( T(\alpha)\equiv \beta\mid\exists A\leq((A,\leq)\text{ es un c.b.o.}\land \alpha=\text{ord}(A,\leq)\land \beta=\text{ord}((A,\leq)_1)) \).

Así pues, si \( (A,\leq) \) es un conjunto bien ordenado, se cumple que \( \text{ord}((A,\leq)_1)=T(\text{ord}(A,\leq)) \).

El teorema siguiente se demuestra de forma análoga al correspondiente a cardinales:

Teorema Se cumple:

1) \( \forall \alpha\beta\in \text{Ord}(\alpha\leq \beta\leftrightarrow T(\alpha)\leq T(\beta)) \)

2) \( \forall \alpha\beta\in \text{Ord}(\alpha= \beta\leftrightarrow T(\alpha)=T(\beta)) \)

3) \( \forall \alpha\beta\in \text{Ord}\ T(\alpha+\beta)=T(\alpha)+T(\beta) \)

4) \( \forall \alpha\beta\in \text{Ord}\ T(\alpha\beta)=T(\alpha)T(\beta) \)

No necesitaremos los resultados sobre la suma y el producto de ordinales.

Teniendo en cuenta que el conjunto base de \( (A,\leq)_1 \) es \( \mathcal P_1A \), cuyo cardinal es \( T(|A|) \), la relación siguiente resulta inmediata:

\( \forall \alpha\in \text{Ord}\ \text{card}(T(\alpha))=T(\text{card}(\alpha)) \).

En estos términos podemos corregir el falso teorema que daba lugar a la antinomia de Burali-Forti:

Teorema: Si \( \alpha \) es un ordinal, entonces \( \text{ord}(\text{Ord}_\alpha^<,\leq)=T^2(\alpha) \).

Demostración:
Sea \( \alpha=\text{ord}(A,\leq) \). Entonces \( T^2(\alpha)=T(T(\alpha)) \) es el ordinal de \( ((A,\leq)_1)_1 \), que claramente es el conjunto \( \mathcal P_1\mathcal P_1A=\{\{\{x\}\}\mid x\in A\} \) con el orden dado por

\( \{\{a\}\}\leq_2\{\{b\}\}\leftrightarrow a\leq b \).

Definimos \( f:\mathcal P_1\mathcal P_1A\longrightarrow \text{Ord}_\alpha^< \) mediante \( f(\{\{a\}\})=\text{ord}(A_a^<,\leq) \).

Los mismos argumentos empleados en el "falso teorema" que originaba la antinomia de Burali-Forti prueban ahora que \( f: ((A,\leq)_1)_1\longrightarrow (\text{Ord}_\alpha^<,\leq) \) es una semejanza, luego \( \text{ord}(\text{Ord}_\alpha^<,\leq)=\text{ord}(((A,\leq)_1)_1)=T^2(\alpha) \).
[cerrar]

De aquí no extraemos la conclusión (contradictoria) de que todo ordinal es menor que \( \Omega \), sino que, para todo ordinal \( \alpha \), se cumple que \( T^2(\alpha)<\Omega \). En particular, \( T^2(\Omega)<\Omega \).

Más aún, se cumple que \( T(\Omega)<\Omega \), pues si fuera \( T(\Omega)\leq \Omega \), aplicando \( T \) obtendríamos que \( T^2(\Omega)\leq T(\Omega)\leq \Omega \), contradicción.

Aplicando \( T \) sucesivamente obtenemos una sucesión decreciente de ordinales:

\( \cdots <T^3(\Omega)<T^2(\Omega)<T(\Omega)<\Omega \),

la cual, pese a las apariencias, no contradice al hecho de que el conjunto \( \text{Ord} \) está bien ordenado.

Hay una caracterización importante del ordinal \( T(\alpha) \):

Si \( (A,\leq) \) es un conjunto bien ordenado, podemos considerar el conjunto de sus secciones iniciales:

\( S(A,\leq)\equiv \{X\mid \exists a\in A\ X=A_a^<\} \),

y es fácil ver que \( (S(A,\leq), \subset) \) es un conjunto bien ordenado. Podemos definir una semejanza \( f: (A,\leq)_1\longrightarrow (S(A,\leq),\subset) \) mediante \( f(\{a\})=A_a^< \).

Aquí es crucial que \( A_a^< \) tiene tipo una unidad mayor que el tipo de \( a \), por lo que no podemos definir \( f(a)=A_a^< \), sino que hemos de trabajar con \( \mathcal P_1A \) en lugar de con \( A \).

Así pues, si \( \alpha=\text{ord}(A,\leq) \), concluimos que \( T(\alpha)=\text{ord}(S(A,\leq),\subset) \).

Con palabras, \( T(\alpha) \) es el tipo de orden del conjunto de las secciones iniciales de \( (A,\leq) \) ordenadas por la inclusión.

Teorema: Un ordinal \( \alpha \) es de la forma \( \alpha=T(\beta) \), para otro ordinal \( \beta \) si y sólo si \( \text{card}(\alpha)\leq \kappa_1 \).

Demostración:
Si \( \alpha=t(\beta) \), entonces \( \text{card}(\alpha)=T(\text{card}(\beta))\leq T(\kappa_0)=\kappa_1 \).

Recíprocamente, si \( \text{card}(\alpha)\leq \kappa_1 \), podemos considerar un conjunto bien ordenado tal que \( \alpha=\text{ord}(A,\leq) \), y entonces \( |A|\leq \kappa_1 = |\mathcal P_1V| \), luego existe \( f: A\longrightarrow \mathcal P_1V \) inyectiva. Si llamamos \( A'=f[A] \) y definimos la relación de orden

\( \leq'\equiv \{(x,y)\mid \exists uv\in A(x=f(u)\land y=f(v)\land u\leq v)\} \),

es claro que \( f:(A,\leq)\lognrightarrow (A',\leq') \) semejanza, luego no perdemos generalidad si suponemos que \( A\subset \mathcal P_1V \). Entonces definimos \( B=\{x\mid \{x\}\in A\} \), de modo que \( A=\mathcal P_1B \), y definimos \( \leq_B=\{(u,v)\mid u\in B\land v\in B\land \{u\}\leq\{v\}\} \), de modo que \( (B,\leq_B) \) es un conjunto bien ordenado y \( (A,\leq)=(B,\leq_B)_1 \), luego \( \alpha=\text{ord}((B,\leq_B)_1)=T(\beta) \), donde \( \beta=\text{ord}(B,\leq_B) \).
[cerrar]

Así, para todo ordinal \( \alpha \) que cumpla la condición del teorema, está definido

\( T^{-1}(\alpha)\equiv \beta\mid (\beta\in \text{Ord}\land T(\beta)=\alpha) \).

Para terminar contamos los ordinales. Notemos que \( \kappa_2<\kappa_0 \), y que (suponiendo AE) todo cardinal \( \kappa \) distinto de \( \kappa_0 \) tiene un inmediato sucesor al que representamos por \( \kappa^+ \), luego en particular está definido \( \kappa_2^+ \).

Teorema (AE): \( |\text{Ord}|=\kappa_2^+ \).

Demostración:
Hemos visto que, para todo ordinal \( \alpha \) se cumple que \( \text{ord}(\text{Ord}_\alpha^<,\leq)=T^2(\alpha) \), luego aplicando la función \( \text{card} \) resulta que \( T^2(\text{card}(\alpha))=|\text{Ord}_\alpha^<|\leq |\text{Ord}| \).

En particular, \( |\text{Ord}_\alpha^<|=T^2(\text{card}(\alpha))\leq T^2(\kappa_0)=\kappa_2 \).

Es decir, todas las secciones iniciales de \( \text{Ord} \) tenen cardinal a lo sumo \( \kappa_2 \).

Por otra parte, consideremos un conjunto bien ordenado \( (A,\leq) \) sobre un conjunto de cardinal \( \kappa_2^+ \). Entonces se tiene que cumplir una de las tres posibilidades siguientes:

1) \( (A,\leq) \) es semejante a \( (\text{Ord},\leq) \).

2) \( (A,\leq) \) es semejante a un segmento inicial de \( (\text{Ord},\leq) \).

3) \( (\text{Ord},\leq) \) es semejante a un segmento inicial de \( (A,\leq) \).

Ahora bien, hemos descartado la posibilidad 2), ya que hemos visto que los segmentos iniciales de \( \text{Ord} \) no pueden tener cardinal \( \kappa_2^+ \), luego, o bien se da el caso 1) y \( |\text{Ord}|=\kappa_2^+ \), o bien se da el caso 3) y \( |\text{Ord}|\leq \kappa_2^+ \). En ambos casos \( |\text{Ord}|\leq \kappa_2^+ \).

Por otro lado, si tomamos un ordinal \( \alpha \) tal que \( \text{card}(\alpha)=\kappa_0 \), hemos visto que \( |\text{Ord}|\geq T^2(\text{card}(\alpha))=\kappa_2 \). En definitiva, sólo hay dos posibilidades: \( |\text{Ord}|=\kappa_2 \) o \( |\text{Ord}|=\kappa_2^+ \). Falta, pues, descartar la primera.

Si \( |\text{Ord}|=\kappa_2 \), entonces \( \text{card}(\Omega)=\kappa_2 \), luego está definido \( T^{-1}(\Omega) \), que es un ordinal tal que \( T(\text{card}(T^{-1}(\Omega))=\text{card}(\Omega)\leq T(\kappa_1) \), luego \( \text{card}(T^{-1}(\Omega)\leq \kappa_1 \), luego está definido \( \alpha=T^{-2}(\Omega) \), y así \( \Omega=T^2(\alpha)=\text{ord}(\text{Ord}_\alpha^<,\leq) \), pero esto es absurdo, pues significa que el segmento inicial \( \text{Ord}_\alpha^< \) es semejante a \( \text{Ord} \), y un conjunto bien ordenado nunca es semejante a ninguno de sus segmentos.
[cerrar]

En los próximos mensajes construiremos la función \( \aleph \), que recorre los cardinales infinitos y definiremos la exponenciación de cardinales.
Título: La sucesión de los álefs
Publicado por: Carlos Ivorra en 18 Enero, 2012, 11:22 am
El conjunto \( K_i \) de todos los cardinales infinitos es un conjunto bien ordenado con un máximo elemento igual a \( \kappa_0 \). Por lo tanto, su ordinal será de la forma \( \alpha_0+1 \). Explícitamente:

\( \alpha_0\equiv \text{ord}(\{\kappa\mid \kappa\in K\land \aleph_0\leq \kappa<\kappa_0\},\leq) \).

De este modo, cada ordinal \( \alpha\leq \alpha_0 \) es el ordinal de un segmento inicial de \( K_i \) determinado por un único cardinal infinito \( \kappa \). Esto nos permite definir:

\( \aleph_\alpha\equiv \kappa\mid \text{ord}(\{\mu\mid \mu\in K\land |\mathbb{N}|\leq \mu<\kappa\},\leq)=\alpha \).

En palabras: \( \aleph_\alpha \) es el cardinal infinito que cumple que el conjunto de cardinales infinitos menores que él tiene ordinal \( \alpha \).

Observemos que \( \aleph_0 \) en este sentido es el cardinal infinito tal que el conjunto de cardinales infinitos menores es vacío, es decir, es el menor cardinal infinito, luego es el mismo cardinal al que ya habíamos llamado \( \aleph_0 \).

Notemos que, por definición de \( \alpha_0 \), se cumple que \( \aleph_{\alpha_0}=\kappa_0 \). Por otra parte, es inmediato que si \( \alpha<\beta\leq \alpha_0 \), entonces \( \aleph_\alpha<\aleph_\beta \), así como que \( \aleph_{\alpha+1} \) (si está definido) es el menor cardinal mayor que \( \aleph_\alpha \), es decir, \( \aleph_{\alpha+1}=\aleph_\alpha^+ \).

También es muy importante observar que el término \( \aleph_\alpha \) está estratificado con tipo dos unidades inferior al de \( \alpha \).

Para cada \( \alpha\leq \alpha_0 \) llamaremos \( \omega_\alpha=\text{In}(\aleph_\alpha) \), es decir, el menor ordinal cuyo cardinal asociado es \( \aleph_\alpha \). En particular \( \omega_0=\omega \).

Teorema: \( T(\alpha_0)<\alpha_0 \) y para todo ordinal \( \alpha\leq \alpha_0 \) se cumple que \( T(\aleph_\alpha)=\aleph_{T(\alpha)} \).

Demostración
Observemos que la definición del conjunto siguiente está estratificada:

\( X=\{\alpha\mid \alpha\in \text{Ord}\land\alpha\leq\alpha_0\land (\alpha_0<T(\alpha)\lor(T(\alpha)\leq\alpha_0\land T(\aleph_\alpha)\neq \aleph_{T(\alpha)}))\} \).

Vamos a probar que \( X=\emptyset \), lo cual equivale a que, para todo ordinal \( \alpha\leq\alpha_0 \), se cumple que \( T(\alpha_0)\leq\alpha_0 \) y \( T(\aleph_\alpha)=\aleph_{T(\alpha)} \).

En efecto, si el conjunto no es vacío, tiene un mínimo elemento \( \alpha \), de modo que para todo \( \delta<\alpha \) se cumple que \( T(\delta)\leq\alpha_0 \) y \( T(\aleph_\delta)=\aleph_{T(\delta)} \). Consideramos el conjunto

\( Y=\{x\mid \exists \delta(\delta\in \text{ord}\land \delta<\alpha\land x=\aleph_{T(\delta)})\} \).

Observemos que se trata del conjunto de todos los cardinales infinitos menores que \( T(\aleph_\alpha) \), pues si \( \aleph_0\leq \mu<T(\aleph_\alpha) \), entonces \( \aleph_0\leq T^{-1}(\mu)<\aleph_\alpha \) (aquí hemos usado que, como \( \mu \) es infinito, \( T^{-1}(\mu) \) también lo es), luego existe un \( \delta<\alpha \) tal que \( T^{-1}(\mu)=\aleph_\delta \), luego \( \mu=T(\aleph_\delta)=\aleph_{T(\delta)}\in Y \).

Por otra parte, si \( \text{ord}(A,\leq)=\alpha \), la aplicación \( \mathcal P_1A\longrightarrow Y \) dada por \( \{a\}\mapsto \aleph_{T(\text{ord}(A_a^<,\leq))} \) está bien definida y es una semejanza, luego \( \text{ord}(Y,\leq)=\text{ord}(A,\leq)_1=T(\alpha) \).

Con esto hemos probado que el conjunto de todos los cardinales infinitos menores que \( T(\aleph_\alpha) \) tiene ordinal \( T(\alpha) \), y esto significa que existe \( \aleph_{T(\alpha)}=T(\aleph_\alpha) \).

En particular \( \aleph_{T(\alpha_0)}=T(\aleph_{\alpha_0})=T(\kappa_0)<\kappa_0=\aleph_{\alpha_0} \), luego \( T(\alpha_0)<\alpha_0 \).
[cerrar]

Puesto que \( T(0)=0,\ T(1)=1,\ T(2)=2,\ldots \) podemos asegurar que \( \alpha_0 \) no es \( 0 \), ni \( 1 \), ni \( 2 \), etc., por lo que existen los cardinales \( \aleph_0<\aleph_1<\aleph_2<\cdots \), pero esto no significa que podamos demostrar que \( \alpha_0\geq \omega \). Es consistente que \( \alpha_0 \) sea un ordinal finito y, por consiguiente, \( \aleph_\omega \) no esté definido, pero dicho ordinal finito no puede ser \( 0 \), ni \( 1 \), ni \( 2 \), ni ningún número finito que podamos escribir explícitamente. (Así es NFA.)

En el próximo mensaje construiremos la exponenciación de cardinales.
Título: La exponenciación cardinal en NFA
Publicado por: Carlos Ivorra en 21 Enero, 2012, 06:04 pm
Para introducir la exponenciación ordinal definimos, para cada par de conjuntos \( A \) y \( B \) el conjunto

\( A^B=\{f\mid f: B\longrightarrow A\} \).

Una comprobación rutinaria muestra que si \( |A|=|A'| \) y \( |B|=|B'| \) entonces \( |A^B| = |A'^{B'}| \). Esto da pie a definir la exponenciación de cardinales como en ZFC, es decir, de modo que \( |A^B| = |A|^{|B|} \), pero hay un inconveniente técnico que hace desaconsejable esta definición. Observemos que \( A^B \) está estratificado con tipo una unidad superior a los tipos de \( A \) y \( B \), por lo que la definición de exponenciación de ZFC haría que el tipo de \( \kappa^\mu \) fuera una unidad superior a los tipos de \( \kappa \) y \( \mu \), y así no podríamos considerar a la exponenciación como una aplicación \( K\times K\longrightarrow K \). En su lugar, es más conveniente adoptar la definición siguiente:

Definición:  \( (\ )^{(\ )}\equiv \{x\mid \exists AB(|A^B|\leq\kappa_1\land x=((|A|,|B|), T^{-1}(|A|^{|B|})))\} \)

En palabras: dados dos cardinales \( \kappa \) y \( \mu \), consideramos conjuntos tales que \( |A|=\kappa \) y \( |B|=\mu \). La condición \( |A^B|\leq \kappa_1 \) equivale a que esté definido el cardinal \( T^{-1}(|A^B|) \) y, en tal caso, definimos la exponencial \( \kappa^\mu = T^{-1}(|A^B|) \). En caso contrario la exponenciación no está definida.

Así, como \( T^{-1} \) rebaja el tipo una unidad, tenemos que el tipo de \( T^{-1}(|A^B|) \) es sólo una unidad superior a los tipos de \( A \) y \( B \), es decir, el mismo que el de \( (|A|, |B|) \), por lo que la aplicación \( (|A|,|B|)\mapsto T^{-1}(|A^B|) \) está bien definida, y es la que hemos tomado como exponenciación cardinal.

En resumen, la relación fundamental es que

\( |A^B| = T^{-1}(|A^B|) \),

teniendo en cuenta que la exponencial sólo está definida cuando \( |A^B|\leq \kappa_1 \).

Las propiedades siguientes se demuestran sin dificultad (entendiendo que un miembro está definido si y sólo si lo está el otro):

\( \kappa^{\mu+\nu}=\kappa^\mu\kappa^\nu,\qquad (\kappa\mu)^\nu=\kappa^\nu\mu^\nu,\qquad (\kappa^\mu)^\nu=\kappa^{\mu\nu} \).

Por ejemplo, para probar la primera observamos que existe una biyección natural \( A^{B\cup C}\longrightarrow A^B\times A^C \) (donde \( B\cap C=\emptyset \)), con lo que \( |A^{B\cup C}|=|A^B||A^C| \). De aquí se sigue que el miembro izquierdo es \( \leq \kappa_1 \) si y sólo si los dos factores de la derecha lo son, y en tal caso podemos aplicar \( T^{-1} \) (que conmuta con el producto de cardinales) y concluir que \( T^{-1}(|A^{B\cup C}|)=T^{-1}(|A^B|)T^{-1}(|A^C|) \), luego \( |A|^{|B\cup C|}=|A|^{|B|}|A|^{|C|} \).

Notemos que la presencia de \( T^{-1} \) es necesaria para probar algo tan "obvio" como que \( \kappa^1=\kappa \). En efecto, si \( |A|=\kappa \) y \( |B|=1 \), entonces \( B=\{u\} \), para cierto objeto \( u \), pero no podemos definir una biyección \( A^B\longrightarrow A \). Lo que podemos definir de forma natural es una biyección \( A^B\longrightarrow \mathcal P_1A \) mediante \( f\mapsto \{f(u)\} \), pues \( f(u) \) tiene tipo una unidad menor que \( f \), luego necesitamos pasar a \( \{f(u)\} \) para que la asignación conserve el tipo, condición necesaria para que la definición de la biyección esté estratificada y, por consiguiente, su existencia pueda ser justificada. Así concluimos que \( |A^B| = |\mathcal P_1A|=T(|A|) \), luego \( \kappa =|A|=T^{-1}(|A^B|)=|A|^{|B|}=\kappa^1  \).

Así pues, si no hubiéramos incluido el término \( T^{-1} \) en la definición no podríamos haber justificado esta propiedad, ni tampoco que \( \kappa^2 = \kappa\kappa \), pues es consecuencia de la anterior: \( \kappa^2 = \kappa^{1+1}=\kappa^1\kappa^1 = \kappa\kappa \).

Veamos ahora que la exponenciación cardinal nos permite calcular el cardinal de cualquier conjunto de la forma \( A^B \), aunque dicho cardinal sea mayor que \( \kappa_1 \):

Teorema: Para todo par de conjuntos \( A \) y \( B \) se cumple que \( |A^B|=T(|A|)^{T(|B|)} \).

Demostración
Consideramos la biyección \( F:\mathcal P_1A^{\mathcal P_1B}\longrightarrow \mathcal P_1(A^B) \) dada por

\(
f\mapsto F(f)=x\mid \exists g(x=\{g\}\land g: B\longrightarrow A\land \forall b\in B\ f(\{b\})=\{g(b)\}).
 \)

Es claro que la definición de \( F \) está estratificada y que es ciertamente una biyección, por lo que \( |\mathcal P_1A^{\mathcal P_1B}|=T(|A^B|)\leq T(\kappa_0)=\kappa_1 \). Esto implica que está definida la potencia \( |\mathcal P_1A|^{|\mathcal P_1B|}=|A^B| \), es decir, \( |A^B| = T(|A|)^{T(|B|)} \).
[cerrar]

La condición para que esté definido \( \kappa^\mu \) no es muy práctica, pues casi supone conocer cuánto da \( \kappa^\mu \) para determinar si existe. Sin embargo, el teorema anterior implica una condición suficiente mucho más natural:

Teorema: Si \( \kappa,\mu\leq \kappa_1 \), entonces está definido \( \kappa^\mu \) y además se cumple que \( T(\kappa^\mu)=T(\kappa)^{T(\mu)} \).

Demostración
Sea \( |A|=T^{-1}(\kappa) \) y \( |B|=T^{-1}(\mu) \), de modo que \( \kappa =T(|A|) \), \( \mu=T(|B|) \). Ahora basta aplicar el teorema anterior a los conjuntos \( A \) y \( B \), según el cual está definida la potencia \( \kappa^\mu= T(|A|)^{T(|B|)}=|A^B| \).

Por otra parte, si aplicamos el teorema anterior a conjuntos que cumplan \( |A|=\kappa \) y \( |B|=\mu \), obtenemos que \( T(\kappa)^{T(\mu)}=T(|A|)^{T(|B|)}=|A^B|=T(\kappa^\mu) \).
[cerrar]

La relación con los conjuntos de partes es la siguiente:

Teorema: Para todo conjunto \( A \) se cumple que \( |\mathcal PA| = 2^{T(|A|)} \).

Demostración
Basta observar que la biyección natural \( \{0,1\}^A\longrightarrow \mathcal PA \) está bien definida, lo que nos da que \( |\mathcal PA|=T(2)^{T(|A|)}=2^{T(|A|)} \).
[cerrar]

Ejemplo: Se cumple que \( |\mathbb{N}|=2^{\aleph_0} \).

En efecto, \( |\mathcal P\mathbb{N}|=2^{T(\aleph_0)}=2^{\aleph_{T(0)}}=2^{\aleph_0} \).

El teorema de Cantor afirma que \( |\mathcal PA|>T(|A|) \), luego si \( 2^\kappa \) está definido (en particular para \( \kappa\leq \kappa_1) \) tenemos la desigualdad

\( \kappa<2^\kappa \)

(pues \( \kappa=|A|<T^{-1}(|\mathcal PA|)=2^\kappa \)). Así pues, la versión del teorema de Cantor en ZFC en términos de la exponenciación de cardinales es válida igualmente en NFA (aunque limitada a cardinales \( \leq \kappa_1 \)).

En particular \( \kappa_1<2^{\kappa_1}=T(2)^{T(|V|)}=|\mathcal PV|\leq \kappa_0 \).
Título: Re: NFA
Publicado por: argentinator en 21 Enero, 2012, 11:28 pm
Citar
¿Qué es NFA? Es una teoría de conjuntos. Su historia se remonta hasta 1937, cuando Quine publicó su libro "New Foundations for Mathematical Logic", en el que presentaba una teoría de conjuntos (hoy conocida como NF, por "New Foundations"

Dado que el inventor de la teoría es de habla inglesa, y su libro se publicó en inglés, pienso que hay que respetar esto y conservar la abreviatura original NFU en lugar de castellanizarla con NFA.
Conviene (1) mantener una notación uniforme en todos los idiomas, y (2) a la hora de elegir idioma pienso que hay que elegir el que usó el creador de la teoría o lo que se ha hecho estándar.

Se complica cuando el autor es chino, pero bueno, ya se verá.

El otro autor que contribuyó al parecer en forma importante a la teoría es Jensen, de habla inglesa:

Citar
en 1969, cuando Jensen presentó una corrección de NF [...]
 su nombre en inglés, NFU, viene de Urelement que es el nombre que dan a los átomos [...]

Pienso que para tener el derecho de usar siglas en castellano, necesitamos tener una teoría desarrollada por un matemático hispanoparlante.

Y luego, claro, que esas siglas o nomenclatura se usen en todo el planeta uniformemente.

En realidad la falta de uniformidad es algo que me pone los pelos de punta, porque si hay un lugar donde debe haber acuerdo y falta de ambigüedad es en la matemática.

P.D.: Obviamente que este asunto es el menos importante de todos los que se puedan discutir.

______________

Citar
Axioma de los átomos: \( \forall xy(x\in y\rightarrow \text{cto}\,y) \)

En palabras: Si un objeto tiene elementos, entonces es un conjunto o, equivalentemente, los átomos son objetos sin elementos.

Tengo dudas sobre cómo está enunciado esto.
Yo diria que: "un objeto es un conjunto si EXISTE otro objeto que pertenece a él".
Así, escribiría: \( \forall y(\exists{x}(x\in y)\qquad \rightarrow \text{cto}\,y) \).

Por contrarrecíproco sale que un no-conjunto (o átomo) no tiene elementos.

¿Y el \( \emptyset \)? ¿Conjunto o átomo?

__________________

Citar
A decir verdad, no vamos a tomar el axioma de los átomos como axioma de NFA por una razón muy simple: aunque lo hiciéramos, jamás tendríamos ocasión de usarlo.

Entonces no entiendo dónde se produce la distinción de Jensen entre una teoría que sí tiene átomos.

El uso del operador "cto" en las fórmulas y axiomas debiera ser suficiente, pero deja la puerta abierta a una teoría muy general, que no se sabe si tiene o no tiene algún átomo.

_____________

Citar
En palabras: Dos pares ordenados son iguales si y sólo si tienen la misma primera componente y la misma segunda componente.

De este modo, por el mero hecho de introducir el nuevo signo, estamos diciendo que, para cada par de objetos \( x \)  e \( y \), existe un tercer objeto (no especificamos si es un átomo o un conjunto)

Tomo nota de este hecho de que los pares ordenados no se sabe si son conjuntos o no.

También se pone en evidencia que siempre se necesita en matemática usar pares o n-uplas ordenadas, aún en las fases más elementales de cualquier teoría, pues se necesitan relaciones y funciones.

___________

Citar
Por consiguiente, la paradoja de Russell no puede demostrarse en NFA, sus axiomas no afirman que exista el conjunto \( \{x\mid x\notin x\} \) , ya que la fórmula que (supuestamente) lo define no está estratificada.

Es interesante notar que, mientras en ZFC se prohíben desde los axiomas conjuntos que den lugar a paradojas,
en NFU (o sea NFA, o sea NFU, o sea...  :banghead: ) en cambio la prohibición se hace directamente desde el lenguaje.

No sé si a alguien se le había ocurrido antes esto de "meterse con el lenguaje".

A simple vista parece también que la estratificación impide conjuntos que cumplan algo como X = {X}, o sea, se impiden aquellos casos patológicos que en ZFC se suprimen por medio del axioma de regularidad.
¿Estoy acertado en esto? Pues a mí me da que si X tiene cierto tipo \( m \), necesariamente {X} tiene un tipo mayor \( m' \), pero entonces por el primer axioma y por ser X = {X}, se obtiene que X tiene tipo \( m' \), pero X no puede tener asociados dos tipos distintos \( m,m' \).

Otra duda técnica: ¿queda claro de entrada este hecho de que un cierto objeto \( x \) no tiene permitido tener asociados dos tipos distintos en una misma fórmula?

_______________

Otra cosa que llama la atención enseguida es que los pares ordenados tienen el mismo tipo que sus componentes.
Eso es algo inesperado para mí, porque uno está acostumbrado a poner en ZFC los pares ordenados como conjuntos que contienen de alguna forma algo enrevesada a sus componentes (\( (x,y)=\{\{x\},\{x,y\}\} \) por ejemplo). El objeto \( \{\{x\},\{x,y\}\} \) tiene tipo 2 veces mayor que el de \( x,y \).

¿Implica necesariamente esto que los pares ordenados en NFU no son conjuntos?

______________

En cuanto a los ordinales de Von Newmann ya lo hemos comentado al principio, que no andan muy bien, pues no ellos necesitarían fórmulas no estratificadas como \( (x\in y)\wedge (x\subset y) \).

Citar
la definición de ordinal (de ZF o MK) no está estratificada, la teoría de ordinales de von Neumann no funciona bien en NFA, pero en su lugar tenemos una definición alternativa de ordinal que funciona perfectamente y es mucho más fiel a la idea original: un ordinal se define como una clase de equivalencia de conjuntos bien ordenados respecto a la relación de isomorfismo. Eso en ZFC o en MK no funciona bien porque se trataría de una relación de equivalencia sobre una clase propia, y cada clase de equivalencia sería una clase propia, pero en NFA el conjunto de todos los conjuntos bien ordenados

Por otra parte Sailor ha puntualizado lo siguiente:

Citar
Una  puntualización (no necesariamente una corrección)  a raiz de un comentario de Argentinator:

 El axioma de estratificación dice que las formulas estratificadas necesariamente definen conjuntos, pero eso no implica que algunos objetos sólo definibles mediante formulas no estratificadas no puedan ser conjuntos. No es demasiado claro Holmes en su libro, y es aquí donde mi puntualización ya se transforma en  una duda: Cuando se refiere a los ordinales de Von Neumann Holmes afirma que podemos considerar que existen o que no existen. Creo que lo que trata de decir (evitandose usar la palabra "modelo") es que NFU tiene modelos con  ordinales de Von Neumann, y otros sin ellos;y  que por tanto es conveniente definir los ordinales de otra forma, que es la que comenta Ivorra, para asegurarnos que existen en la teoría (o sea, que existen en todos los modelos de la teoría).

La verdad es que en ZFC uno "echa de menos" el enfoque de los cardinales a través de "clases de equivalencia".

En NGB o en MK uno puede definir los cardinales como clases de equivalencia, pero no los puede usar, porque no puede formar "conjuntos cuyos elementos son números cardinales".

Más aún, Carlos ha respondido en parte lo siguiente:

Citar
El concepto de ordinal de von Neumann se puede definir en toda teoría de conjuntos entre cuyos teoremas se encuentren el axioma de extensionalidad, la existencia de la unión de dos conjuntos, del complemento de dos conjuntos, del par desordenado de dos conjuntos y del conjunto vacío. No hace falta nada más, en particular no hace falta ningún axioma de formación de conjuntos o subconjuntos a partir de fórmulas.

Todos estos resultados son teoremas de NFA, luego en NFA puedes definir el concepto de ordinal (de von Neumann) e incluso el de número natural (de von Neumann, de modo que , etc.)

Ahora bien, las fórmulas "x es un ordinal" o "x es un número natural" (en este sentido de von Neumann) no están estratificadas y, por ello dan muy poco juego. En NFA no es posible demostrar que exista el conjunto de los números naturales en este sentido (pero sí que se pueden definir de otro modo, estratificado, y todo va bien).

Esta respuesta contiene un hecho interesante.
Es sabido que en el tema de la incompletitud de Godel, la incompletitud surge al pretender utilizar una fórmula que diga algo como "x es un número natural".
Según lo dicho por Carlos, esta frase no existe para ordinales de Von Neumann en NFU.
Me pregunto si hay alguna relación entre estos temas de la estratificación y la incompletitud de Godel.
Mi olfato dice que la incompletitud de Godel sigue firme en NFU, pero bueno, no sé.

En realidad los números naturales y los infinitos siempre traen incomodidades.
Son lo más básico de la matemática, y sin embargo nunca se encuentra una teoría que sea del todo "cómoda" o que exprese o refleje todo lo que hace falta o se desea.
Como yo lo veo, esto es indicio de un problema más profundo, y es que nadie entiende qué son los números naturales ni mucho menos los transfinitos,
por lo cual no puedo admitir que se los utilice despreocupadamente al construir una teoría axiomática.

Este asunto de la estratificación es una idea interesante, pero a mí lo que no me gusta es el hecho de asignarle un número natural o entero a una expresión.
Podría admitirla sin problemas, en cambio, si la cantidad de tipos aceptadas fuera finita, y no todo el rango de los enteros.

Según ví en Wikipedia, NF es equivalente a \( \textsf{NF}_4 \), en que se admiten 4 grados de estratificación como máximo. (Si es que he entendido bien el asunto  :banghead: ).

¿Se puede convertir NFU en algo como un \( \textsf{NFU}_4 \), digamos?

____________________

En cuanto a lo que pregunté arriba sobre el conjunto vacío, creo que está muy bien explicado acá:

Citar
Alguien podría pensar que podríamos haber incorporado el axioma de los átomos a la teoría a la vez que eliminábamos el signo primitivo cto definiendo un átomo como un objeto sin elementos y un conjunto como un objeto con elementos. Eso no es conveniente porque nos interesa que la teoría tenga un conjunto vacío, es decir, un objeto que no tiene elementos y, a pesar de ello, no es un átomo, sino un conjunto. El axioma de extensionalidad implica que el conjunto vacío es el único conjunto sin elementos, pues dos conjuntos vacíos serían dos conjuntos con los mismos elementos (ninguno) y no puede haber dos conjuntos distintos con los mismos elementos. En cambio, el axioma de extensionalidad no afecta a los átomos, por lo que nada impide que haya muchos átomos distintos y que ninguno tenga elementos.

_________________

Lo siguiente no lo tengo claro:

Citar
Cabe destacar algunas relaciones obvias que pueden "chirriar" a una mente clásica: \( V\in C,\quad C\in V, \quad V\in V,\quad C\in C \)



Yo no tengo una "mente clásica" y sin embargo me chirría igual, porque creí que la fórmula \( V\in V \) no está estratificada. Entonces, ¿cómo es posible escribir eso?

Lo mismo en esto:

Citar

(...)  el tipo de \( \bigcup X \) y \( \bigcap X \) es siempre una unidad inferior al tipo de \( X \) (...)

en contraste con esto otro:

Citar
\( \bigcup\emptyset = \emptyset,\quad \bigcap\emptyset =V \)

¿Entonces el vacío y el universo no tienen tipo? ¿Cómo se entiende esto?

________________

He llegado hasta el tema de definir funciones en NFU (y de ojeada lo de los ordinales y cardinales).

En cuanto a las funciones, me parece natural preguntar si es posible definir funciones de un conjunto \( A \) en \( \mathcal PA \), pues dominio e imagen tienen tipos distintos en la estratificación.

Es muy natural en álgebra definir un isomorfismo que haga algo como esto:
\( f:A\to\mathcal PA,\quad f(x) = \{x\} \).

¿Se puede hacer esto en NFU? ¿Es muy complicado?

¿Hay algo importante que no estoy entendiendo?

Título: Re: NFA
Publicado por: Carlos Ivorra en 22 Enero, 2012, 03:44 am
Hola, argentinator. Gracias por tus comentarios.

Pienso que para tener el derecho de usar siglas en castellano, necesitamos tener una teoría desarrollada por un matemático hispanoparlante.

Y luego, claro, que esas siglas o nomenclatura se usen en todo el planeta uniformemente.

En realidad la falta de uniformidad es algo que me pone los pelos de punta, porque si hay un lugar donde debe haber acuerdo y falta de ambigüedad es en la matemática.

P.D.: Obviamente que este asunto es el menos importante de todos los que se puedan discutir.

Ciertamente, es lo menos importante. Mi opinión es que, igual que decimos "conjunto" y no "set" y decimos "átomo" y no "urelement" podemos decir NFA y no NFU. No veo por qué toda palabra es traducible pero las sílabas no pueden serlo. NFA no es sino el nombre de una teoría, y no nombras en alemán la teoría de la relatividad porque Einstein fuera alemán.

Citar
Axioma de los átomos: \( \forall xy(x\in y\rightarrow \text{cto}\,y) \)

En palabras: Si un objeto tiene elementos, entonces es un conjunto o, equivalentemente, los átomos son objetos sin elementos.

Tengo dudas sobre cómo está enunciado esto.
Yo diria que: "un objeto es un conjunto si EXISTE otro objeto que pertenece a él".
Así, escribiría: \( \forall y(\exists{x}(x\in y)\qquad \rightarrow \text{cto}\,y) \).

Las dos fórmulas son lógicamente equivalentes. Una implica la otra y viceversa. No sé si cuestionas esto o sólo dices que psicológicamente es más clara la forma que propones.

Por contrarrecíproco sale que un no-conjunto (o átomo) no tiene elementos.

Cierto. Ése es el contenido del axioma de los átomos: que no hay relaciones de pertenencia entre ellos.

¿Y el \( \emptyset \)? ¿Conjunto o átomo?

Es un conjunto. No creo que haga falta aclarar mucho porque tú mismo dices más abajo que ya lo tienes claro. El axioma de los átomos dice que los átomos no tienen elementos, pero no que todo objeto que no tenga elementos sea un átomo. Por el contrario, el axioma de formación de conjuntos implica la existencia de un conjunto sin elementos y el axioma de extensionalidad (que sólo afecta a conjuntos) implica que el conjunto vacío es el único conjunto sin elementos.

Citar
A decir verdad, no vamos a tomar el axioma de los átomos como axioma de NFA por una razón muy simple: aunque lo hiciéramos, jamás tendríamos ocasión de usarlo.

Entonces no entiendo dónde se produce la distinción de Jensen entre una teoría que sí tiene átomos.

El uso del operador "cto" en las fórmulas y axiomas debiera ser suficiente, pero deja la puerta abierta a una teoría muy general, que no se sabe si tiene o no tiene algún átomo.

Creo que aquí te confundes: el axioma de los átomos no dice nada sobre que existan o no existan átomos. Sólo dice que, en caso de que haya átomos, no se dan relaciones de pertenencia entre ellos.

El axioma que afirma que no existen átomos es el que he llamado NA y ese sí que es crucial. Aún no he llegado ahí, pero veremos que el axioma de elección implica que existen átomos, luego, si queremos usarlo, no podemos permitirlos el lujo de suponer que no existen.

Quizá la confusión se debe a que implícitamente estás identificando "átomo" con objeto sin elementos (distinto del conjunto vacío). El axioma de los átomos justifica esa identificación, si es que quieres hacerla, pero no es necesario. Oficialmente un "átomo" es simplemente un objeto que no es un conjunto, por lo que el axioma de extensionalidad no dice nada sobre sus posibles elementos y dichos "posibles elementos" son irrelevantes en la teoría, hasta el punto de que puedes suponer sin cambiar nada esencial que los átomos no tienen elementos (y es lo más natural, aunque en la práctica no te aporta nada relevante).

Más claramente: ¿puede ocurrir que \( a \) y \( b \) sean dos átomos y se cumpla \( a\in b \). Pues, sin el axioma de los átomos puede ocurrir, pero no significa nada, porque, como el axioma de extensionalidad no afecta a los átomos, no puedes considerar a éstos como conjuntos. Aunque sucediera que \( a \) es el único objeto que pertenece al objeto \( b \), eso no haría a \( b \) igual al conjunto \( \{a\} \). Digamos que \( \{a\} \) sería el autétntico conjunto que tiene a \( a \) por único elemento y el átomo \( b \) sería un objeto que anecdoticamente contiene a \( a \).

Otra forma de verlo: imagina que tienes un modelo de NFA (tú llámala NFU si quieres, que me parece muy bien) en el que se cumple el axioma de los átomos, es decir, los átomos no tienen elementos. Ahora imagina que modificas el modelo modificando la relación de pertenencia de modo que un determinado átomo pase a ser elemento de otro átomo. ¿Qué sucede entonces? Pues nada, que sigues teniendo un modelo de NFA en el que ya no se cumple el axioma de los átomos, pero los demás axiomas se siguen cumpliendo, porque ningún axioma de NFA afirma nada sobre posibls elementos de átomos.

Citar
En palabras: Dos pares ordenados son iguales si y sólo si tienen la misma primera componente y la misma segunda componente.

De este modo, por el mero hecho de introducir el nuevo signo, estamos diciendo que, para cada par de objetos \( x \)  e \( y \), existe un tercer objeto (no especificamos si es un átomo o un conjunto)

Tomo nota de este hecho de que los pares ordenados no se sabe si son conjuntos o no.

Más adelante demostraremos que muchos pares ordenados son necesariamente átomos (aunque es consistente exigir que un par ordenado de conjuntos sea un conjunto).

También se pone en evidencia que siempre se necesita en matemática usar pares o n-uplas ordenadas, aún en las fases más elementales de cualquier teoría, pues se necesitan relaciones y funciones.

Eso es indudable.

Citar
Por consiguiente, la paradoja de Russell no puede demostrarse en NFA, sus axiomas no afirman que exista el conjunto \( \{x\mid x\notin x\} \) , ya que la fórmula que (supuestamente) lo define no está estratificada.

Es interesante notar que, mientras en ZFC se prohíben desde los axiomas conjuntos que den lugar a paradojas,
en NFU (o sea NFA, o sea NFU, o sea...  :banghead: ) en cambio la prohibición se hace directamente desde el lenguaje.

No sé si a alguien se le había ocurrido antes esto de "meterse con el lenguaje".

Bueno, NFA es el último paso en una cadena de simplificaciones de la teoría de los Principia Mathematica.

A simple vista parece también que la estratificación impide conjuntos que cumplan algo como X = {X}, o sea, se impiden aquellos casos patológicos que en ZFC se suprimen por medio del axioma de regularidad.
¿Estoy acertado en esto? Pues a mí me da que si X tiene cierto tipo \( m \), necesariamente {X} tiene un tipo mayor \( m' \), pero entonces por el primer axioma y por ser X = {X}, se obtiene que X tiene tipo \( m' \), pero X no puede tener asociados dos tipos distintos \( m,m' \).

Aquí creo que empiezas a mostrar dos errores que a medida que avanza el mensaje se van agravando:

1) En la teoría original de Russell y en algunas de sus simplificaciones, cada objeto tiene un tipo asignado, pero en NFA los tipos sólo se asignan a los términos del lenguaje, es decir, a los nombres de los objetos, lo cual permite que un mismo objeto aparezca con tipos distintos si se le nombra con nombres distintos.

El caso más trivial es el siguiente: Tú puedes nombrar el conjunto vacío con el término \( \{x\mid x\neq x\} \), paro también con el término \( \{y\mid y\neq y\} \), donde \( x \) e \( y \) son variables distintas, y nada te impide estratificar una fórmula asignando tipos distintos a las variables \( x \) e \( y \), con lo que puedes hacer que el conjunto vacío aparezca en ellas con tipos distintos, correspondientes a nombres distintos.

Luego tienes el caso de los parámetros. En uno de los mensajes pongo un ejemplo con la unión. El axioma de formación de conjuntos te asegura, en particular que

\( \forall XY\exists Z(\forall u(u\in Z\leftrightarrow u\in X\lor u\in Y)) \),

porque esta fórmula está estratificada. Una vez cuentas con esta fórmula, nada te impide aplicarla a un conjunto \( X \) cualquiera y al conjunto \( Y=\{X\} \), con lo que obtienes la existencia del conjunto \( X\cup\{X\} \), sin que importe que este término no esté estratificado.

2) Nunca debes olvidar que el axioma de formación de conjuntos es únicamente una condición suficiente para que exista un conjunto, pero no necesaria.

Tienes razón en que el axioma de formación de conjuntos no te permite asegurar que exista un conjunto tal que \( X=\{X\} \), pero tampoco te dice que no exista. Ahora mismo no sabría decirte si es consistente con NFA que exista un conjunto así. Ya pensaré en ello.

Otra duda técnica: ¿queda claro de entrada este hecho de que un cierto objeto \( x \) no tiene permitido tener asociados dos tipos distintos en una misma fórmula?

Ojo, un objeto no, una variable. Y sólo en una fórmula estratificada, es decir, una fórmula a la que pretendas aplicar el axioma de formación de conjuntos. Por lo demás, puedes demostrar fórmulas no estratificadas, como

\( \forall X(X\in X\cup\{X\}) \)

Esto es un teorema (no estratificado) de NFA. Si no queda claro con las explicaciones que he dado antes dilo e insisto en ello. (Lo mismo vale para cualquier otra cosa que haya dicho o diga luego, claro.)

Otra cosa que llama la atención enseguida es que los pares ordenados tienen el mismo tipo que sus componentes.
Eso es algo inesperado para mí, porque uno está acostumbrado a poner en ZFC los pares ordenados como conjuntos que contienen de alguna forma algo enrevesada a sus componentes (\( (x,y)=\{\{x\},\{x,y\}\} \) por ejemplo). El objeto \( \{\{x\},\{x,y\}\} \) tiene tipo 2 veces mayor que el de \( x,y \).

Explico la situación en el mensaje #13. No sé si lo has visto. Si no lo has visto será más fácil que lo mires antes de que yo repita lo dicho allí, si lo has visto y no queda claro, dilo y lo hablamos.

¿Implica necesariamente esto que los pares ordenados en NFU no son conjuntos?

En NF se pueden definir pares ordenados nivelados que son conjuntos, pero con una definición bastante más complicada que la usual que indicas. En NFU dicha definición vale para definir pares ordenados nivelados de conjuntos cuyos elementos sean todos conjuntos, pero para definir pares ordenados nivelados de objetos cualesquiera resulta que muchos de ellos tienen que ser necesariamente átomos. Eso lo demostraré un poco más adelante.

En cuanto a los ordinales de Von Newmann ya lo hemos comentado al principio, que no andan muy bien, pues no ellos necesitarían fórmulas no estratificadas como \( (x\in y)\wedge (x\subset y) \).

Pero eso no impide que puedan definirse. Se pueden definir exactamente igual que en ZFC (una definición rápida sería definirlos como conjuntos transitivos bien ordenados por la inclusión, lo cual no requiere el axioma de formación de conjuntos, y nada te impide demostrar que \( 0=\emptyset \) es un ordinal de von Neumann, y que \( 1=\{0\} \) también lo es, y \( 2=\{0,1\} \), etc. pero poco más puedes probar. En cuanto quieres definir conjuntos de ordinales, te encuentras con que no puedes aplicar el axioma de formación de conjuntos), aunque no puedes demostrar que exista el ordinal \( \omega \) (como en ZFC sin el axioma de infinitud).

La verdad es que en ZFC uno "echa de menos" el enfoque de los cardinales a través de "clases de equivalencia".

Je, je. Eso está entre las virtudes de NFA.

En NGB o en MK uno puede definir los cardinales como clases de equivalencia, pero no los puede usar, porque no puede formar "conjuntos cuyos elementos son números cardinales".

Exacto.

Más aún, Carlos ha respondido en parte lo siguiente:

Citar
El concepto de ordinal de von Neumann se puede definir en toda teoría de conjuntos entre cuyos teoremas se encuentren el axioma de extensionalidad, la existencia de la unión de dos conjuntos, del complemento de dos conjuntos, del par desordenado de dos conjuntos y del conjunto vacío. No hace falta nada más, en particular no hace falta ningún axioma de formación de conjuntos o subconjuntos a partir de fórmulas.

Todos estos resultados son teoremas de NFA, luego en NFA puedes definir el concepto de ordinal (de von Neumann) e incluso el de número natural (de von Neumann, de modo que , etc.)

Ahora bien, las fórmulas "x es un ordinal" o "x es un número natural" (en este sentido de von Neumann) no están estratificadas y, por ello dan muy poco juego. En NFA no es posible demostrar que exista el conjunto de los números naturales en este sentido (pero sí que se pueden definir de otro modo, estratificado, y todo va bien).

Esta respuesta contiene un hecho interesante.
Es sabido que en el tema de la incompletitud de Godel, la incompletitud surge al pretender utilizar una fórmula que diga algo como "x es un número natural".

Esto no lo entiendo. En toda teoría de conjuntos tienes una fórmula que dice "x es un número natural", a saber, la definición de "número natural".

Según lo dicho por Carlos, esta frase no existe para ordinales de Von Neumann en NFU.

No te sigo. En el párrafo que citas digo expresamente que en NFA puedes definir la fórmula "x es un número natural (en el sentido de von Neumann)", pero que la fórmula no es estratificada y eso hace que se pueda hacer poca cosa con los números naturales de von Neumann. Ni siquiera puedes demostrar que para todo número natural n (de los definidos en NFA como clases de equivalencia) existe un número natural de von Neumann \( n' \) tal que \( |n'|=n \).

Me pregunto si hay alguna relación entre estos temas de la estratificación y la incompletitud de Godel.

Como ya digo, no he entendido lo que has dicho sobre la incompletitud y los números naturales.

Mi olfato dice que la incompletitud de Godel sigue firme en NFU, pero bueno, no sé.

Por supuesto. Los teoremas de incompletitud se aplican a toda teoría recursiva (es decir, en los que esté claro qué es un axioma y qué no lo es, como es el caso de NFA) en la que se pueda definir de algún modo el concepto de "número natural" con el único requisito de que se puedan demostrar los axiomas de Peano, y en NFA se pueden definir los números naturales (no como en ZFC, sino como clases de equivalencia) y demostrar los axiomas de Peano. Esto basta para asegurar que los teoremas de incompletitud son aplicables a NFA.

En realidad los números naturales y los infinitos siempre traen incomodidades.

No sé si te has fijado en que en NFA se construyen los números naturales sin necesidad de un axioma de infinitud. ¿A que es curioso?

Son lo más básico de la matemática, y sin embargo nunca se encuentra una teoría que sea del todo "cómoda" o que exprese o refleje todo lo que hace falta o se desea.
Como yo lo veo, esto es indicio de un problema más profundo, y es que nadie entiende qué son los números naturales ni mucho menos los transfinitos,
por lo cual no puedo admitir que se los utilice despreocupadamente al construir una teoría axiomática.

Bueno, si entramos otra vez en eso no saldremos nunca.  :D Creo que no puedo decirte nada al respecto que no te haya dicho ya.

Este asunto de la estratificación es una idea interesante, pero a mí lo que no me gusta es el hecho de asignarle un número natural o entero a una expresión.

Estaba esperando esto desde el primer día. Mucho has tardado en decirlo. ;D

Podría admitirla sin problemas, en cambio, si la cantidad de tipos aceptadas fuera finita, y no todo el rango de los enteros.

Lo de admitir tipos enteros es sólo por comodidad, porque si a una fórmula estratificada le sumas un mismo número natural a todos los tipos, obtienes una nueva estratificación, por lo que no pierdes generalidad si supones que los tipos son naturales. Por otra parte, para estratificar una fórmula concreta sólo necesitas una cantidad finita de tipos, que podrías incluso acotar a priori a partir de la estructura de la fórmula. Te digo esto porque es importante para ti. Para mí no tiene la importancia que tú le das.

Según ví en Wikipedia, NF es equivalente a \( \textsf{NF}_4 \), en que se admiten 4 grados de estratificación como máximo. (Si es que he entendido bien el asunto  :banghead: ).

¿Se puede convertir NFU en algo como un \( \textsf{NFU}_4 \), digamos?

Pues no sé si cuatro concretamente, pero sí sé que NFA es finitamente axiomatizable, es decir, que puedes sustituir los infinitos casos particulares del esquema de formación de conjuntos por unos pocos casos particulares concretos (te los podría enumerar, si quieres, pero ahora mismo no los tengo a mano). De ese modo, puedes eliminar por completo el concepto de fórmula estratificada de los axiomas de NFA. Basta con que digas: estas fórmulas concretas (casos particulares del esquema de formación de conjuntos) son axiomas, y ya está. Luego puedes demostrar como teorema el axioma de formación de conjuntos para cualquier fórmula estratificada (sin límite para el número de tipos admisibles). Esto es más fuerte que lo que tú pedías. No sé si estarás de acuerdo y si te resolverá tu objeción de principios.

En cuanto a lo que pregunté arriba sobre el conjunto vacío, creo que está muy bien explicado acá:

Citar
Alguien podría pensar que podríamos haber incorporado el axioma de los átomos a la teoría a la vez que eliminábamos el signo primitivo cto definiendo un átomo como un objeto sin elementos y un conjunto como un objeto con elementos. Eso no es conveniente porque nos interesa que la teoría tenga un conjunto vacío, es decir, un objeto que no tiene elementos y, a pesar de ello, no es un átomo, sino un conjunto. El axioma de extensionalidad implica que el conjunto vacío es el único conjunto sin elementos, pues dos conjuntos vacíos serían dos conjuntos con los mismos elementos (ninguno) y no puede haber dos conjuntos distintos con los mismos elementos. En cambio, el axioma de extensionalidad no afecta a los átomos, por lo que nada impide que haya muchos átomos distintos y que ninguno tenga elementos.

Vale

Lo siguiente no lo tengo claro:

Citar
Cabe destacar algunas relaciones obvias que pueden "chirriar" a una mente clásica: \( V\in C,\quad C\in V, \quad V\in V,\quad C\in C \)

Yo no tengo una "mente clásica" y sin embargo me chirría igual, porque creí que la fórmula \( V\in V \) no está estratificada. Entonces, ¿cómo es posible escribir eso?

Por "clasica" quería decir simplemente "que piensa en términos de ZFC". Aquí volvemos a lo que te señalaba antes: el axioma de formación de conjuntos prohibe usar fórmulas no estratificadas para definir conjuntos (y no en el sentido de asegurar que las fórmulas no estratificadas no definen conjuntos, sino únicamente en el de no asegurar que lo hacen), pero eso no significa que no puedas demostrar teoremas no estratificados.

Pese a todo, estos ejemplos no serían realmente un ejemplo. La fórmula

\( \{x\mid x=x\}\in \{y\mid y=y\} \)

Sí que está estratificada, y es equivalente a \( V\in V \). Aquí es esencial lo que te decía de que los tipos no se asignan a objetos, sino a nombres de objetos, y podemos jugar con que un mismo objeto admite nombres distintos.

No obstante, ya te he puesto antes un ejemplo de teorema genuinamente no estratificado, como \( X\in X\cup\{X\} \).

Insisto: en NFU no están prohibidos los teoremas no estratificados, sino únicamente no tenemos garantías de que las fórmulas no estratificadas definan conjuntos, lo cual no significa que no los definan. Por ejemplo, la fórmula \( u\in X\lor u=X \) sí que define un conjunto, aunque no está estratificada. El conjunto

\( \{u\mid u\in X\lor u=X\} \)

existe, lo cual no se deduce de aplicar directamente el axioma de formación de conjuntos (sería incorrecto), pero sí de aplicarlo a la fórmula estratificada \( u\in X\lor u=Y \) y luego particularizar el axioma es decir, la fórmula

\( \forall XY\exists Z\forall u(u\in Z\leftrightarrow u\in X\lor u=Y) \)

al caso \( X=Y \).

Si esto te parece "trampa" insiste, porque es fundamental que quede claro.

Lo mismo en esto:

Citar
(...)  el tipo de \( \bigcup X \) y \( \bigcap X \) es siempre una unidad inferior al tipo de \( X \) (...)

en contraste con esto otro:

Citar
\( \bigcup\emptyset = \emptyset,\quad \bigcap\emptyset =V \)

¿Entonces el vacío y el universo no tienen tipo? ¿Cómo se entiende esto?

Ningún objeto tiene tipo. Sólo los nombres de objetos tienen tipo. Los casos que citas se pueden ver como fórmulas estratificadas con el truco que ya te he explicado:

\( \bigcup \{x\mid x\neq x\}=\{y\mid y\neq y\} \),

pero, aunque no fuera así, insisto en que hay muchos teoremas de NFA que no están estratificados. No pasa nada por ello. La única restricción concerniente a la estratificación es que no es lítico tomar como axioma de NFA un caso particular del axioma de formación de conjuntos cuya fórmula no esté estratificada, y hemos visto un ejemplo de un caso particular de dicho axioma con una fórmula no estratificada que, a pesar de no ser un axioma de NFA, resulta ser un teorema de NFA. Ojo con eso.

He llegado hasta el tema de definir funciones en NFU (y de ojeada lo de los ordinales y cardinales).

En cuanto a las funciones, me parece natural preguntar si es posible definir funciones de un conjunto \( A \) en \( \mathcal PA \), pues dominio e imagen tienen tipos distintos en la estratificación.

Es muy natural en álgebra definir un isomorfismo que haga algo como esto:
\( f:A\to\mathcal PA,\quad f(x) = \{x\} \).

¿Se puede hacer esto en NFU? ¿Es muy complicado?

Por ejemplo, tú puedes probar que

\( \forall AB\exists f(f: A\longrightarrow \mathcal PB\land \forall a\in A\ f(a)=\emptyset) \),

que es una fórmula estratificada, y luego puedes particularizar este teorema al caso \( B=A \) y tienes probada la existencia de una aplicación \( f: A\longrightarrow \mathcal PA \) dada por \( a\mapsto \emptyset \).

Ahora bien, puede probarse (está probado más allá del mensaje hasta el que has llegado) que no existe ninguna aplicación \( V\longrightarrow \mathcal PV \) tal que \( x\mapsto \{x\} \).

Aún no hemos llegado a ello, pero veremos más adelante que los conjuntos \( X \) tales que existe una aplicación \( X\longrightarrow \mathcal PX \) dada por \( x\mapsto \{x\} \) se llaman estrictamente cantorianos, y es consistente añadir a NFA un axioma que asegure que los conjuntos "normales" como \( \mathbb{N} \) son estrictamente cantorianos, mientras que los conjuntos "raros" como \( V \) no lo son.

¿Hay algo importante que no estoy entendiendo?

Sí, hay un par de cosas importantes que no estás entendiendo: 1) que los tipos se asignan a nombres y no a conjuntos y 2) que no hay ningún problema en demostrar en NFA fórmulas no estratificadas. La estratificación sólo es un requisito para tomar como axioma la existencia de un conjunto definido por una fórmula. Nada más.

He tratado de ser lo más claro posible, pero lo cierto es que todo esto está lleno de sutilezas. No dudes en preguntar todo lo que no veas claro (y lo mismo vale para cualquier otro lector). El único interés que tiene que publique esto así y no como un pdf para que cada cual lo entienda como pueda es la oportunidad de resolver todas las dudas conceptuales (o no conceptuales) que puedan surgir.
Título: Re: NFA
Publicado por: argentinator en 22 Enero, 2012, 04:14 am
Bueno, gracias por aclarar todas mis dudas.

Citar
Sí, hay un par de cosas importantes que no estás entendiendo: 1) que los tipos se asignan a nombres y no a conjuntos y 2) que no hay ningún problema en demostrar en NFA fórmulas no estratificadas. La estratificación sólo es un requisito para tomar como axioma la existencia de un conjunto definido por una fórmula. Nada más.

Claro. Lo que yo entendía era que el "tipo" podía ser un número entero que variaba como uno quisiera, de fórmula en fórmula, pero que un cierto objeto x, si tenía menor tipo que u, entonces en cualquier fórmula de NFU tenía que aparecer así, estratificadamente, siendo x de tipo menor que u.

Pero como lo que se estratifican son sólo las variables, entonces ya no tengo claro esto que dije acerca del Axioma de Regularidad de ZFC.

En cuanto a los números naturales de Von Neumann, el problema es que entendía mal la estratificación.
No me podía imaginar una fórmula cuyos "objetos" fueran todos los números naturales.

Tampoco entendía lo que me decís en tu crítica (2). Ahora creo que ya lo entiendo.

______

Bueno, pero y entonces ¿para qué estratificar? Je, je.

Uno podría intentar una forma rebuscada de la paradoja de Russell reetiquetando variables.
Aunque no lo he intentado, y de sólo pensarlo se ve que no se puede hacer esa maldad, porque hay que escribir la fórmula \( x\not\in x \) en la definición del conjunto de Russell.

_______

Citar
No sé si te has fijado en que en NFA se construyen los números naturales sin necesidad de un axioma de infinitud. ¿A que es curioso?

Todavía no llegué a esa parte, pero ciertamente es interesante, sobretodo para un archi-ultrafinitista anarquista como yo.

En cuanto a las cosas que dije de Godel, no tienen ningún sentido.

_______


Citar
es decir, que puedes sustituir los infinitos casos particulares del esquema de formación de conjuntos por unos pocos casos particulares concretos (te los podría enumerar, si quieres, pero ahora mismo no los tengo a mano). De ese modo, puedes eliminar por completo el concepto de fórmula estratificada de los axiomas de NFA.

Maravilloso.  ;D

_____


A los demás comentarios te los respondería con un "ok", jeje.

Mi conclusión es que el tema de estratificar es, en efecto, una sutileza muy grande.
Sigue siendo cierto que es una restricción del lado del lenguaje, pero es más sutil de lo que me imaginaba el principio, pues en nada afecta ni a los objetos de la teoría ni a la forma de ciertos enunciados.

Título: Re: NFA
Publicado por: argentinator en 22 Enero, 2012, 05:53 am
Otra cuestión es que en ZFC, el precio que hay que pagar por evitar las paradojas y el conjunto universal es una larga lista de axiomas. Hay que ir poniendo axiomas para todo: que los pares ordenados, que la unión, que el conjunto potencia, etc.

Todo lo que uno define con cierta función proposicional "tendría que ser" (="uno desearía que sea")  un conjunto, y listo.

En cambio en NFU anotaste sólo 3 axiomas muy sencillos y naturales, lo cual me resulta más satisfactorio.
Título: Re: NFA
Publicado por: Carlos Ivorra en 22 Enero, 2012, 01:26 pm
Bueno, gracias por aclarar todas mis dudas.

A mandar.

Bueno, pero y entonces ¿para qué estratificar? Je, je.

Uno podría intentar una forma rebuscada de la paradoja de Russell reetiquetando variables.
Aunque no lo he intentado, y de sólo pensarlo se ve que no se puede hacer esa maldad, porque hay que escribir la fórmula \( x\not\in x \) en la definición del conjunto de Russell.

Lo que estás diciendo es que la condición de estratificación es más débil de lo que podría parecer, hasta el punto de que uno puede "sentirse con fuerzas" para tratar de burlarla y demostrar la paradoja de Russell, pero no se puede porque Jensen demostró que a partir de un modelo de ZFC puedes construir un modelo de NFA, de modo que si pudieras demostrar la paradoja de Russell en NFA también podrías demostrar una contradicción en ZFC (que podría ser, pero lo que está en juego no es la consistencia de una teoría rara, sino la consistencia de ZFC).

Citar
es decir, que puedes sustituir los infinitos casos particulares del esquema de formación de conjuntos por unos pocos casos particulares concretos (te los podría enumerar, si quieres, pero ahora mismo no los tengo a mano). De ese modo, puedes eliminar por completo el concepto de fórmula estratificada de los axiomas de NFA.

Maravilloso.  ;D

Si te interesa la prueba la puedo poner. Es parecida a la demostración de que NBG es finitamente axiomatizable.

Mi conclusión es que el tema de estratificar es, en efecto, una sutileza muy grande.
Sigue siendo cierto que es una restricción del lado del lenguaje, pero es más sutil de lo que me imaginaba el principio, pues en nada afecta ni a los objetos de la teoría ni a la forma de ciertos enunciados.

Su origen está en la lógica de los Principia Mathematica. Alguien (no recuerdo ahora quién) la simplificó creando la Teoría de Tipos Simple, en la que existen conjuntos de tipo cero, conjuntos de tipo uno, conjuntos de tipo dos, etc. (aquí el tipo sí que es un número asignado a cada objeto y no a los nombres de los objetos), de tal modo que los conjuntos de tipo uno sólo pueden tener elementos de tipo cero, los conjuntos de tipo dos sólo pueden tener elementos de tipo uno, etc. Esto hace que exista un conjunto vacío de tipo cero, un conjunto vacío de tipo uno, etc., y todos ellos son conjuntos distintos. Lo que hizo Quine fue darse cuenta de que si en lugar de asignar tipos a los objetos se los asignamos a los nombres de los objetos, no necesitamos infinitos conjuntos vacíos, sino que basta uno solo que admita nombres de tipos distintos.



No me quedé satisfecho con la explicación que te di sobre el axioma de los átomos. Aunque no me has objetado nada al respecto te voy a poner un ejemplo que se me ha ocurrido y que puede ser ilustrativo:

Imagina que los conjuntos son cajas numeradas y los átomos son bolas numeradas. Cada caja tiene una etiqueta, como C1, C3, C8, B4, B5, que significa que la caja en cuestión tiene dentro las cajas 1, 3, 8 y las bolas 4, 5. En estos términos, la relación de pertenencia es: un objeto A (caja o bola) pertenece a otro objeto B si está dentro de él o, equivalentemente, si B es una caja en cuya etiqueta aparece el nombre de A. La teoría NFA es entonces una teoría sobre las cajas y los objetos que contienen.

Ahora bien, ¿qué pasa si alguien decide colgar etiquetas a las bolas? ¿Qué pasa si a la bola 5 le colgamos una etiqueta en la que pone B3, C8? Pues no pasa nada. Aunque le pongamos etiqueta, la bola 5 sigue siendo una bola y no una caja. Podemos decir que la bola 3 pertenece a la bola 5 porque su nombre aparece en su etiqueta, pero esa "pertenencia" es una pertenencia falsa, porque la bola 5 no es una caja.

Los axiomas de NFA hablan de bolas, de cajas y de etiquetas de cajas. Que a una bola le cuelgues una etiqueta o que quites todas las etiquetas de las bolas no afecta en nada a lo que los axiomas de NFA dicen de las bolas y las cajas. El axioma de los átomos afirma que "las bolas no tienen etiquetas" lo cual simplemente elimina unos objetos redundantes en la teoría (las etiquetas posibles de las bolas o, equivalentemente, relaciones de pertenencia entre átomos que pueden darse o no darse porque los axiomas no hablan de ellas, pero que, si se dan, no son más que convenios gratuitos, como si dices que una manzana pertenece a una pera. Puedes decirlo coherentemente, pero no aporta nada.)

Otra cuestión es que en ZFC, el precio que hay que pagar por evitar las paradojas y el conjunto universal es una larga lista de axiomas. Hay que ir poniendo axiomas para todo: que los pares ordenados, que la unión, que el conjunto potencia, etc.

Todo lo que uno define con cierta función proposicional "tendría que ser" (="uno desearía que sea")  un conjunto, y listo.

En cambio en NFU anotaste sólo 3 axiomas muy sencillos y naturales, lo cual me resulta más satisfactorio.

Je, je. Ésa es otra de las virtudes de NFA. Si no tuviera ninguna virtud, sería tonto estudiarla. Ahora, a medida que te familiarizas con la "dama virtuosa" le irás viendo "excentricidades" que al principio pueden parecer simpáticas, pero al final... ya veremos. Hay quienes los amores a primera vista les duran para siempre y hay quienes se desengañan a medida que profundizan en la relación.
Título: Re: NFA
Publicado por: argentinator en 22 Enero, 2012, 06:06 pm

Otra cuestión es que en ZFC, el precio que hay que pagar por evitar las paradojas y el conjunto universal es una larga lista de axiomas. Hay que ir poniendo axiomas para todo: que los pares ordenados, que la unión, que el conjunto potencia, etc.

Todo lo que uno define con cierta función proposicional "tendría que ser" (="uno desearía que sea")  un conjunto, y listo.

En cambio en NFU anotaste sólo 3 axiomas muy sencillos y naturales, lo cual me resulta más satisfactorio.

Je, je. Ésa es otra de las virtudes de NFA. Si no tuviera ninguna virtud, sería tonto estudiarla. Ahora, a medida que te familiarizas con la "dama virtuosa" le irás viendo "excentricidades" que al principio pueden parecer simpáticas, pero al final... ya veremos. Hay quienes los amores a primera vista les duran para siempre y hay quienes se desengañan a medida que profundizan en la relación.

Si hablamos de "amor", estoy enamorado del deseo específico que nombré: que haya pocos axiomas simples y naturales.
Que NFU tenga esos atributos no hace que sienta "amor" por esa teoría.

En realidad tengo "interés" por todas las posibles teorías que compartan esos atributos, y luego habrá que ver cuál los satisface mejor.
Con esa teoría me caso...  :P

Pero resulta que si uno realmente ha visto tantas teorías, entonces deja de tener sentido que se aferre a una sola de ellas, ya que en realidad uno se convierte en un estudioso de una "teoría acerca de las teorías de conjuntos".

__________

En el fondo lo que me gusta es demostrar sin suponer nada, sin axiomas. Total, si hablamos de "amores" y de gustos, uno puede desear amores imposibles si así lo quiere.
Título: La axiomatización finita de NFA
Publicado por: Carlos Ivorra en 22 Enero, 2012, 06:24 pm
Como le interesa a argentinator expongo aquí una axiomática alternativa de NFA que requiere sólo un número finito de axiomas. Más concretamente, llamamos NFA' a la teoría axiomática cuyos axiomas son el axioma de extensionalidad, el axioma de los pares ordenados y la lista finita de axiomas que daremos aquí. Se cumplen dos hechos:

1) Todos los axiomas que vamos a presentar aquí son teoremas de NFA, de donde se sigue que todo teorema de NFA' es un teorema de NFA.

2) En NFA' se pueden demostrar todos los casos particulares del esquema de formación de conjuntos, por lo que todo teorema de NFA es un teorema de NFA'.

Esto significa que si definimos una teoría por sus teoremas y no por sus axiomas (es decir, si definimos NFA como el conjunto de los teoremas que se deducen de los axiomas de NFA e igualmente con NFA'), entonces NFA y NFA' son la misma teoría.

El hecho 1) lo iremos probando sobre la marcha, es decir, cada vez que presentemos un axioma comprobaremos que es un teorema de NFA. Luego demostraremos 2). Mantenemos el convenio de que las letras mayúsculas representan conjuntos.

Axioma del conjunto vacío: \( \exists A\forall x\ x\notin A \)

Ya sabemos que esto es un teorema de NFA. El axioma de extensionalidad nos da que el conjunto \( A \) dado por el axioma del conjunto vacío es único, luego podemos definir \( \emptyset\equiv A\mid \forall x\ x\notin A \).

Axioma del complemento: \( \forall A\exists B\forall x(x\in B\leftrightarrow x\notin A) \)

Esto es un teorema de NFA, pues en NFA podemos definir \( B=\{x\mid x\notin A\} \), ya que la fórmula que define el conjunto está estratificada.

Nuevamente, el axioma de extensionalidad asegura que el conjunto \( B \) es único, luego podemos definir \( \overline A\equiv B\mid \forall x(x\in B\leftrightarrow x\notin A) \).

A su vez podemos definir \( V=\overline \emptyset \) y se cumple que \( \forall x\ x\in V \).

Axioma del par: \( \forall xy\exists A\forall u(u\in A\leftrightarrow u=x\lor u=y) \)

También sabemos que este axioma es un teorema de NFA. Por extensionalidad el conjunto \( A \) es único y podemos definir

\( \{x,y\}\equiv A\mid \forall u(u\in A\leftrightarrow u=x\lor u=y) \).

Abreviaremos \( \{x\}\equiv \{x,x\} \).

Axioma de la unión: \( \forall A\exists B\forall x(x\in B\leftrightarrow \exists C(x\in C\land C\in A)) \).

También sabemos que esto es un teorema de NFA y por extensionalidad el conjunto \( B \) es único, por lo que podemos definir

\( \bigcup A\equiv B\mid \forall x(x\in B\leftrightarrow \exists C(x\in C\land C\in A)) \).

Esto nos permite definir \( A\cup B \equiv \bigcup\{A,B\} \), y así se cumple que

\( \forall ABx(x\in A\cup B\leftrightarrow x\in A\lor x\in B) \).

Ahora definimos \( A\cap B\equiv \overline{\overline A\cup \overline B} \), con lo que

\( \forall ABx(x\in A\cap B\leftrightarrow x\in A\land x\in B) \).

La definición de inclusión en NFA (restringida a conjuntos) es válida también en NFA'. Con esto queda probado que el álgebra de conjuntos en NFA' es la misma que en NFA.

Axioma del producto cartesiano: \( \forall AB\exists C\forall x(x\in C\leftrightarrow \exists uv(u\in A\land v\in B\land x=(u,v))) \)

Esto es un teorema de NFA, pues hemos visto que en NFA existe el producto cartesiano. Ahora vemos que en NFA' podemos definir igualmente:

\( A\times B\equiv C\mid \forall x(x\in C\leftrightarrow \exists uv(u\in A\land v\in B\land x=(u,v))) \)

Introducimos ahora la notación siguiente: \( A^1\equiv A \) y, si \( n\geq 1 \) es un número natural (metamatemático), convendremos en que \( A^{n+1}\equiv A^n\times A \).

Esto significa que si escribimos, por ejemplo, \( A^4 \), esto ha de entenderse como una abreviatura por \( A^4\equiv A\times(A\times(A\times A)) \)

Análogamente, en lugar de representar un elemento de \( A^4 \) en la forma \( (a,(b,(c,d))) \), escribiremos simplemente \( (a,b,c,d)\equiv (a,(b,(c,d))) \), pero es importante que los paréntesis omitidos son los que estamos indicando y no otros. En general estamos adoptando el convenio

\( (x_1,\ldots, x_n)\equiv (x_1,(x_2,\ldots, x_n)) \),

lo cual vale para \( n=2 \) si entendemos que \( (x)\equiv x \).

Adoptaremos el convenio usual según el cual \( x\,R\,y\equiv (x,y)\in R \).

Axioma de la diagonal: \( \exists A\forall x(x\in A\leftrightarrow \exists u\ x=(u,u)) \)

Esto es un teorema de NFA, pues en NFA podemos definir el conjunto \( \{x\mid \exists u\ x=(u,u)\} \), ya que la fórmula está estratificada.

Definimos \( I\equiv A\mid \forall x(x\in A\leftrightarrow \exists u\ x=(u,u)) \). Podemos pensar en \( I \) como la relación identidad en \( V^2 \) o también como la función identidad \( I:V\longrightarrow V \).

Axioma de la inclusión: \( \exists A\forall x(x\in A\leftrightarrow \exists CD(x=(C,D)\land C\subset D)) \)

Observamos que en NFA podemos definir \( \{(C,D)\mid C\subset D\} \), luego este axioma es un teorema de NFA.

Definimos \( \text{INC}\equiv A\mid \forall x(x\in A\leftrightarrow \exists CD(x=(C,D)\land C\subset D)) \)

Axioma de la relación inversa: \( \forall R\exists S\forall x(x\in S\leftrightarrow \exists uv(x=(u,v)\land (v,u)\in R)) \)

También es claro que en NFA podemos definir \( \{(u,v)\mid (v,u)\in R\} \), luego este axioma es un teorema de NFA. Definimos \( R^{-1}\equiv S\mid \forall x(x\in S\leftrightarrow \exists uv(x=(u,v)\land (v,u)\in R)) \)

Axioma de la composición: \( \forall RS\exists T\forall x(x\in T\leftrightarrow \exists uvw(x=(u,w)\land (u,v)\in S\land (v,w)\in R)) \)

Es claro que esto es un teorema de NFA. Definimos \( R\circ S\equiv T\mid \forall x(x\in T\leftrightarrow \exists uvw(x=(u,w)\land (u,v)\in S\land (v,w)\in R)) \).

Conviene introducir la notación siguiente, donde \( n \) es un número natural metamatemático:

\( R^{(0)}\equiv I,\quad R^{(1)}\equiv R,\quad R^{(n+1)}\equiv R\circ R^{(n)} \)

Axioma del dominio: \( \forall R\exists D\forall x(x\in D\leftrightarrow \exists u\ (x,u)\in R) \)

Sabemos que en NFA se puede demostrar la existencia del dominio, luego este axioma es un teorema de NFA. Definimos \( \mathcal DR\equiv D\mid \forall x(x\in D\leftrightarrow \exists u\ (x,u)\in R) \).

A su vez podemos definir \( \mathcal RR\equiv \mathcal DR^{-1} \), de modo que

\( \forall Rx(x\in \mathcal RR\leftrightarrow \exists u\ (u,x)\in R)) \).

Ahora podemos definir \( C\equiv \mathcal R\text{INC} \). Teniendo en cuenta que la definición de inclusión exige que los objetos sean conjuntos, vemos que

\( x\in C\leftrightarrow \exists u\ u\subset x\leftrightarrow \text{cto}\,x \). Así pues, \( C \) es el conjunto de todos los conjuntos.

Axioma de elevación: \( \forall R\exists S\forall x(x\in S\leftrightarrow \exists uv(x=(\{u\},\{v\})\land (u,v)\in R)) \)

Esto es un teorema de NFA, pues en NFA podemos definir \( \{x\mid \exists uv(x=(\{u\},\{v\})\land (u,v)\in R))\} \), ya que la fórmula está estratificada.

Definimos \( R_1\equiv S\mid\forall x(x\in S\leftrightarrow \exists uv(x=(\{u\},\{v\})\land (u,v)\in R)) \).

Para cada natural \( n\geq 1 \) definimos \( R_{n+1}\equiv (R_n)_1 \), es decir, que \( R_4 \) no es más que una abreviatura por \( R_4\equiv ((R_1)_1)_1)_1 \).

Axiomas de proyección:

\( \exists A\forall x(x\in A\leftrightarrow \exists uv\ x=((u,v),u)) \)

\( \exists B\forall x(x\in B\leftrightarrow \exists uv\ x=((u,v),v)) \)

Es claro que los conjuntos \( A \) y \( B \) pueden obtenerse en NFA a partir del axioma de formación de conjuntos (con fórmulas estratificadas, por supuesto).

Definimos

\( \Pi_1\equiv A\mid \forall x(x\in A\leftrightarrow \exists uv\ x=((u,v),u)) \)

\( \Pi_2\equiv B\mid \forall x(x\in B\leftrightarrow \exists uv\ x=((u,v),v)) \)

Observemos que (con la misma definición de aplicación que hemos dado en NFA) se cumple que \( \Pi_1:V\times V\longrightarrow V \) es la aplicación dada por \( \Pi_1(u,v)=u \), mientras que \( \Pi_2 \) es la aplicación que cumple \( \Pi_2(u,v)=v \).

Éstos son todos los axiomas de NFA', en total son trece axiomas de formación de conjuntos (si contamos como dos los axiomas de proyección) más el axioma de extensionalidad y el del par. De todos modos, estas cuentas son subjetivas, porque podríamos sustituir estos axiomas por la conjunción de todos ellos y entonces NFA' tendría un único axioma.

Ya hemos comprobado que todos los axiomas de NFA' son axiomas de NFA, por lo que todos los teoremas de NFA' son teoremas de NFA. Ahora falta probar el recíproco, para lo cual basta ver que en NFA' se puede demostrar cualquier caso particular del axioma de formación de conjuntos.

Empezamos considerando el caso de una fórmula estratificada \( \phi \) que esté formada mediante signos lógicos distintos del descriptor a partir de subfórmulas de los tipos:

\( \text{cto}\,x,\quad x=y,\quad x\in y,\quad x=(y,z) \).

Sean \( x_1,\ldots, x_n \) todas las variables que aparecen en ella (libres o ligadas) y sea \( t_i \) el tipo de la variable \( x_i \) en una estratificación prefijada. Sea \( N \) un número natural mayor o igual que todos los \( t_i \). Definimos

\( P_N^n\equiv V_{(N-t_1)}\times \cdots \times V_{(N-t_n)} \).

Ejemplo: Imaginemos que la fórmula \( \phi \) tiene cinco variables de tipos \( 0,0,1,2,3 \) y que tomamos \( N=3 \). Entonces

\( P_3^5\equiv V_{(5)}\times V_{(5)}\times V_{(4)}\times V_{(3)}\times V_{(2)} \).

Un elemento arbitrario de \( P_3^5 \) es de la forma \( (\{\{\{\{\{x_1\}\}\}\}\},\{\{\{\{\{x_2\}\}\}\}\},\{\{\{\{x_3\}\}\}\},\{\{\{x_4\}\}\},\{\{x_5\}\}) \), donde \( x_1,x_2,x_3,x_4,x_5 \) son objetos cualesquiera. Abreviaremos esto como \( (\{x_1\}_5,\{x_2\}_5,\{x_3\}_4,\{x_4\}_4,\{x_5\}_3) \). En general, \( \{x\}_n \) representará una \( x \) dentro de \( n \) llaves anidadas.

La definición de \( P_N^n \) hace que la estratificación fijada en \( \phi \) hace que todos los elementos de \( P_N^n \) estén estratificados con tipo \( N \). En general, cuando \( x\in P_n^N \), llamaremos \( x_i \), no a su componente \( i \)-ésima, sino a su componente \( i \)-ésima desprovista de las \( N-t_i \) llaves que le corresponden según la definición de \( P_N^n \).

Veamos ahora que pada cada índice \( 1\leq i\leq N \) existe el conjunto

\( X_i^{n,N}\equiv \{(x,\{x_i\}_{N-t_i})\mid x\in P_N^n\} \).

Demostración:
En efecto, para \( 1\leq i\leq n-1 \) definimos

\( X_i^{n,N}\equiv (P_N^n\times V)\cap (\Pi_1\circ \Pi_2^{(i-1)}) \),

que es un conjunto cuya existencia está justificada en NFA' (hemos probado que existen las proyecciones, y las composiciones, y la intersección, etc.).

Vamos a probar que este conjunto definido así es igual al definido antes, es decir, hemos de probar que sus elementos son exactamente los pares \( (x,\{x_i\}_{N-t_i}) \), con \(  x\in P_N^n \). Ahora bien, si \( u\in X_i^{n,N} \) según la definición que acabamos de dar, tenemos que \( u=(x,y) \), donde \( x\in P_N^n \) e \( y=\Pi_1(\Pi_2^{(i-1)}(x)) \)

Si \( x=(\{x_1\}_{N-t_1},\ldots, \{x_n\}_{N-t_n}) \), recordando que hemos definido una n-tupla como un par ordenado cuya primera componente es la primera componente de la n-tupla y cuya segunda componente es la n-1-tupla formada por las restantes componentes, es claro que cada vez que aplicamos \( \Pi_2 \) a una n-tupla le estamos quitando la primera componente, luego

\( \Pi_2^{(i-1)}(x)=(\{x_i\}_{N-t_i},\ldots, \{x_n\}_{N-t_n}) \), y al aplicar \( \Pi_1 \) a este objeto obtenemos precisamente \( y=\{x_i\}_{N-t_i} \), como queríamos probar.

Para \( i=n \) debemos modificar la definición:

\( X_n^{n,N}\equiv (P_N^n\times V)\cap \Pi_2^{(n-1)} \)

Pues al quitar la primera componente n-1 veces a una n-tupla nos quedamos con su n-sima componente.
[cerrar]

Definición: Si \( \psi \) es una subfórmula de \( \phi \), diremos que \( \psi \) está representada respecto de \( N \) si existe el conjunto

\( R_\psi\equiv \{x\mid x\in P_N^n\land \psi(x_1,\ldots, x_n)\} \)

Toda subfórmula de tipo \( \text{cto}(x_i) \) está representada.

Demostración
Basta observar que

\( \{x\mid x\in P_N^n\land \text{cto}\,x_i\}=\mathcal D(X_i^{n,N}\cap (V\times C_{(N-t_i)})) \).

En efecto, un objeto \( x \) está en el conjunto de la derecha si y sólo si existe otro objeto \( y \) tal que \( (x,y)\in X_i^{n,N}\cap (V\times C_{(N-t_i)}) \), lo cual equivale a que \( x\in P_N^n \), \( y=\{x_i\}_{N-t_i} \) y \( y\in C_{(N-t_i)} \) y las dos últimas condiciones equivalen a que \( x_i\in C \), es decir, a \( \text{cto}\,x_i \).
[cerrar]

Definición: \( R\otimes S\equiv (\Pi_1^{-1}\circ R)\cap (\Pi_2^{-1}\circ S)=\{(x,(y,z))\mid x\,R\,y\land x\,S\,z\} \).

Demostración (de la igualdad)
Un objeto está en \( \Pi_1^{-1}\circ R \) si es de la forma \( (x,u) \) y existe un \( y \) tal que \( (x,y)\in R \) y \( (y,u)\in \Pi_1^{-1} \). Lo segundo significa que \( (u,y)\in \Pi_1 \), es decir, que \( u=(y,v) \), para cierto \( v \).

Si además \( (x,(y,v))\in \Pi_2^{-1}\circ S \), entonces existe un \( z \) tal que \( (x,z)\in S \) y \( (z,(y,v))\in \Pi_2^{-1} \), lo que equivale a que \( ((y,v),z))\in \Pi_2 \), luego \( v=z \), con lo que el objeto de partida era \( (x(y,z)) \), con \( (x,y)\in R \) y \( (x,z)\in S \). Igualmente se prueba la inclusión contraria.
[cerrar]

Veamos ahora que

\( X_i^{n,N}\otimes X_j^{n,N}=\{(x,(\{x_i\}_{N-t_i},\{x_j\}_{N-t_j}))\mid x\in P_N^n\} \).

Demostración:
Un elemento de este conjunto es de la forma \( (x,(y,x)) \), donde \( (x,y)\in X_i^{n,N} \) y \( (x,z)\in X_j^{n,N} \), lo que equivale a que \( y=\{x\}_{N-t_i}, z=\{x\}_{N-t_j} \).
[cerrar]

Como consecuencia, todas las subfórmulas de \( \phi \) de tipo \( x_i=x_j \) están representadas.

Demostración:
Como \( \phi \) está estratificada, tiene que ser \( t_i=t_j \). Basta observar que

\( \{x\mid x\in P_N^n\land x_i=x_j \}=\mathcal D((X_i^{n,N}\otimes X_j^{n,N})\cap (V\times I_{M-t_i})) \).
[cerrar]

La subfórmulas de \( \phi \) de tipo \( x_i\in x_j \) también están representadas.

Demostración:
Por definición de estratificación tiene que ser \( t_j=t_i+1 \). Observemos que \( x_i\in x_j \) equivale a que \( \{x_i\}\subset x_j \), y también a que \( (\{x_i\}_{N-t_i}, \{x_j\}_{N-t_j})\in \text{INC}_{N-t_j} \). Por lo tanto:

\( \{x\mid x\in P_N^n\land x_i=
\in x_j \}=\mathcal D((X_i^{n,N}\otimes X_j^{n,N})\cap (V\times \text{INC}_{M-t_j})) \).
[cerrar]

Las subfórmulas de tipo \( x_k=(x_i,x_j) \) también están representadas.

Demostración:
Por definición de estratificación tenemos que \( t_i=t_j=t_k \). Además

\( x_k=(x_i,x_j)\leftrightarrow (x_k,x_i)\in \Pi_1\land (x_k,x_j)\in \Pi_2 \), luego

\( \{x\mid x\in P_N^n\land x_k=(x_i,x_j)\} \) es igual a

\( \mathcal D((X_k^{n,N}\otimes X_i^{n,N})\cap (V\times (\Pi_1)_{N-t_k}))\cap \mathcal D((X_k^{n,N}\otimes X_j^{n,N})\cap (V\times (\Pi_2)_{N-t_k})) \)
[cerrar]

Es inmediato que si \( \psi \) y \( \chi \) son subfórmulas de \( \phi \) representadas por los conjuntos \( R_\psi \) y \( R_\chi \), entonces \( \overline{R_{\psi}} \) representa a \( \lnot\psi \), \( R_\psi\cap R_\chi \) representa a \( \psi\land \chi \), \( R_\psi\cup R_\chi \) representa a \( \psi\lor \chi \), \( \overline{R_\psi}\cup R_\chi \) representa a \( \psi\rightarrow \chi \) y \( (\overline{R_\psi}\cup R_\chi)\cap (\overline{R_\chi}\cup R_\psi) \) representa a \( \psi\leftrightarrow \chi \).

Veamos ahora que si una subfórmula es de tipo \( \psi\equiv \exists x_i\chi \) y \( \chi \) está representada, entonces \( \psi \) también lo está.

Demostración:
Una comprobación rutinaria muestra que

\( A=X_1^{n,N}\otimes \cdots \otimes (V\times V_{N-t_i})\otimes \cdots \otimes X_n^{n,N} \)

\( = \{(x,\{x_1\}_{N-t_1},\ldots , \{u\}_{N-t_i},\ldots, \{x_n\}_{N-t_n})\mid x\in P_N^n\} \).

Aquí hemos de entender el mismo convenio de asociación que en los productos cartesianos, es decir, que los productos se van calculando de derecha a izquierda.

Entonces \( \mathcal D(A\cap (V\times R_{\chi})) \) representa a \( \psi \). En efecto, un objeto \( x \) está en este conjunto si y sólo si existe un \( y \) tal que \( (x,y)\in A\cap (V\times R_\chi) \), lo cual equivale a que \( x\in P_N^n \), \( y=(\{x_1\}_{N-t_1},\ldots , \{u\}_{N-t_i},\ldots, \{x_n\}_{N-t_n}) \), para cierto objeto \( u \) y \( y\in R_{\chi} \). Lo último equivale a que \( y\in P_N^n \) y \( \chi(x_1,\ldots, u,\ldots, x_n) \).

Por lo tanto, los elementos del conjunto son los \( x\in P_n^N \) tales que existe un \( u \) tal que \( \chi(x_1,\ldots, u,\ldots, x_n) \) o lo que es lo mismo, los \( x\in P_N^n \) tales que \( \exists x_i\chi(x_1,\ldots, x_n)\equiv \psi(x_1,\ldots, x_n) \), que es lo que había que probar.
[cerrar]

Finalmente, si una subfórmula es de la forma \( \psi\equiv \forall x_i\chi \) y \( \chi \) está representada, hemos visto que \( \lnot\chi \) está representada, y que \( \exists x_i\lnot\chi \) está representada, y que \( \lnot\exists x_i\lnot\chi \) está representada, pero esta fórmula es equivalente a \( \psi \), luego \( \psi \) está representada.

Con esto podemos asegurar que \( \phi \) está representada.

Demostración:
Podemos probar que toda subfórmula \( \psi \) de \( \phi \) está representada. En efecto, si no fuera así, habría una subfórmula \( \psi \) no representada que podemos tomar de longitud mínima. No puede ser de tipo \( \text{cto}\,x_i \), ni \( x_i=x_j \), ni \( x_i\in x_j \) ni \( x_k=(x_i,x_j) \), pues hemos probado que todas estas subfórmulas están representadas, pero tampoco puede ser de tipo \( \lnot \chi \), pues entonces \( \chi \) tendría longitud menor, luego está representada, y hemos probado que entonces \( \psi \) también lo está. Igualmente se descarta que pueda ser de tipo \( \chi_1\land \chi_2 \), \( \chi_1\lor \chi_2 \), etc., pues hemos probado que cualquier fórmula construida a partir de fórmulas representadas está representada, y así tenemos una contradicción.

Como toda subfórmula de \( \phi \) está representada, en particular lo está la propia \( \phi \).
[cerrar]

En definitiva, hemos probado que toda fórmula estratificada construida a partir de subfórmulas de tipos \( \text{cto}\,x_i,\quad x_i=x_j,\quad x_i\in x_j,\quad x_k=(x_i,x_j) \) está representada, es decir, existe el conjunto

\( R_\phi=\{x\mid x\in P_N^n\land \phi(x_1,\ldots, x_n)\} \)

El orden de las variables lo hemos elegido arbitrariamente. Podemos suponer que las variables \( x_1,\ldots, x_m \) están ligadas y las restantes libres. Aplicando \( \mathcal R \) a este conjunto \( m \) veces obtenemos un conjunto análogo con \( n-m \) componentes donde ahora todas las variables son libres. Renumerándolas, tenemos el conjunto

\( R'_\phi=\{x\mid x\in P_N^n\land \phi(x_1,\ldots, x_n)\} \)

donde ahora \( x_1,\ldots x_n \) son sólo las variables libres en \( \phi \). Vamos a renombrar estas variables libres como \( x,x_1,\ldots, x_n \) con tipos \( t, t_1,\ldots, t_n \). Así, si fijamos objetos \( x_1,\ldots, x_n \), podemos construir el conjunto

\( \mathcal D(R_\phi\cap (V_{N-t})\times \{(\{x_1\}_{N-t_1},\ldots, \{x_n\}_{N-t_n})\}) \)

que no es sino \( \{x\mid \phi(x,x_1,\ldots, x_n)\} \), es decir, el axioma de formación de conjuntos para la fórmula \( \phi \).

Para acabar de demostrar que NFA' es equivalente a NFA sólo falta probar que toda fórmula estratificada es equivalente en NFA' a una fórmula estratificada en las condiciones de la que hemos considerado (es decir, sin descriptores y cuyas subfórmulas atómicas sean de los tipos indicados).

En primer lugar observamos que toda fórmula estratificada puede transformarse en otra equivalente (también estratificada) en la que el signo \( \text{cto} \) sólo aparezca seguido de una variable,  el signo \( \in \) sólo aparezca en subfórmulas de tipo \( x\in y \) y el signo \( = \) sólo aparezca en subfórmulas de tipo \( x=y \), \( x=(y,z) \) o \( x=y\mid \psi \).

Por ejemplo, si la fórmula dada contiene una subfórmula de tipo \( (t_1,t_2)\in t_3 \), para ciertos términos \( t_1, t_2, t_3 \), la estratificación exige que los dos primeros tengan el mismo tipo n y el segundo n+1. Entonces podemos sustituir la subfórmula por

\( \exists uvwx(u=t_1\land v=t_2\land w=t_3\land x=(u,v)\land x\in w) \).

Es claro que esto mantiene la estratificación. Analizando un número finito de posibilidades similares a ésta llegamos a lo que hemos afirmado. Por último, basta observar que toda subfórmula de tipo \( x=y\mid \psi(y) \) se puede sustituir por \( \psi(x) \), lo cual conserva la estratificación, pues el tipo de \( x \) tiene que ser el mismo que el de \( y \). Esto nos lleva a una fórmula estratificada equivalente del tipo que hemos considerado en la demostración del axioma de formación de conjuntos.

Terminamos observando que, en realidad, al sustituir \( x=y\mid \psi(y) \) por \( \psi(x) \) estamos asumiendo que sólo empleamos descripciones cuando está justificando que existe un único objeto que cumple la descripción. Si no es así el argumento requiere un retoque técnico (esencialmente, sustituir \( x=y\mid \psi(y) \) por una fórmula que diga que \( x \) es el único que cumple \( \psi(x) \) si efectivamente hay un único objeto que lo cumpla o bien \( x=\emptyset \) en caso contrario), pero no vamos a entrar en esos detalles técnicos.
Título: La función beth
Publicado por: Carlos Ivorra en 28 Enero, 2012, 03:31 pm
Necesitamos construir la función beth. La idea es la siguiente, partimos de un cardinal arbitrario \( \kappa \) y queremos definir \( \beth_\alpha(\kappa) \), donde \( \alpha \) varía en los ordinales, de modo que

\( \beth_0(\kappa)=\kappa,\ \ \beth_{\alpha+1}(\kappa)=2^{\beth_\alpha(\kappa)} \) y, para todo ordinal límite \( \lambda \), \( \beth_\lambda(\kappa)=\sup\{\beth_\delta(\kappa)\mid \delta<\lambda\} \).

Aquí nos encontramos con un problema que no tiene paralelo en ZFC, y es que \( 2^\mu \) no siempre está definido, por lo que puede suceder que \( \beth_\alpha(\kappa) \) esté definido pero \( \beth_{\alpha+1}(\kappa) \) no lo esté. Para evitar que este inconveniente complique la definición de la función beth definimos como sigue la función exponencial \( \exp: K\cup\{\emptyset\}\longrightarrow K\cup\{\emptyset\} \)

\( \exp\equiv\{ (\kappa,\mu)\mid\kappa\in K\land (\kappa\leq \kappa_1\land \mu=2^\kappa)\lor (\kappa<\kappa_1\land \mu=\emptyset)\}\cup\{(\emptyset,\emptyset)\} \).

Escribimos así la definición para poner en evidencia que está estratificada, pero sería más natural escribir:

\( \exp \kappa=\left\{\begin{array}{ll}2^\kappa&\text{si }\kappa\leq \kappa_1,\\ \emptyset&\text{si }\kappa_1<\kappa\lor \kappa=\emptyset.\end{array}\right. \)

De este modo tenemos la garantía de que \( \exp\kappa \) siempre está definido, y coincide con \( 2^\kappa \) cuando es posible. En particular, si \( \kappa\leq\kappa_1 \), entonces \( \exp\kappa\in K \) y \( \kappa<\exp\kappa \). Ahora definimos

\( I(\kappa)\equiv \{A\mid A\subset K\cup\{\emptyset\}\land \kappa\in A\land \forall \mu(\mu\in A\rightarrow \exp\mu\in A)\land
 \)

\(
\forall B(B\subset A\cap K\land B\neq\emptyset\land \forall \mu\in B\exp\mu\in B\rightarrow \sup B\in A)\} \)

En palabras: \( I(\kappa) \) es el conjunto de todos los subconjuntos \( A \) de \( K\cup\{\emptyset\} \) que cumplen tres propiedades:

1) \( \kappa\in A \)

2) Si \( \mu\in A \) entonces \( \exp\mu\in A \)

3) Si \( B \) es un subconjunto no vacío de \( A\cap K \) con la propiedad de que para todo \( \mu\in B \) también \( \exp\mu\in B \), entonces el supremo de \( B \) está en \( A \).

Definimos \( C(\kappa)=\bigcap I(\kappa) \) y \( \Phi(\kappa)=C(\kappa)\setminus\{\emptyset\} \).

La idea es que \( C(\kappa) \) es el menor subconjunto de \( K \) que contiene a \( \kappa \) y del que no nos salimos ni aplicando exp a cualquiera de sus elementos ni tomando supremos de subconjuntos cerrados para exp. Si al aplicar repetidamente exp y tomar supremos llegamos a salirnos de K y nos aparece el conjunto vacío, al final lo eliminamos definiendo \( \Phi(\kappa) \).

Estas ideas se plasman en las propiedades siguientes, que son inmediatas a partir de la definición (notemos únicamente que \( I(\kappa)\neq \emptyset \), porque \( K\in I(\kappa) \)).

a) \( \Phi(\kappa)\subset K\land \kappa\in \Phi(\kappa) \)

b) \( \forall \mu(\mu\in \Phi(\kappa)\land \exp\mu\neq\emptyset\rightarrow \exp\mu\in \Phi(\kappa)) \)

c) \( \forall B(B\subset\Phi(\kappa)\land B\neq \emptyset\land \forall \mu\in B\ \exp\mu\in B\rightarrow \sup B\in \Phi(\kappa)) \)

Por ejemplo, b) se demuestra observando que si \( \mu\in \Phi(\kappa) \), entonces está en todos los elementos de \( I(\kappa) \), luego \( \exp\mu \) también está en todos los elementos de \( I(\kappa) \), luego está en \( C(\kappa) \) y como no es vacío, está en \( \Phi(\kappa) \). Pero lo más importante es que \( \Phi(\kappa) \) es el mínimo conjunto que cumple estas propiedades:

d) Si un conjunto \( A \) cumple

d1) \( A\subset\kappa\land\kappa\in A \)

d2) \( \forall\mu(\mu\in A\land \exp\mu\neq\emptyset\rightarrow \mu\in A) \)

d3) \( \forall B(B\subset A\land B\neq\emptyset\land \forall \mu(\mu\in B\land \exp\mu\neq\emptyset\rightarrow \sup B\in A)) \)

entonces \( \Phi(\kappa)\subset A \).

En efecto, basta observar que \( A\cup\emptyset\in I(\kappa) \).

De aquí deducimos varias consecuencias:

e) Todo \( \mu\in \Phi(\kappa) \) está en uno de los tres casos siguientes: \( \mu=\kappa \) o bien \( \mu=\exp\nu \), para cierto \( \nu\in \Phi(\kappa) \), o bien \( \mu=\sup B \), para cierto conjunto no vacio \( B\subset \Phi(\kappa) \) cerrado para exp.

Para probarlo basta aplicar d) al conjunto

\( A\equiv\{\mu\mid \mu\in \Phi(\kappa)\land (\mu=\kappa\lor \exists \nu\in \Phi(\kappa)\ \mu=\exp\nu) \)

\(
\lor \exists B(B\subset \Phi(\kappa)\land B\neq \emptyset\land \forall \mu\in B\ \exp\mu\in B\land \mu=\sup B))\} \)

f) \( \mu\in \Phi(\kappa)\rightarrow \kappa\leq\mu \).

Basta aplicar d) al conjunto \( A\equiv\{\mu\mid \mu\in K\land \kappa\leq \mu\} \).

g) \( \forall\mu\nu\in \Phi(\kappa)(\exp\nu\neq\emptyset\rightarrow \mu\leq\nu\lor 2^\nu\leq\mu) \)

Es decir, \( 2^\nu \) es el inmediato sucesor de \( \nu \) en \( \Phi(\kappa) \).

Demostración
Fijamos un \( \nu\in \Phi(\kappa) \) tal que \( \exp\nu\neq\emptyset \) y supongamos que existe un \( \mu\in\Phi(\kappa) \) tal que \( \nu<\mu<2^\nu \). Podemos tomar el menor \( \nu \) posible y, para dicho \( \nu \), tomamos el menor \( \mu \) posible.

No puede ser \( \mu=\kappa \), pues entonces \( \mu\leq\nu \) por f).

Tampoco puede ser que \( \nu \) sea el supremo de un conjunto \( B\subset \Phi(\kappa) \) cerrado para exp, pues entonces, o bien \( \forall\xi\in B\ \xi\leq \nu \), en cuyo caso \( \mu\leq \nu \), o bien existe un \( \xi\in B \) tal que \( \nu\leq\xi \), pero \( \xi<\exp\xi\in B \), luego \( \xi<\mu \) y, por la minimalidad de \( \mu \), tenemos que \( 2^\nu\leq \xi<\mu \), contradicción.

Así pues, según e) existe un \( \xi\in \Phi(\kappa) \) tal que \( \mu=2^\xi \). Como \( \nu<2^\xi<2^\mu \), tiene que ser \( \xi<\nu \).

Ahora distinguimos las tres posibilidades para \( \nu \):

No puede ser \( \nu=\kappa \), pues entonces \( \xu\leq \xi \).

Tampoco puede ser que \( \nu \) sea el supremo de un conjunto cerrado para exp, pues entonces, como \( \xi<\nu \), sería \( \mu=2^\xi\leq \nu \).

Por lo tanto \( \nu=2^\pi \) para cierto \( \pi\in \Phi(\kappa) \), luego \( \pi<\nu \) y \( 2^\pi<2^\xi<2^\nu \), luego \( \pi<\xi<\nu=2^\pi \), en contra de la minimalidad de \( \nu \).
[cerrar]

h) Existe un único \( \mu\in \Phi(\kappa) \) tal que \( \exp\mu=\emptyset \) y, más concretamente, \( \mu \) es el máximo de \( \Phi(\kappa) \).

Demostración:
Si \( \forall \mu\in \Phi(\kappa)\ \exp\mu\neq \emptyset \), podríamos aplicar la propiedad c) a \( B=\Phi(\kappa) \) y concluir que \( \mu=\sup\phi(\kappa)\in \Phi(\kappa) \), pero entonces \( \mu<\exp\mu\in \Phi(\kappa) \), contradicción.

Basta probar que \( \mu \) es el máximo de \( \Phi(\kappa) \), pues entonces será obviamente el único con esta propiedad. Supongamos, pues, que existe \( \nu\in \Phi(\kappa) \) tal que \( \mu<\nu \). Podemos suponer que \( \nu \) es el mínimo posible.

No puede ser \( \nu=\kappa \), pues entonces \( \mu<\kappa \), en contra de f)

Tampoco puede ser que \( \nu \) sea el supremo de un subconjunto de \( \Phi(\kappa) \) cerrado para exp, pues entonces existiría un \( \xi \) en dicho conjunto tal que \( \mu<\xi<\nu \), pero como \( \exp\xi\neq\emptyset \), resulta que \( \mu<\xi\leq\kappa_1 \), luego \( \exp\mu\neq\emptyset \).

Por lo tanto \( \nu=\exp\xi \), para cierto \( \xi\in \Phi(\kappa), \xi<\nu \). Por la minimalidad de \( \nu \) tenemos que \( \xi\leq \mu<2^\xi \), pero esto contradice a g).
[cerrar]

Sólo necesitamos una última propiedad técnica:

i) \( \mu\in \Phi(\kappa)\leftrightarrow T(\mu)\in \Phi(T(\kappa)) \).

Demostración
Si existe un \( \mu\in \Phi(\kappa) \) tal que \( T(\mu)\notin \Phi(T(\kappa) \), podemos tomar el mínimo posible (aquí es importante que el conjunto cuyo mínimo estamos tomando está definido por una propiedad estratificada). Obviamente no puede ser \( \mu=\kappa \). Si \( \mu=2^\nu \), para un \( \nu\in \Phi(\kappa) \), por la minimalidad de \( \mu \) tenemos que \( T(\nu)\in \Phi(T(\kappa)) \), luego \( T(\mu)=2^{T(\nu)}=T(2^\nu)\in \Phi(T(\kappa)) \).

Si \( \mu=\sup B \), para cierto \( B\subset \Phi(\kappa) \) cerrado para exp, entonces \( C=\{T(\nu)\mid \nu\in B\}\subset \Phi(T(\kappa)) \) por la minimalidad de \( \mu \), y es fácil ver que \( T(\mu)=\sup C \). Esto implica que \( T(\mu)\in \Phi(T(\kappa)) \), contradicción. La implicación contraria se prueba igualmente.
[cerrar]

Puesto que \( \Phi(\kappa) \) tiene un máximo elemento, su ordinal será un ordinal sucesor. Esto justifica la definición siguiente:

Definición: Llamaremos \( \beta_0(\kappa)\equiv \text{ord}(\Phi(\kappa),\leq) \). Así cada ordinal \( \alpha\leq \beta_0(\kappa) \) es el ordinal de un segmento inicial de \( \Phi(\kappa) \), luego podemos definir

\( \beth_\alpha(\kappa)\equiv\mu\mid (\mu\in \Phi(\kappa)\land \text{ord}(\Phi(\kappa)^<_\mu,\leq)=\alpha). \)

Así \( \beth_\alpha(\kappa) \) es el \( \alpha \)-ésimo elemento de \( \Phi(\kappa) \). Notemos que el término \( \beth_\alpha(\kappa) \) está estratificado con tipo dos unidades menor que el tipo de \( \alpha \) y el mismo tipo que \( \kappa \). Por ello, técnicamente, la "función beth" no es una función, es decir, no podemos definir una función \( \beth(\kappa): \text{Ord}^<_{\beta_0(\kappa)}\longrightarrow K \), ya que para ello el tipo de \( \alpha \) tendría que ser el mismo que el de \( \beth_\alpha(\kappa) \).

Llamaremos \( \beta_0\equiv \beta_0(\aleph_0) \) y \( \beth_\alpha\equiv \beth_\alpha(\aleph_0) \).

Ahora probamos que la definició de \( \beth \) cumple lo que pretendíamos que cumpliera:

Teorema Para todo cardinal \( \kappa \) se cumple que \( \beth_0(\kappa)=\kappa \), si \( \alpha<\beta_0(\kappa) \) entonces \( \beth_{\alpha+1}(\kappa)=2^{\beth_\alpha(\kappa)} \) y si \( \lambda\leq \beth_0(\kappa) \) es un ordinal límite, entonces \( \beth_\lambda(\kappa)=\sup\{\beth_\delta(\kappa)\mid \delta<\lambda\} \).

Demostración:
\( \beth_0(\kappa) \) es el elemento de \( \Phi(\kappa) \) cuya sección inicial es vacía, es decir, su mínimo, que es \( \kappa \) por f).

\( \beth_\alpha(\kappa) \) y \( \beth_{\alpha+1}(\kappa) \) son los elementos de \( \Phi(\kappa) \) cuyas secciones iniciales tienen ordinal \( \alpha \) y \( \alpha+1 \) respectivamente, luego el segundo es el inmediato sucesor del primero en \( \Phi(\kappa) \), luego es \( 2^{\beth_\alpha(\kappa)} \) por g).

Si \( B=\{\beth_\delta(\kappa)\mid \delta<\lambda\} \), entonces \( \mu=\sup B\in \Phi(\kappa) \) por c) y la parte ya probada. Sea \( \alpha=\text{ord}(\Phi(\kappa)_\mu^<,\leq) \), de modo que \( \mu=\beth_\alpha(\kappa) \). Hay que probar que \( \alpha=\lambda \). Si \( \delta<\lambda \), entonces \( \beth_\delta(\kappa)<\mu \) determina un segmento de ordinal \( \delta \), necesariamente menor que \( \alpha \), luego \( \lambda\leq \alpha \). Si \( \delta<\alpha \), entonces existe un \( \nu<\mu \) cuyo segmento inicial tiene ordinal \( \delta \), pero entonces, por definición de \( \mu \), existe un \( \delta'<\lambda \) tal que \( \nu<\beth_{\delta'}(\kappa) \), luego \( \delta \) (el ordinal del segmento de \( \nu \)) es menor que \( \delta' \) (el ordinal del segmento de \( \beth_{\delta'}(\kappa) \)), luego \( \delta<\lambda \), con lo que \( \alpha\leq \lambda \).
[cerrar]

Necesitaremos un último resultado:

Teorema: Para todo cardinal \( \kappa \) y todo ordinal \( \alpha\leq \beta_0(\kappa) \) se cumple que \( T(\alpha)\leq \beta_0(T(\kappa)) \) y \( T(\beth_\alpha(\kappa))=\beth_{T(\alpha)}(T(\kappa)) \).

Demostración
Consideramos el conjunto

\( X=\{\alpha\mid \alpha\in \text{Ord}\land \alpha\leq \beta_0(\kappa)\land (\beta_0(T(\kappa))<T(\alpha)\lor
 \)

\(
(T(\alpha)\leq \beta_0(T(\kappa))\land T(\beth_\alpha(\kappa))\neq \beth_{T(\alpha)}(T(\kappa)))))\} \).

Basta probar que es vacío. En caso contrario tiene un mínimo elemento \( \alpha\leq \beta_0(\kappa) \), de modo que si \( \delta<\alpha \) entonces

\( T(\delta)\leq \beta_0(T(\kappa))\land T(\beth_\delta(\kappa))=\beth_{T(\delta)}(T(\kappa)) \).

Por i) tenemos que \( T(\beth_\alpha(\kappa))\in \Phi(T(\kappa)) \). Veamos que el conjunto

\( Y=\{\beth_{T(\delta)}(T(\kappa)\mid \delta<\alpha)\} \)

está formado por los elementos de \( \Phi(T(\kappa)) \) menores que \( T(\beth_\alpha(\kappa)) \). En efecto, si \( \mu\in \Phi(T(\kappa9) \) cumple \( \mu<T(\beth_\alpha(\kappa)) \), por i) tenemos que \( T^{-1}(\mu)\in \Phi(\kappa) \), luego \( T^{-1}(\mu)<\beth_\alpha(\kappa) \) y \( \mu=\beth_{T(\delta)}(T(\kappa)) \).

Así pues, basta probar qe el ordinal de \( Y=\phi(T(\kappa))^<_{T(\beth_\alpha(\kappa))} \) es \( T(\alpha) \). Para ello tomamos un conjunto bien ordenado tal que \( \text{ord}(A,\leq)=\alpha \) y definimos una aplicación \( f:\mathcal P_1A\longrightarrow Y \) mediante

\( f(\{a\})=\beth_{T(\text{ord}(A_a^<,\leq))}(T(\kappa)) \).

(Aquí es crucial que el miembro derecho y \( \{a\} \) tienen el mismo tipo.) Es claro que \( f \) es una semejanza, luego \( \text{ord}(Y,\leq)=\text{ord}(\mathcal P_1A,\leq)=T(\alpha) \), como había que probar.
[cerrar]

Para terminar observamos que si un cardinal cumple \( T(\kappa)=\kappa \) entonces \( \kappa\leq \kappa_1 \) y \( T(2^\kappa)=2^\kappa \). En efecto, \( \kappa\leq \kappa_0 \), luego \( \kappa= T(\kappa)\leq T(\kappa_0)=\kappa_1 \) y \( T(2^\kappa)=T(2)^{T(\kappa)}=2^\kappa \).

Por lo tanto, como \( T(\aleph_0)=\aleph_0 \), está definido \( \beth_1 \) y cumple \( T(\beth_1)=T(2^{\aleph_0})=2^{\aleph_0}=\beth_1 \), luego está definido \( \beth_2 \) y cumple \( T(\beth_2)=\beth_2 \) y, en general, tenemos definidos los cardinales

\( \beth_0<\beth_1<\beth_2<\beth_3<\cdots \),

lo cual no significa que \( \beth_n \) esté definido para todo \( n\in \mathbb{N} \).
Título: Existencia de átomos
Publicado por: Carlos Ivorra en 04 Febrero, 2012, 01:15 pm
Recordemos que los axiomas de NFA no afirman la existencia de átomos, sino que meramente permiten su existencia. En principio, podríamos haber evitado la distinción entre átomos y conjuntos y haber desarrollado toda la teoría bajo el supuesto de que todos los objetos son conjuntos. Esto es lo que hace la teoría original de Quine, NF.

Más aún, si examinamos todos los razonamientos que hemos hecho hasta ahora, no es que en ninguno de ellos se haya supuesto la existencia de átomos, sino que ni siquiera ha sido necesario contemplar explícitamente dicha posibilidad, es decir, nunca ha hecho falta distinguir casos como: "si \( x \) es un átomo razonamos así, y si es un conjunto razonamos de esta otra forma", o "si el conjunto \( X \) contiene átomos entonces pasa esto, mientras que si no contiene átomos pasa esto otro".

A la vista de lo visto, sería razonable pensar que tanto la existencia como la no existencia de átomos es consistente con los axiomas de NFA, y nada parece cambiar porque añadamos el axioma de elección. Sin embargo no es así. Ahora vamos a probar que el axioma de elección permite demostrar la existencia de átomos o, lo que es lo mismo, la teoría NF, en la que no existen átomos, es contradictoria con el axioma de elección. Respecto a la situación sin el axioma de elección, no se sabe cuál es: nadie ha construido nunca un modelo de NFA (necesariamente que no cumpla el axioma de elección) en el que no haya átomos, por lo que no se sabe si NF es consistente. En el caso de NFA + AE es posible construir modelos (con átomos) a partir de modelos de teorías más débiles que ZFC, por lo que NFA + AE es consistente si ZFC es consistente.

Mantenemos la notación del post anterior en el que hemos construido la función beth. Necesitamos unos pocos hechos adicionales:

a) Si \( \kappa\leq \kappa_1 \), entonces \( |\Phi(\kappa)|\geq 2 \).

En efecto, sabemos que si \( \kappa\leq \kappa_1 \) está definido \( \exp \kappa\neq \emptyset \), por lo que \( \kappa,\exp\kappa\in \Phi(\kappa) \).

b) Si \( \kappa_1<\kappa \), entonces \( |\Phi(\kappa)|=1 \).

En efecto, se cumple que \( \Phi(\kappa)=\{\kappa\} \), pues \( \exp\kappa=\emptyset \) y basta tomar \( A=\{\kappa\} \) en la propiedad d) del post anterior.

c) Si \( \kappa\leq\kappa_1 \), entonces \( |\Phi(\kappa)|=|\Phi(\exp \kappa)|+1 \).

Aplicamos dos veces la propiedad d) del post anterior para concluir que \( \Phi(\kappa)=\{\kappa\}\cup \Phi(\exp\kappa) \). Tomando \( A=\{\kappa\}\cup \Phi(\exp\kappa) \) concluimos que \( \Phi(\kappa)\subset \{\kappa\}\cup\Phi(\exp\kappa) \) y con \( A=\Phi(\kappa) \) obtenemos que \( \Phi(\exp\kappa)\subset \Phi(\kappa) \). Además \( \kappa\notin \Phi(\exp\kappa) \), pues en caso contrario sería \( \exp\kappa\leq\kappa \).

d) Si \( \Phi(T(\kappa)) \) es finito, entonces \( \Phi(\kappa) \) es finito.

Basta considerar \( f:\mathcal P_1\Phi(\kappa)\longrightarrow \Phi(T(\kappa)) \) dada por \( f(\{\mu\})=T(\mu) \). La definición es correcta porque está estratificada y por la propiedad i) del post anterior. Obviamente es inyectiva, luego \( |\mathcal P_1\Phi(\kappa)| = T(|\Phi(\kappa)|) \) es finito, luego \( |\Phi(\kappa)| \) también es finito.

Ahora ya podemos demostrar el teorema principal:

Teorema (Specker) \( |\mathcal PV|<|V| \)

Demostración:

En general se cumple que \( |\mathcal PV| \leq |V| \). Supongamos que \( |\mathcal PV| =|V| \), es decir, que \( \exp \kappa_1 = \kappa_0 \) (recordemos que \( |\mathcal PV| = 2^{T(|V|)}=2^{\kappa_1}=\exp\kappa_1 \)).

\( \bullet \) Si \( \kappa_1<\kappa \) entonces \( |\Phi(T(\kappa))|=2 \) o \( 3 \).

En efecto, tenemos que \( T(\kappa_1)<T(\kappa)\leq T(\kappa_0)=\kappa_1 \), luego

\( \exp T(\kappa)\geq \exp T(\kappa_1) = T(\exp\kappa_1)=T(\kappa_0)=\kappa_1 \).

Si \( \exp T(\kappa)=\kappa_1 \), entonces es fácil ver que \( \Phi(T(\kappa))=\{T(\kappa),\kappa_1,\kappa_0\} \).

Si \( \exp T(\kappa)>\kappa_1 \), entonces \( \Phi(T(\kappa))=\{T(\kappa),\exp T(\kappa)\} \).

\( \bullet \) Si \( \Phi(\kappa) \) es finito, entonces \( |\Phi(T(\kappa))|=T(|\Phi(\kappa)|)+1 \) o \( |\Phi(T(\kappa))|=T(|\Phi(\kappa)|)+2 \).

En efecto, esta afirmación está estratificada, luego podemos considerar el conjunto de todos los cardinales \( \kappa \) que no la cumplen. Si existiera alguno, podríamos tomar uno de ellos con \( |\Phi(\kappa)| \) mínimo.

Si \( |\Phi(\kappa)|=1 \), necesariamente \( \exp \kappa=\emptyset \), luego \( \kappa_1<\kappa \), luego \( |\Phi(T(\kappa))|=2 \) o \( 3 \) por el apartado anterior. Como \( T(|\Phi(\kappa)|)=T(1)=1 \), resulta que \( \kappa \) no es un contraejemplo y tenemos una contradicción.

Si \( |\Phi(\kappa)|>1 \), entonces \( \kappa\leq \kappa_1 \) por b), y hemos visto que \( |\Phi(\kappa)|=|\Phi(\exp\kappa)|+1 \), luego \( |\Phi(\exp\kappa)|<|\Phi(\kappa)| \), porque son cardinales finitos. Por la minimalidad de \( |\Phi(\kappa)| \) tenemos que

\( |\Phi(T(\exp\kappa))|=T(|\Phi(\exp\kappa)|)+1 \) o \( |\Phi(T(\exp\kappa))|=T(|\Phi(\exp\kappa)|)+2 \)

y por consiguiente

\( |\Phi(T(\kappa))|=|\Phi(\exp T(\kappa))|+1 = |\Phi(T(\exp\kappa))|+1 = T(|\phi(\exp\kappa)|)+2 \text{ o } 3 \)

\(  = T(\Phi(\kappa))+ 1\text{ o }2 \),

luego \( \kappa \) tampoco es un contraejemplo al enunciado, contradicción.

\( \bullet \) Sea \( A = \{\kappa\mid \kappa\in K\land \Phi(\kappa)\text{ es finito}\} \).

Tenemos que \( A \) no es vacío, pues \( \Phi(\kappa_0)=\{\kappa_0\} \), luego podemos tomar su mínimo elemento \( \kappa \). Como \( \Phi(\kappa) \) es finito, también lo es el cardinal \( T(|\Phi(\kappa)|) \) (porque T transforma cardinales finitos en cardinales finitos), y acabamos de probar que entonces \( \Phi(T(\kappa)) \) también es finito. Por la minimalidad de \( \kappa \) tiene que ser \( \kappa\leq T(\kappa)\leq T(\kappa_0)=\kappa_1 \), luego \( \mu = T^{-1}(\kappa)\leq \kappa \).

Como \( \Phi(T(\mu))=\Phi(\kappa) \) es finito, la propiedad d) nos da que \( \Phi(\mu) \) es finito, luego \( \kappa\leq \mu \) por la minimalidad de \( \kappa \). En definitiva, \( \kappa=\mu \), luego la última afirmación que hemos probado se reduce a que \( |\Phi(\kappa)| =|T(\Phi(\kappa))|+1\text{ o }2 \).

En definitiva, hemos encontrado un número natural \( n=|\Phi(\kappa)| \) tal que \( n=T(n)+1 \) o bien \( n=T(n)+2 \), pero esto es imposible, ya que, como T conserva sumas y productos y \( T(1)=1 \), se cumple que \( n \) es par si y sólo si \( T(n) \) es par, luego no puede darse la igualdad  \( n=T(n)+1 \). La otra alternativa tampoco es posible, porque \( n \) y  \( T(n) \) son congruentes módulo \( 3 \). Esta contradicción termina la prueba del teorema.


Ahora la existencia de átomos es inmediata. Observemos que \( \mathcal PV \) es el conjunto \( C \) de todos los conjuntos, luego si definimos \( A \) como el conjunto de todos los átomos, tenemos que \( V=\mathcal PV+A \), donde la unión es disjunta. Como \( \mathcal PV \) es infinito, el teorema anterior implica que \( |V|=|\mathcal PV|+|A|=|A| \), luego el teorema anterior equivale a que \( |\mathcal PV|<|A| \).

Así pues, no es que existan átomos, sino que existen más átomos que conjuntos, lo cual es bastante curioso, dado que cada átomo \( a \) permite formar al menos un conjunto \( \{a\} \).
Título: Limitaciones de NFA
Publicado por: Carlos Ivorra en 12 Febrero, 2012, 05:14 pm
En NFA hemos demostrado los axiomas de Peano y, en particular, el principio de inducción:

\( \forall X(X\subset \mathbb{N}\land 0\in X\land \forall n\in X\ n+1\in X\rightarrow X=\mathbb{N}) \).

Sin embargo, no todas las demostraciones por inducción que podemos considerar "razonables" son formalizables en NFA. Veamos un ejemplo sencillo. El "teorema" siguiente no es demostrable en NFA:

Si A es un conjunto de cardinal \( m\in \mathbb{N} \) cuyos elementos son conjuntos disjuntos dos a dos de cardinal \( n\in \mathbb{N} \), entonces \( |\bigcup A|=mn \).

Para demostrar esto, una forma razonable de abordar el problema sería razonar por inducción sobre m. Con precisión, esto significa considerar la fórmula

\( \phi(m)\equiv m\in \mathbb{N}\land \forall A(|A|=m\land \forall x\in A\ (\text{cto}\,x\land |x|=n\land{} \)

\( \forall xy\in A(x\neq y\rightarrow x\cap y=\emptyset)\rightarrow |\bigcup A|=mn) \).

No hay ninguna dificultad en demostrar em NFA que se cumple \( \phi(0)\land \forall m\in \mathbb{N}\ (\phi(m)\rightarrow \phi(m+1)) \). Sin embargo, de aquí no podemos concluir que \( \forall m\in \mathbb{N}\ \phi(m) \).

En efecto, para poder aplicar el principio de inducción necesitamos definir el conjunto

\( X=\{m\mid \phi(m)\} \),

y no podemos garantizar la existencia de este conjunto porque la fórmula \( \phi \) no está estratificada. En efecto, si intentamos estratificarla obtenemos:

\( m_1\in \mathbb{N}_2\land \forall A_1(|A_1|_2=m_2\land \forall x_0\in A_1\ (\text{cto}\,x_0\land |x_0|_1=n_1\land{} \)

\( \forall x_0y_0\in A_1(x_0\neq y_0\rightarrow x_0\cap y_0=\emptyset_0)\rightarrow |\bigcup A_1|_1=m_1n_1) \).

Vemos que la primera aparición de la variable \( m \) está forzada a tener tipo 2 si \( A \) tiene tipo 1, mientras que, en su segunda aparición, el tipo 1 de la variable \( A \) hace que \( \bigcup A \) tenga tipo 0, luego su cardinal tiene tipo 1, luego \( m \) y \( n \) han de tener tipo 1 para que su producto \( mn \) tenga también tipo 1. Así pues, como no podemos asignar un único tipo a \( m \), concluimos que \( \phi \) no puede estratificarse.

En general, para demostrar por inducción en NFA que todos los números naturales cumplen una propiedad \( \phi(m) \) necesitamos que la fórmula \( \phi \) esté estratificada.

Si alguien intenta demostrar el "teorema" de una forma más directa (por ejemplo, construyendo una biyección entre \( \bigcup A \) y el producto cartesiano de \( A \) y un conjunto de cardinal \( n \)), se encontrará inevitablemente con problemas relacionados con la estratificación que le impedirán concluir la prueba.

Veamos un caso más flagrante y llamativo:

Para cada número natural \( n \) podemos definir \( I_n=\{m\mid m\in \mathbb{N}\land 1\leq m\leq n\} \) o, más informalmente, \( I_n=\{1,\ldots, n\} \). Parece que la respuesta a "¿cuál es el cardinal de \( I_n \)?" es obvia y, sin embargo, no es posible demostrar en NFA que \( \forall n\in \mathbb{N}\ |I_n|=n \).

Podemos demostrar que \( I_0 = \emptyset \) y que \( \forall n\in \mathbb{N}\ I_{n+1}=I_n\cup \{n+1\} \), de donde se sigue inmediatamente que \( |I_0|=0 \) y que \( \forall n\in \mathbb{N}\ |I_{n+1}|=|I_n|+1 \) o, incluso:

\( |I_0|=0\land \forall n\in \mathbb{N}\ (|I_n|=n\rightarrow |I_{n+1}|=n+1) \).

Sin embargo, esto no permite concluir que \( \forall n\in \mathbb{N}\ |I_n|=n \). Nuevamente, para que la inducción fuera válida en NFA seria necesario contar con el conjunto

\( X=\{n\mid n\in \mathbb{N}\land |I_n|=n\} \),

pero no podemos asegurar la existencia de este conjunto ya que la fórmula \( |I_n|=n \) no está estratificada. Si asignamos a la variable \( n \) el tipo 0, por ejemplo, nos vemos obligados a asignar tipo 1 al término \( I_n \) y tipo 2 a su cardinal, luego no podemos igualar este cardinal a \( n \) sin violar la estratificación.

Las restricciones de estratificación no limitan únicamente los razonamientos por inducción. Por ejemplo, si tenemos una relación de equivalencia \( R \) en un conjunto \( X \), podemos definir el conjunto cociente \( X/R \), pero no podemos definir la proyección canónica \( p: X\longrightarrow X/R \) dada por \( p(x) = [x]_R \), pues el tipo de la clase de equivalencia es una unidad superior al tipo de \( x \), mientras que para definir una aplicación a partir del teorema de formación de conjuntos necesitamos que en la definición \( x \) y \( p(x) \) tengan el mismo tipo.

Una ligera variante de este inconveniente es que si, por ejemplo, construimos \( \mathbb{Z} \) de la forma habitual, como un cociente de \( \mathbb{N}\times \mathbb{N} \) respecto de una relación de equivalencia, no podemos definir la aplicación \( i:\mathbb{N}\longrightarrow \mathbb{Z} \) que identifica a cada número natural con un número entero, y lo mismo pasa al construir \( \mathbb{Q} \) a partir de \( \mathbb{Z} \), etc.

Similarmente, no sólo no podemos demostrar que todo conjunto \( X \) cumple \( |X|<|\mathcal PX| \), sino que ni siquiera podemos demostrar que \( |X|\leq |\mathcal PX| \), pues en el mensaje anterior hemos visto que \( |\mathcal PV|<|V| \). En particular, esto significa que no existe la aplicación \( V\longrightarrow V \) dada por \( x\mapsto \{x\} \), lo cual no es contradictorio, ya que la definición no está estratificada y nada nos asegura que tenga que existir.

Estos hechos hacen que NFA resulte inviable como alternativa a ZFC como base del razonamiento matemático, es decir, como teoría marco en la que formalizar todas las matemáticas usuales (donde el matiz "usuales" pretende excluir los resultados que requieren hipótesis adicionales como la hipótesis del continuo, etc.). Ahora bien, sucede que NFA puede extenderse con axiomas que relajen parcialmente los requisitos de estratificación, de modo que los problemas que acabamos de mostrar se resuelvan satisfactoriamente (o, por lo menos, de forma que no sea descabellado sentirse satisfecho con la solución, aunque ahí ya entran los gustos de cada cual).

El más básico de estos axiomas es el llamado axioma de cómputo, (AC), propuesto originalmente por Rosser, y que es precisamente la afirmación \( \forall n\in \mathbb{N}\ |I_n|=n \). Aunque aparentemente así resolvamos sólo uno de los muchos problemas que podemos encontrarnos al tratar de usar NFA como "teoría de conjuntos básica" mediante un remedio ad hoc, sucede que esta afirmación tiene muchas consecuencias que bastan para convertir a NFA en una teoría "manejable". (De hecho, todos los problemas que hemos puesto como ejemplo en este mensaje se resuelven con AC). No obstante, hay otros axiomas que nos proporcionan un mayor grado de lo que Holmes llama "subversión" (de la estratificación), es decir que nos permiten definir conjuntos mediante fórmulas no estratificadas (siempre que se cumpla alguna otra condición, pues si elimináramos completamente las restricciones de estratificación estaríamos abriendo la puerta a todas las paradojas clásicas de la teoría de conjuntos).

Algunos de estos axiomas adicionales (entre ellos AC) tienen la propiedad de que, al añadirlos a NFA, la teoría que obtenemos sigue siendo más débil que ZFC, en el sentido de que puede probarse que si ZFC es consistente, entonces NFA + AC es consistente. Otros dan lugar a teorías más fuertes que ZFC, en el sentido de que puede probarse que si NFA más el axioma en cuestión es consistente, entonces también lo es ZFC, mientras que el recíproco no puede probarse.

Notemos que no es evidente cómo sacar partido al aparentemente modesto axioma de cómputo. Por ello dedicaremos el próximo mensaje a introducir (sin salir de NFA) algunos conceptos que serán útiles para expresar posibles violaciones de la estratificación consistentes con la teoría, y en el mensaje siguiente usaremos AC para demostrar algunos resultados de subversión expresados en términos de los conceptos estudiados previamente.

La idea básica es que en el próximo mensaje definiremos los llamados conjuntos cantorianos y conjuntos fuertemente cantorianos, que vienen a ser un análogo a los conjuntos que pueden definirse en ZFC, de tal forma que relajar las condiciones de estratificación sobre ellos no da lugar a contradicciones. Así, por ejemplo, probaremos que \( \mathbb{N} \) es un conjunto cantoriano, mientras que \( V \) no lo es. Por eso podemos tomarnos ciertas libertades con \( \mathbb{N} \) que no podríamos tomarnos con \( V \) sin caer en contradicciones.
Título: Conjuntos cantorianos
Publicado por: Carlos Ivorra en 19 Febrero, 2012, 04:11 pm
Recordemos que la versión del teorema de Cantor que puede probarse en NFA es la que afirma que \( |\mathcal P_1X|<|\mathcal PX| \), donde \( \mathcal P_1X = \{\{x\}\mid x\in X\} \). Recordemos también que tenemos definido un operador \( T \) sobre los cardinales de modo que \( T(|X|) = |\mathcal P_1X| \). Puesto que la exponenciación de cardinales ha sido definida de modo que \( |\mathcal PX| = T(2^{|X|}) \), resulta que el teorema de Cantor en términos de la exponencial es idéntico al de ZFC: \( \kappa<2^\kappa \).

Lo "razonable" sería que \( |\mathcal P_1X| = |X| \) o, equivalentemente, que el operador \( T \) fuera la identidad. Sin embargo, hemos visto que \( T(|V|)<|V| \). Ahora bien, sucede que estas patologías sólo se dan (o, al menos, es consistente que sólo se den) en los conjuntos "extraños" (para alguien familiarizado con ZFC). Para precisar esta idea introducimos la definición siguiente:

Definición: Un conjunto \( X \)  es cantoriano si \( |\mathcal P_1X| = |X| \) y es fuertemente cantoriano si existe una aplicación \( f: X\longrightarrow \mathcal P_1X \) tal que \( \forall x\in X\ f(x)=\{x\} \).

Observemos que a partir de los axiomas de NFA no podemos asegurar en general que exista la aplicación que aparece en la definición de conjunto fuertemente cantoriano, pues la fórmula que la define como conjunto no está estratificada, pero el hecho de que el axioma de formación de conjuntos no asegure su existencia no significa que no se pueda probar su existencia de otro modo o, al menos, que no sea consistente que exista. En algunos casos no lo será, pero en otros puede serlo.

Desde luego, si existe la aplicación de la definición de conjunto fuertemente cantoriano, es claramente biyectiva, luego todo conjunto fuertemente cantoriano es cantoriano.

Observamos a continuación que estos dos conceptos dependen únicamente del cardinal de un conjunto, pues si dos conjuntos \( X \) e \( Y \) tienen el mismo cardinal, existe una biyección \( g: X\longrightarrow Y \), la cual induce una biyección \( \bar g:\mathcal P_1X\longrightarrow \mathcal P_1Y \) dada por \( \bar g(\{x\})=\{g(x)\} \) (notemos que la definición de \( \bar g \) está estratificada), luego si \( X \) es cantoriano, tenemos que \( |Y| = |X| = |\mathcal P_1X| = |\mathcal P_1Y| \), luego \( Y \) también es cantoriano.

Igualmente, si \( X \) es fuertemente cantoriano y consideramos la biyección \( f: X\longrightarrow \mathcal P_1X \) dada por la definición, sucede que la composición \( \bar g\circ f\circ g^{-1} \) es precisamente la aplicación \( y\mapsto \{y\} \), luego \( Y \) es también fuertemente cantoriano.

Esto da sentido a la definición siguiente:

Definición: Un cardinal \( \kappa \) es cantoriano (resp. fuertemente cantoriano) si los conjuntos de cardinal \( \kappa \) son cantorianos (resp. fuertemente cantorianos).

Teniendo en cuenta la definición del operador \( T \), es evidente que un cardinal es cantoriano si y sólo si \( T(\kappa)=\kappa \).

Alternativamente, un conjunto \( X \) es cantoriano si y sólo si cumple el teorema de Cantor en su versión de ZFC, es decir, si y sólo si \( |X|<|\mathcal PX| \).

Recordemos que tenemos definido también un operador \( T \) sobre ordinales, de modo que si \( \alpha = \mbox{ord}(A,\leq) \), entonces \( T(\alpha) \) es el ordinal del conjunto \( \mathcal P_1A \) con el orden dado por \( \{x\}\leq_1\{y\}\leftrightarrow x\leq y \).

Diremos que un ordinal es cantoriano si cumple \( T(\alpha) = \alpha \).

La relación \( \text{card}(T(\alpha))=T(\text{card}(\alpha)) \) implica que si \( \alpha \) es un ordinal cantoriano entonces \( \mbox{card}(\alpha) \) es un cardinal cantoriano, aunque el recíproco no es necesariamente cierto.

Con más detalle: si \( \alpha = \mbox{ord}(A,\leq) \), entonces \( \text{card}(\alpha) = |A| \). Si \( \alpha \) es cantoriano entonces el conjunto bien ordenado \( (\mathcal P_1A,\leq_1) \) es semejante a \( (A,\leq) \), luego en particular \( \mathcal P_1A \) tiene el mismo cardinal que \( A \), luego \( A \) es cantoriano y \( \text{card}(\alpha) \) también. Pero la existencia de una biyección entre \( A \) y \( \mathcal P_1A \) no implica que dicha biyección sea una semejanza.

Diremos que un ordinal \( \alpha \) es fuertemente cantoriano si \( \text{card}(\alpha) \) es un cardinal fuertemente cantoriano.

En tal caso, si \( \alpha = \text{ord}(A,\leq) \) tenemos una biyección \( A\longrightarrow \mathcal P_1A \) dada por \( x\mapsto \{x\} \), y esta biyección concreta sí que es una semejanza \( (A,\leq)\longrightarrow (\mathcal P_1A,\leq_1) \), por lo que \( T(\alpha)=\alpha \) y \( \alpha \) es cantoriano.

Veamos algunas propiedades:

Teorema: Se cumple:

a) Todo subconjunto de un conjunto fuertemente cantoriano es fuertemente cantoriano.

b) Si \( \kappa \) es un cardinal fuertemente cantoriano, todo cardinal \( \mu\leq \kappa \) es fuertemente cantoriano.

c) Si \( \alpha \) es un ordinal fuertemente cantoriano, todo ordinal \( \beta\leq\alpha \) es fuertemente cantoriano.

d) Si \( X \) es un conjunto (fuertemente) cantoriano, \( \mathcal PX \) también lo es.

e) Si \( \kappa \) es un cardinal (fuertemente) cantoriano, \( 2^\kappa \) también lo es.

f)  Si \( \kappa \) es un cardinal (fuertemente) cantoriano, \( \kappa^+ \) también lo es.


Demostración
a) Si \( X \) es fuertemente cantoriano e \( Y\subset X \), existe \( f: X\longrightarrow \mathcal P_1X \) tal que \( f(x)=\{x\} \), y es claro que \( f|_Y \) cumple la definición para \( Y \).

b) es consecuencia inmediata de a), y a su vez c) es consecuencia de b).

d) Si \( X \) es cantoriano, entonces \( T(|X|)=|X| \), luego

\( T(|\mathcal PX|) = T(2^{T(|X|)} = T(2^{|X|})=|\mathcal PX| \),

luego \( \mathcal PX \) es cantoriano.

Si \( X \) es fuertemente cantoriano, sea \( f: X\longrightarrow \mathcal P_1X \) según la definición y sea \( \bar f: \mathcal PX\longrightarrow \mathcal P\mathcal P_1X \) la aplicación dada por \( \bar f(A) = f[A] \), claramente bien definida, y sea \( g_ \mahtcal P\mathcal P_1X\longrightarrow \mathcal P_1\mathcal PX \) la biyección dada por

\( g(A)=\{\bigcup A\} \),

que también está definida, pues la unión baja una unidad el tipo y \( \{\} \) lo vuelve a subir una unidad. Es fácil ver que \( g\circ \bar f: \mathcal PX\longrightarrow \mathcal P_1\mathcal PX \) viene dada por \( A\mapsto \{A\} \), luego \( \mathcal PX \) es fuertemente cantoriano.

e) Se sigue inmediatamente de d). Notemos no obstante que si \( \kappa \) es cantoriano, como \( \kappa\leq\kappa_0 \), tenemos que \( \kappa=T(\kappa)\leq T(\kappa_0)=\kappa_1 \), y ésta es la condición para que la exponencial \( 2^\kappa \) esté definida.

f) Observemos que si \( \kappa \) es cantoriano entonces \( \kappa<\kappa_0, \) pues ciertamente \( \kappa_0 \) no es cantoriano, luego está definido \( \kappa^+. \) Entonces \( T(\kappa^+)=T(\kappa)^+=\kappa^+, \) luego \( \kappa^+ \) es cantoriano.

Si \( \kappa \) es fuertemente cantoriano, entonces \(  \kappa^+\leq 2^\kappa \) y, como \( 2^k \) es fuertemente cantoriano por e), concluimos que \( \kappa^+ \) también lo es por b).
[cerrar]

Los recíprocos de todas las afirmaciones del teorema anterior son ciertas para conjuntos, cardinales y ordinales fuertemente cantorianos, pero en realidad se cumple algo más fuerte:

Teorema: Si \( \alpha \) es un ordinal tal que todo ordinal \( \delta<\alpha \) es cantoriano, entonces \( \alpha \) es fuertemente cantoriano, luego todo ordinal \( \delta<\alpha \) es fuertemente cantoriano.

Demostración
Podemos considerar la aplicación\(  f:\text{Ord}_\alpha^<\longrightarrow \mathcal P_1(\text{Ord}_\alpha^<) \) dada por \( f(\delta)=\{T^{-1}(\delta)\} \) (pues la definición está estratificada), pero por hipótesis no es sino \( \delta\mapsto \{\delta\} \), luego el conjunto \( \text{Ord}_\alpha^< \) es fuertemente cantoriano, luego \( |\text{Ord}_\alpha^<| = \text{card}(T^2(\alpha)) \) es un cardinal fuertemente cantoriano, luego \( T^2(\alpha) \) es un ordinal fuertemente cantoriano, luego cantoriano, luego \( T^3(\alpha)=T^2(\alpha) \), luego \( T(\alpha)=\alpha \), luego \( \alpha = T^2(\alpha) \) es fuertemente cantoriano.
[cerrar]

Ejemplo: Consideremos un conjunto con tres elementos, como \( X=\{0,1,2\} \). Entonces podemos definir

\( f=\{(0,\{0\}),(1,\{1\}), (2,\{2\})\} \)

y claramente \( f: X\longrightarrow \mathcal P_1X \) cumple \( \forall x\in X\ f(x)=\{x\} \), luego \( X \) es un conjunto fuertemente cantoriano y \( 3 \) es un cardinal fuertemente cantoriano.

Del mismo modo se puede probar que los cardinales \( 0, 1, 2, 3, \ldots \) son fuertemente cantorianos, lo cual no significa que podamos probar que todos los números naturales son cardinales fuertemente cantorianos, ni siquiera cantorianos.

Puesto que hemos probado que \( T(\aleph_\alpha) = \aleph_{T(\alpha)} \), es evidente que si \( \alpha \) es un ordinal cantoriano entonces \( \aleph_\alpha \) es un cardinal cantoriano. En particular podemos asegurar que los cardinales \( \aleph_0, \aleph_1, \aleph_2, \ldots \) son cantorianos, lo cual no significa que \( \aleph_n \) sea cantoriano para todo número natural \( n \).

Similarmente, también sabemos que \( T(\beth_\alpha) = \beth_{T(\alpha)} \), por lo que si \( \alpha \) es un ordinal cantoriano entonces \( \beth_\alpha \) es un cardinal cantoriano, luego \( \beth_0=2^{\aleph_0}, \beth_1, \beth_2\ldots \) son cardinales cantorianos, lo cual no significa que todo \( \beth_n \) lo sea.

También se cumple que \( T(\omega_\alpha)=\omega_{T(\alpha)} \), luego si \( \alpha \) es un ordinal cantoriano, también lo es \( \omega_\alpha \). En particular, los ordinales \( \omega=\omega_0, \omega_1, \omega_2,\ldots \) son cantorianos.

Sin embargo, como veremos, la propiedad realmente útil es la de ser fuertemente cantoriano, y en NFA no podemos probar siquiera que el cardinal \( \aleph_0 \) sea fuertemente cantoriano (ni que todos los números naturales lo sean). Ahí es donde entra en juego el axioma de cómputo que trataremos en el mensaje siguiente. De momento terminamos con una observación:

Teorema: No existe ningún conjunto cuyos elementos sean los conjuntos (resp. ordinales, cardinales) cantorianos.

Demostración
Si existiera el conjunto de los cardinales cantorianos, también existiría su complementario, el conjunto \( N \) de los cardinales no cantorianos, y habría un mínimo cardinal no cantoriano \( \kappa \). Si \( T(\kappa)<\kappa \), entonces \( T(\kappa) \) es cantoriano, luego \( T^2(\kappa)=T(\kappa) \), luego \( T(\kappa)=\kappa \) y \( \kappa \) es cantoriano, contradicción. Si \( \kappa<T(\kappa) \) entonces \( T^{-1}(\kappa)<\kappa \), luego \( T^{-1}(\kappa) \) es cantoriano, luego \( \kappa=T(\kappa) \) y \( \kappa \) es cantoriano, contradicción.

Para ordinales se razona análogamente. Si existiera el conjunto \( C \) de los conjuntos cantorianos, la aplicación \( \mathcal P_1C\longrightarrow K \) dada por\(  \{X\}\mapsto |X| \) tendría por imagen al conjunto de los cardinales cantorianos, y acabamos de probar que no existe tal conjunto.
[cerrar]
Título: Re: NFA
Publicado por: Raúl Aparicio Bustillo en 24 Febrero, 2012, 11:06 am
 
Citar
sucede que NFA puede extenderse con axiomas que relajen parcialmente los requisitos de estratificación

Con axiomas adicionales conseguimos demostrar que existen "más" conjuntos definibles con fórmulas no estratificadas, eso imagino que resulta útil. Ahora, si mantenemos los axiomas que ya tenemos, está muy claro que ya tenemos colecciones perfectamente definibles que no existen como conjuntos en NFA (los conjuntos cantorianos, por ejemplo), la aplicación \(  x\longrightarrow{\color{red}\{x\}}  \) con dominio el conjunto de todos los átomos. No están estratificadas sus definiciones, no hay el menor síntoma de inconsistencia en la teoría, pero resulta bastante chocante que el hecho de considerar que las definiciones estratificadas definan conjuntos ya te obliga a que ciertas definiciones no estratificadas no sean conjuntos.
Título: Re: NFA
Publicado por: Carlos Ivorra en 24 Febrero, 2012, 03:05 pm
Hola. Te he retocado el LaTeX, porque si no, no se entendía.

Con axiomas adicionales conseguimos demostrar que existen "más" conjuntos definibles con fórmulas no estratificadas, eso imagino que resulta útil.

Poco menos que imprescindible.

Ahora, si mantenemos los axiomas que ya tenemos, está muy claro que ya tenemos colecciones perfectamente definibles que no existen como conjuntos en NFA (los conjuntos cantorianos, por ejemplo),

Supongo que te refieres a la colección de todos los conjuntos cantorianos. En efecto, así es.

la aplicación \(  x\longrightarrow{\color{red}\{x\}}  \) con dominio el conjunto de todos los átomos. No están estratificadas sus definiciones, no hay el menor síntoma de inconsistencia en la teoría,

No es que no haya síntomas de inconsistencia, sino que sabemos que la teoría es "más consistente" que ZFC, en el sentido de que la consistencia de ZFC implica la de NFA, pero no al revés, podría ocurrir que NFA fuera consistente y ZFC contradictorio, pero nunca al revés.

pero resulta bastante chocante que el hecho de considerar que las definiciones estratificadas definan conjuntos ya te obliga a que ciertas definiciones no estratificadas no sean conjuntos.

No creo que sea tan raro. Eso lo sabes desde el primer momento: la estratificación está puesta para evitar ante todo que se pueda probar que la colección de todos los conjuntos que no se pertenecen a sí mismos sea un conjunto, y de hecho podemos probar que no lo es porque de serlo sería paradójico. Son las paradojas las que permiten probar que determinadas colecciones no son conjuntos. Y eso será así en cualquier teoría de conjuntos.
Título: Re: NFA
Publicado por: Raúl Aparicio Bustillo en 24 Febrero, 2012, 09:45 pm
Citar
La estratificación está puesta para evitar ante todo que se pueda probar que la colección de todos los conjuntos que no se pertenecen a sí mismos sea un conjunto

Pero da la sensación de que la restricción es demasiado fuerte, desde luego la estratificación evita la circularidad en las fórmulas atómicas con el relator \(  \in{}  \). Una circularidad en la que para ver si un objeto pertenece a un conjunto haya que mirar precisamente dicho hecho, como ocurre en la paradoja de Russell ( y sospecho que en muchas de las otras paradojas conjuntistas) no es aceptable . Pero ese tipo de circularidad ya se evita exigiendo que sean de tipos diferentes los 2 términos relacionados, no veo la necesidad de que sean tipos consecutivos y de que se mantengan los tipos de cada variable cuando se forma una fórmula compuesta a partir de 2 atómicas diferentes del tipo \(  a \in{b}  \).

Tampoco termino de ver la necesidad de igualar el tipo de las variables en relaciones con \(  =   \), y en las componentes del par ordenado. Aunque desde luego esta última no sé si necesaria pero es al menos una forma de evitar la inconsistencia en el momento en que la teoría hay conjuntos no cantorianos: no veo otra forma de prohibir una biyección de la forma \(  x\longrightarrow{\{x\}}  \) sobre un dominio que sea no cantoriano,  y tengo la sospecha que el "capricho" de tener conjunto universal en nuestra teoría y complementario de cada conjunto con respecto al universo también, hace que efectivamente tenga que haber conjuntos no cantorianos (este último parrafo tan sólo son conjeturas)
Título: Re: NFA
Publicado por: Carlos Ivorra en 24 Febrero, 2012, 09:57 pm
Pero ese tipo de circularidad ya se evita exigiendo que sean de tipos diferentes los 2 términos relacionados, no veo la necesidad de que sean tipos consecutivos y de que se mantengan los tipos de cada variable cuando se forma una fórmula compuesta a partir de 2 atómicas diferentes del tipo \(  a \in{b}  \).

Tampoco termino de ver la necesidad de igualar el tipo de las variables en relaciones con \(  =   \),

Sin esas condiciones podrías expresar \( x\notin x \) en la forma \( \exists y (y\notin x\land y = x) \) y la fórmula estaría estratificada (si es que admites que los tipos en \( y = x \) no necesiten ser iguales, o que cada variable pueda tener un tipo diferente en cada subfórmula).

y en las componentes del par ordenado.

Eso no es una restricción, en el sentido de una limitación, sino todo lo contrario, algo que te da más posibilidades que si no dispusieras de ello.

Aunque desde luego esta última no sé si necesaria pero es al menos una forma de evitar la inconsistencia en el momento en que la teoría hay conjuntos no cantorianos: no veo otra forma de prohibir una biyección de la forma \(  x\longrightarrow{\{x\}}  \) sobre un dominio que sea no cantoriano,  y tengo la sospecha que el "capricho" de tener conjunto universal en nuestra teoría y complementario de cada conjunto con respecto al universo también, hace que efectivamente tenga que haber conjuntos no cantorianos (este último parrafo tan sólo son conjeturas)

Claro. El conjunto \( V \) no puede ser cantoriano, pues si lo fuera llevaría a una contradicción al aplicarle el argumento del teorema de Cantor.
Título: Re: NFA
Publicado por: Raúl Aparicio Bustillo en 25 Febrero, 2012, 12:05 am
Citar
Sin esas condiciones podrías expresar \(  x\notin x  \) en la forma \(  \exists y (y\notin x\land y = x)  \)y la fórmula estaría estratificada (si es que admites que los tipos en [y = x] no necesiten ser iguales, o que cada variable pueda tener un tipo diferente en cada subfórmula).

Pues es verdad. Pero ¿la estratificación es la condición mínima que hay que exigir a una fórmula para que no encierre algún tipo de circularidad, en el caso de fórmulas construidas mediante conectivas lógicas a partir de relaciones \(  \in{}]  \) e =  ?

Lo que no termino de ver es cuando dices refiriendote a la estratificación de los pares ordenados:

Citar
Eso no es una restricción, en el sentido de una limitación, sino todo lo contrario, algo que te da más posibilidades que si no dispusieras de ello.

La no existencia de la función singlete \(  x\longrightarrow{{x}}  \)  en conjuntos no cantorianos salva a la teoría de la inconsistencia claro, en ese sentido da más posibilidades, si no no hay universo, pero la verdad que se hace algo incomodo tener que descartar funciones definibles

¿también se podría, no exigiendo la estratificación a los pares ordenados, camuflar la circularidad de ciertas definiciones usando pares ordenados en la definición ? ¿Encierra quizás por ese motivo la función singlete e \(  x\longrightarrow{{x}}  \)  algún tipo de singularidad?

No sé si en esto último  entraríamos ya en la cuestión de la naturaleza del par ordenado:  intuitivamente da la sensación de que son átomos más bien, supongo que si es un par ordenado de átomos será atomo y que será conjunto sólo en el caso de que el par sea de conjuntos. No sé si tienes pensado en tu exposición de NFA tratar un poco más este tema, pero creo sería interesante

Para poner llaves en LaTeX debes escribir \{ en lugar de {, porque si no, no se ven.
Título: Re: NFA
Publicado por: Carlos Ivorra en 25 Febrero, 2012, 01:18 am
Citar
Sin esas condiciones podrías expresar \(  x\notin x  \) en la forma \(  \exists y (y\notin x\land y = x)  \)y la fórmula estaría estratificada (si es que admites que los tipos en [y = x] no necesiten ser iguales, o que cada variable pueda tener un tipo diferente en cada subfórmula).

Pues es verdad. Pero ¿la estratificación es la condición mínima que hay que exigir a una fórmula para que no encierre algún tipo de circularidad, en el caso de fórmulas construidas mediante conectivas lógicas a partir de relaciones \(  \in{}]  \) e =  ?

Dudo que haya una condición mínima. Hay condiciones que lo consiguen de un modo y otras de otro.

Lo que no termino de ver es cuando dices refiriendote a la estratificación de los pares ordenados:

Citar
Eso no es una restricción, en el sentido de una limitación, sino todo lo contrario, algo que te da más posibilidades que si no dispusieras de ello.

Pues que siempre puedes considerar pares ordenados no estratificados, desde los usuales \( (x, y)\equiv \{\{x\},\{x,y\}\} \), hasta otros que suban más el tipo, como  \( (x, y)\equiv \{\{\{\{x\},\{x,y\}\}\}\} \), e incluso es posible definir (aunque es más complicado) otros que sólo suban el tipo una unidad. Disponer de pares ordenados que no suban el tipo es una ventaja más, algo cuya existencia debes postular (o demostrar), no una limitación que te impida hacer nada. Si no te gustan los pares nivelados, siempre puedes no usarlos. Postular su existencia no te restringe en nada.

La no existencia de la función singlete \(  x\longrightarrow{{x}}  \)  en conjuntos no cantorianos salva a la teoría de la inconsistencia claro, en ese sentido da más posibilidades, si no no hay universo, pero la verdad que se hace algo incomodo tener que descartar funciones definibles

Je, je. Es que eso es lo que hace de NFA una teoría "extraña", que niega la existencia de conjuntos que ningún matemático "clásico" estaría dispuesto a admitir que no existen. Obliga a quien quiera trabajar en la teoría a decir "esto existe, pero oficialmente no existe", y eso no es nada natural. En ZFC también pasa eso, pero con colecciones más "exóticas", alejadas de la práctica matemática corriente. En NFA pasa con conjuntos "cotidianos". No obstante, hay que reconocer que el axioma de cómputo (y alguno otro más fuerte) alivia bastante la situación.

¿también se podría, no exigiendo la estratificación a los pares ordenados, camuflar la circularidad de ciertas definiciones usando pares ordenados en la definición ? ¿Encierra quizás por ese motivo la función singlete e \(  x\longrightarrow{{x}}  \)  algún tipo de singularidad?

No te entiendo. Insisto en que la estratificación de los pares ordenados no es una exigencia, sino una concesión. Bueno, en realidad creo que nos estamos liando: lo que se "exige" o "concede" no es que los pares ordenados estén estratificados, sino que estén nivelados, es decir, que su tipo sea el mismo que el de sus componentes. Siempre puedes definir pares ordenados no estratificados, pero no podrías hacer nada con ellos, porque no podrías usarlos para definir conjuntos. Lo que se hace para simplificar NFA es, no limitarse a construir los pares ordenados usuales, que resultan estratificados de tipo dos unidades superior a los tipos de sus componentes (para obtener esos pares no necesitas aludir a ellos en la definición de estratificación), sino postular la existencia de pares ordenados "mejores", nivelados. Pero postular la existencia de algo no es una limitación, sino todo lo contrario.

No sé si en esto último  entraríamos ya en la cuestión de la naturaleza del par ordenado:  intuitivamente da la sensación de que son átomos más bien, supongo que si es un par ordenado de átomos será atomo y que será conjunto sólo en el caso de que el par sea de conjuntos.

Si no supones la existencia de pares ordenados nivelados, es decir, si no mencionas pares ordenados en la definición de estratificación, puedes trabajar (más incómodamente) con pares ordenados usuales, y demostrar (suponiendo el axioma de infinitud, que está implícito en la existencia de pares nivelados) que \( |V\times V| = |V| \). En otras palabras, pruebas que existe una biyección \( F: V\times V\longrightarrow V \) biyectiva (de hecho, para lo que voy a decir basta con tomar F inyectiva). Entonces, puedes definir \( (x, y) \equiv F(x,y) \) y tienes un término que cumple lo que yo he postulado sobre pares ordenados nivelados.

Puesto que la aplicación F la puedes elegir arbitrariamente y retocarla como quieras, siempre puedes tomarla de forma que la imagen de un par de átomos sea un átomo. Lo que no puedes hacer es exigir que todos los pares ordenados sean conjuntos, porque hemos visto que hay menos conjuntos que átomos, pero sí puedes exigir que todos los pares ordenados sean átomos, o que los de los átomos sean átomos y los de los conjuntos conjuntos, pues siempre puedes tomar una F que cumpla cualquiera de estas condiciones.

No sé si tienes pensado en tu exposición de NFA tratar un poco más este tema, pero creo sería interesante

En la exposición de NFA sería complicado, porque he supuesto la existencia de pares nivelados desde un principio, luego resultaría tonto construirlos en ese contexto. Trataré el asunto al hablar de modelos de NFA.
Título: Re: NFA
Publicado por: Raúl Aparicio Bustillo en 25 Febrero, 2012, 06:41 pm
 
Citar
lo que se "exige" o "concede" no es que los pares ordenados estén estratificados, sino que estén nivelados, es decir, que su tipo sea el mismo que el de sus componentes

Sí, creo que he sido yo el causante de la confusión: cuando decía par ordenado estratificado me refería a par ordenado nivelado . Estratificada o no estratificada está una fórmula, un término en forma de par ordenado está si acaso  nivelado, en el caso de que los términos que formen su primera componente y los que formen su segunda componente tengan el mismo tipo. Y a la hora de mirar la estratificación de la fórmula en la que aparezca el par ordenado al par ordenado nivelado le tengo que asignar el tipo de sus componentes. Si en la fórmula aparece un par ordenado no nivelado entonces la fórmula ya no está estratificada. Es así ¿no?

Citar
¿también se podría, no exigiendo la estratificación a los pares ordenados, camuflar la circularidad de ciertas definiciones usando pares ordenados en la definición ? ¿Encierra quizás por ese motivo la función singlete \(  x\longrightarrow{\{x}\}}  \) algún tipo de singularidad?

Quise decir "circularidad", no singularidad, aunque creo que aún así debería matizar un poco a qué tipo de impredicatividad me estoy refiriendo. Lo dejo para otro post

Título: Re: NFA
Publicado por: Carlos Ivorra en 25 Febrero, 2012, 09:24 pm
Citar
lo que se "exige" o "concede" no es que los pares ordenados estén estratificados, sino que estén nivelados, es decir, que su tipo sea el mismo que el de sus componentes

Sí, creo que he sido yo el causante de la confusión: cuando decía par ordenado estratificado me refería a par ordenado nivelado . Estratificada o no estratificada está una fórmula, un término en forma de par ordenado está si acaso  nivelado, en el caso de que los términos que formen su primera componente y los que formen su segunda componente tengan el mismo tipo. Y a la hora de mirar la estratificación de la fórmula en la que aparezca el par ordenado al par ordenado nivelado le tengo que asignar el tipo de sus componentes. Si en la fórmula aparece un par ordenado no nivelado entonces la fórmula ya no está estratificada. Es así ¿no?

Creo que sigue flotando alguna clase de confusión. El par ordenado que se introduce axiomáticamente está nivelado, en el sentido de que tiene el mismo tipo que sus componentes, mientras que el par ordenado usual de ZFC, es decir, \( (x,y)\equiv \{\{x\},\{x,y\}\} \) está estratificado, y define fórmulas estratificadas, pero no está nivelado, porque tiene tipo dos unidades mayor que sus componentes.
Título: El axioma de cómputo
Publicado por: Carlos Ivorra en 26 Febrero, 2012, 01:29 pm
El axioma de cómputo (en adelante AC) admite varias versiones equivalentes. Fue introducido por Rosser en la forma siguiente:

\( \forall n\in \mathbb{N}\ |\{1,\ldots, n\}|=n \)

Aunque casi pueda parecer "vergonzoso" que esta afirmación necesite ser introducida como axioma, lo cierto es que no puede probarse en NFA. Una consecuencia que puede extraerse de aquí es que el hecho de que algo sea un axioma en una teoría no significa que exprese una verdad profunda y sutil "indemostrable", sino que es una mera cuestión técnica que depende de los axiomas concretos de la teoría. Por ejemplo, el hecho de que en ZFC sin el axioma de infinitud no pueda demostrarse el axioma de infinitud no debe llevarnos a suponer que la existencia de conjuntos infinitos sea algo "dudoso" o "que escapa al razonamiento humano". De hecho, en ZFC sin el axioma de infinitud se demuestra la existencia de infinitos conjuntos (los números naturales, por ejemplo), lo que no puede probarse es que estén contenidos en un conjunto, y eso es un mero tecnicismo (con consecuencias drásticas en la teoría, pero un tecnicismo al fin y al cabo). Por otro lado, en NFA sí que puede demostrarse el axioma de infinitud (la existencia del conjunto de los números naturales).

Hecha esta digresión, pasamos a observar que si llamamos \( I_n = \{m\mid m\in \mathbb{N}\land 1\leq m\leq n\} \), tenemos una biyección \( f: \text{Ord}_n^<\longrightarrow I_n \) dada por \( m\mapsto \text{card}(m)+1 \), con lo que en NFA se prueba que \( |I_n| = |\text{Ord}_n^<| = T^2(n) \).

Por lo tanto, el axioma de cómputo equivale a que todos los ordinales finitos cumplen \( T^2(n)=n \). Ahora bien, esto implica que \( T(n)=n \), ya que si fuera \( T(n)<n \), entonces \( n=T^2(n)<T(n) \), contradicción, y si fuera \( n<T(n) \) entonces \( T(n)<T^2(n)=n \), contradicción.

Por lo tanto, tenemos dos equivalencias del axioma de cómputo:

Todo ordinal finito cumple \( T(n)=n \).
y
Todo cardinal finito (todo número natural) cumple \( T(n)=n \).

En los términos introducidos en el mensaje anterior a su vez podemos expresar estas dos variantes como sigue:

Todos los ordinales finitos son cantorianos.
y
Todos los números naturales son cantorianos.

En el mensaje anterior demostramos que si todos los ordinales menores que un ordinal dado son cantorianos, entonces el ordinal dado es fuertemente cantoriano. Recíprocamente, todos los ordinales menores que un ordinal fuertemente cantoriano son fuertemente cantorianos. Esto nos da las equivalencias siguientes (del axioma de cómputo):

\( \omega \) es un ordinal fuertemente cantoriano.

Todos los ordinales finitos son fuertemente cantorianos.

De las propias definiciones se sigue que cada una de ellas equivale a una de las siguientes:

\( \aleph_0 \) es un cardinal fuertemente cantoriano.

Todos los números naturales son fuertemente cantorianos.

Hasta aquí las equivalencias. Ahora destacamos algunas consecuencias. Por los resultados del mensaje anterior, el hecho de que los ordinales finitos sean fuertemente cantorianos implica que también lo son los cardinales \( \aleph_n \), lo que a su vez implica que \( \kappa_0 \) no puede ser ninguno de ellos, luego existe el cardinal \( \aleph_\omega \), cosa que no puede probarse en NFA. De hecho, \( \aleph_\omega \) es un cardinal fuertemente cantoriano, al igual que  \( \aleph_{\omega+1}, \aleph_{\omega+2},\ldots \aleph_{\omega+\omega} \) y muchos cardinales más. Lo mismo vale cambiando \( \aleph \) por \( \beth \). Esto significa que todos los conjuntos numerables son fuertemente cantorianos, al igual que todos los de cardinal \( 2^{\aleph_0} \) (o menor), y todos los de cardinal \( 2^{2^{\aleph_0}} \), etc.

En resumen, suponiendo AC, cualquier conjunto que se vaya a encontrar cualquier matemático en su trabajo cotidiano (excluyendo a los que trabajan en teoría de conjuntos) es fuertemente cantoriano.

¿Y cuál es la ventaja de disponer de tantos conjuntos fuertemente cantorianos?

Por lo pronto, el hecho de que los números naturales cumplan \( T(n) = n \) y, por consiguiente, \( T^m(n)=n \), para todo número entero \( m \), se traduce en que podemos permitir que los números naturales violen la estratificación en las fórmulas en las que aparezcan. En efecto, el tipo de \( T^m(n) \) es el tipo de la variable \( n \) más el número entero \( m \). Por ejemplo, habíamos explicado que en NFA no podemos demostrar que la unión de \( m \) conjuntos disjuntos de cardinal \( n \) tiene cardinal \( mn \) porque la fórmula siguiente no está estratificada:

\( m_1\in \mathbb{N}_2\land \forall A_1(|A_1|_2=m_2\land \forall x_0\in A_1\ (\text{cto}\,x_0\land |x_0|_1=n_1\land{} \) \( \forall x_0y_0\in A_1(x_0\neq y_0\rightarrow x_0\cap y_0=\emptyset_0)\rightarrow |\bigcup A_1|_1=m_1n_1) \).

El problema está en que la variable \( m \) necesita aparecer con tipo \( 2 \) en un sitio y con tipo \( 1 \) en otro. En NFA esto no tiene arreglo, pero en NFA + AC basta sustituir la fórmula anterior por

\( m_1\in \mathbb{N}_2\land \forall A_1(|A_1|_2=T(m_1)_2\land \forall x_0\in A_1\ (\text{cto}\,x_0\land |x_0|_1=n_1\land{} \) \( \forall x_0y_0\in A_1(x_0\neq y_0\rightarrow x_0\cap y_0=\emptyset_0)\rightarrow |\bigcup A_1|_1=m_1n_1) \).

Como \( \forall m\in \mathbb{N}\ T(m)=m \), esta fórmula es equivalente a la anterior, pero está estratificada y define un conjunto de números naturales, el conjunto

\( A = \{m\mid m\in \mathbb{N}\land \forall A(|A|=T(m)\land \forall x\in A\ (\text{cto}\,x\land |x|=n\land{} \)\( \forall xy\in A(x\neq y\rightarrow x\cap y=\emptyset)\rightarrow |\bigcup A|=mn)\} \),

que es el mismo conjunto que

\( A = \{m\mid m\in \mathbb{N}\land \forall A(|A|=m\land \forall x\in A\ (\text{cto}\,x\land |x|=n\land{} \)\( \forall xy\in A(x\neq y\rightarrow x\cap y=\emptyset)\rightarrow |\bigcup A|=mn)\} \).

Observemos que no podemos asegurar directamente la existencia de este conjunto (con la segunda expresión), porque la fórmula que lo define no está estratificada, pero sí que podemos probar que existe \( A \) con la primera definición y resulta que satisface también la segunda definición no estratificada. En la práctica, en lugar de introducir Ts en las fórmulas, basta tener presente que no es necesario ajustar los tipos de las variables que representen números naturales, pues si incumplen la estratificación siempre se pueden sustituir por términos \( T^k(x) \) (incluso con exponentes diferentes en cada sitio) para cuadrar la estratificación.

Una vez definido \( A \), ya podemos aplicarle el principio de inducción para probar sin dificultad que \( A=\mathbb{N} \). La única dificultad de la prueba era demostrar que existe \( A \).

En realidad, no sólo podemos "subvertir" la estratificación en variables que recorran números naturales, sino en cualquier variable que recorra un conjunto fuertemente cantoriano. En efecto, si \( X \) es un conjunto fuertemente cantoriano y \( f:X\longrightarrow \mathcal P_1X \) es la aplicación dada por \( f(x)=\{x\} \), podemos definir

\( T_X(x)\equiv f^{-1}(\{x\},\qquad T_X^{-1}(x)\equiv \bigcup f(x) \),

de modo que los términos \( T_X(x) \) y \( T_X^{-1}(x) \) están estratificados, el primero sube el tipo una unidad y el segundo lo baja una unidad, pero trivialmente \( \forall x\in X\ T_X(x)=x \) y \( \forall x\in X\ T_X^{-1}(x)=x \).

Esto nos permite ajustar los tipos de una variable \( x \) que recorra el conjunto \( X \) en una fórmula sustituyendo cada una de sus apariciones por un término \( T_X^m(x) \), para un número entero \( m \) adecuado.

Por ejemplo, supongamos que \( X \) es un conjunto fuertemente cantoriano en el que hemos definido una relación de equivalencia \( R \) y queremos definir la proyección \( p: X\longrightarrow X/R \) dada por \( p(x)=[x] \). Esto no puede hacerse si \( X \) es arbitrario, porque la clase de equivalencia \( [x] \) tiene tipo una unidad mayor que \( x \), pero si \( X \) es fuertemente cantoriano basta definir \( p(x) = [T_X^{-1}(x)] \). Ahora la definición está estratificada, pero para todo \( x\in X \) se cumple que \(  [T_X^{-1}(x)] = [x] \), luego hemos definido exactamente la aplicación que queríamos definir.

En la práctica no es necesario introducir el operador \( T_X^{-1} \), sino que basta tener presente que las variables que recorren conjuntos fuertemente cantorianos no necesitan ser estratificadas. Como, según hemos destacado, en las matemáticas "cotidianas" uno trabaja siempre con fórmulas que hablan de objetos pertenecientes a conjuntos fuertemente cantorianos, la conclusión es que la necesidad de comprobar que las fórmulas que usamos para definir conjuntos estén estratificadas desaparece casi por completo. (Sólo hay que recordarla si en algún momento introducimos una condición del tipo "existe un conjunto, o un cardinal, etc. tal que...", sin que podamos prescribir un conjunto fuertemente cantoriano donde sepamos que vaya a estar ese conjunto, o cardinal, etc. que necesitamos que exista.

Naturalmente, las condiciones de estratificación no dejarán de estar presentes en teorías generales: si uno estudia espacios topológicos en general, eso incluye espacios topológicos sobre conjuntos que no sean fuertemente cantorianos. Ahora bien, si uno se restringe a considerar espacios topológicos fuertemente cantorianos (con lo cual no pierde ningún ejemplo "cotidiano" al que se pueda aplicar la topología) automáticamente puede prescindir "casi por completo" de las restricciones de estratificación.

Si el lector considera que esto es demasiado vago (que lo es), ello se debe a que la forma de ilustrar estas ideas es simplemente formalizando las matemáticas en NFA + AC (por ejemplo, la construcción de los conjuntos de números, los resultados básicos del análisis de funciones de variable real, etc.) Al hacerlo se vería claramente que es posible olvidarse completamente de la estratificación.

El axioma de cómputo no puede demostrarse en NFA, y una razón profunda es que implica la existencia de un modelo de NFA y, por consiguiente, la consistencia de NFA. El teorema de incompletitud de Gödel asegura que un axioma que implique la consistencia de una teoría no puede ser demostrado en dicha teoría.

Más precisamente, AC implica la existencia de un modelo de la teoría de Zermelo con el axioma de elección (es decir, ZFC sin el axioma del reemplazo y con el axioma de selección de subconjuntos en su lugar), y a partir de un modelo de ZC puede construirse un modelo de NFA.
Título: Modelos de NFA
Publicado por: Carlos Ivorra en 04 Marzo, 2012, 01:37 pm
Una vez expuestos los resultados básicos de NFA, vamos a ver que en ZFC podemos construir un modelo de NFA, lo cual se traduce en que si ZFC es consistente entonces NFA también lo es. En este mensaje expondremos los preparativos para la prueba.

Más concretamente, vamos a ver que en ZFC puede construirse un modelo de NFA a partir de un modelo de ZC, donde ZC es la teoría que resulta de eliminar el axioma del reemplazo de ZFC y sustituirlo por el axioma de especificación.

Remito al hilo http://rinconmatematico.com/foros/index.php/topic,21322.0.html (http://rinconmatematico.com/foros/index.php/topic,21322.0.html) de argentinator donde están detallados estos axiomas. Allí el axioma de especificación aparece como parte de los axiomas de ZFC, si bien puede demostrarse a partir del axioma de reemplazo, por lo que es redundante, pero si eliminamos el axioma de reemplazo necesitamos incluir el axioma de especificación, ya que deja de ser un teorema.

¿Por qué partimos de un modelo de ZC en lugar de ZFC? Porque en ZFC no puede demostrarse la existencia de un modelo de ZFC, pero es fácil construir modelos de ZC.

En efecto, en ZFC podemos definir: \( V_0 = \emptyset \), para cada ordinal \( \alpha \) tomamos \( V_{\alpha+1}=\mathcal PV_\alpha \) y para cada ordinal límite \( \lambda \) definimos \( V_\lambda = \bigcup\limits_{\delta<\lambda}V_\delta \).

Tenemos así definida una sucesión transfinita de conjuntos, y una simple inducción demuestra que son transitivos, es decir, que si \( x\in V_\alpha \) entonces \( x\subset V_\alpha \).

Es fácil ver que para todo ordinal límite \( \lambda>\omega \) se cumple que \( V_\lambda \) es un modelo (transitivo) de ZC, es decir, que si definimos "conjunto" como "elemento de \( V_\lambda \) y "pertenencia" como la pertenencia usual restringida a \( V_\lambda \) esos conjuntos específicos con esa pertenencia cumplen todos los axiomas de ZC.

Por ejemplo, para probar el axioma de especificación tomamos un conjunto \( A\in V_\lambda \) y otros conjuntos \( x_1,\ldots, x_n\in V_\lambda \) y una fórmula \( \phi(x,x_1,\ldots, x_n) \), y hemos de probar que el conjunto

\( \{x\in A\mid \phi(x,x_1,\ldots, x_n)\} \)

es también un "conjunto" en el sentido específico que le estamos dando a la palabra, es decir, hemos de probar que pertenece a \( V_\lambda \). Ahora bien, existe un \( \delta<\lambda \) tal que \( A\in V_\delta \). Así \( \{x\in A\mid \phi(x,x_1,\ldots, x_n)\}\subset A\subset V_\delta \), luego \( \{x\in A\mid \phi(x,x_1,\ldots, x_n)\}\in \mathcal PV_\delta = V_{\delta+1}\subset V_\lambda \).

Nota para entendidos
En realidad habría que haber probado que el conjunto definido por la relativización de \( \phi \) a \( V_\lambda \) está en \( V_\lambda \), pero da igual, pues si vale para toda fórmula vale también en particular para la relativización de toda fórmula.
[cerrar]

En particular \( V_{\omega+\omega}=V_{\omega\cdot 2} \) es el modelo más sencillo de ZC que puede construirse de esta forma (aunque no el más pequeño, pues, naturalmente, puede probarse que existen modelos numerables). Observemos que está formado por todos los conjuntos que se obtienen a partir del conjunto vacío mediante un número finito de aplicaciones del operador "partes de", lo que nos lleva al conjunto \( V_\omega \) (que sólo contiene conjuntos finitos y es un modelo de ZFC menos el axioma de infinitud), más todos los conjuntos que pueden obtenerse a partir de \( V_\omega \) aplicando otra vez un número finito de veces el operador "partes de".

Según decimos, es posible construir un modelo de NFA a partir de cualquier modelo de ZC, como por ejemplo \( V_{\omega\cdot 2} \)

Cabe advertir que este resultado puede refinarse en varios aspectos: por una parte no es necesario partir de un modelo de ZC, sino que basta un modelo de una teoría más débil, la teoría de MacLane (MAC), que resulta de restringir el axioma de especificación a fórmulas de tipo \( \Delta_0 \), es decir, que sólo tienen cuantificadores acotados, de la forma \( \forall x\in y \) o \( \exists x\in y \); por otra parte, no es necesario trabajar en ZFC, sino que bastaría por ejemplo trabajar en la teoría de Mostowski, que es una teoría que sigue siendo mucho más débil que ZFC y que resulta de añadir un axioma a MAC cuya consistencia es probable a partir de la de MAC (en el mismo sentido en que la consistencia de la hipótesis del continuo es probable a partir de la de ZFC).

También hay otro punto que debe ser matizado: en los modelos \( V_\lambda \) puede definirse la sucesión \( \{V_\delta\}_{\delta<\lambda} \), mientras que la existencia de esta sucesión no puede demostrarse en ZC. Nosotros vamos a apoyarnos en el hecho de que en nuestro modelo de partida pueden definirse los conjuntos \( V_\alpha \) para todo ordinal \( \alpha \) y cumplen que todo conjunto pertenece a un \( V_\alpha \), por lo tanto, estamos partiendo de un modelo que cumple más de lo que podemos asegurar que cumple cualquier modelo de ZC. Esto no hace que la afirmación que hemos hecho sea falsa: a partir de todo modelo de ZC puede construirse un modelo de NFA, porque sucede que a partir de todo modelo de ZC puede construirse un modelo en el que pueden definirse los conjuntos \( V_\alpha \). (En general sólo pueden definirse como una clase bien ordenada de conjuntos, sin que necesariamente se pueda asignar un ordinal a cada uno de ellos, lo cual únicamente complica un poco la notación, al tener que trabajar directamente con los conjuntos \( V_\alpha \) sin poder hacer referencia a los ordinales como subíndices, pero no es un obstáculo esencial. Aquí evitaremos estos tecnicismos partiendo de un modelo de ZC en el que cada ordinal \( \alpha \) determina un conjunto \( V_\alpha \) y todo conjunto pertenece a un \( V_\alpha \).)



El primer paso de la construcción es muy técnico, así que omitiré la prueba, salvo que haya alguien interesado que me pida los detalles. Se trata de probar el teorema siguiente, que es un caso particular del llamado teorema de Ehrenfeucht-Mostowski:

Dado un modelo \( M \) de un lenguaje formal (en nuestro caso el lenguaje de ZFC) en el que pueda definirse un subconjunto \( A\subset M \) (que no tiene por qué ser la extensión de un conjunto en \( M \), y que en nuestro caso será el conjunto formado por los elementos de \( M \) que satisfacen la definición de ordinal infinito, que no es un conjunto en \( M \)) en el que pueda definirse una relación de orden total (en nuestro caso la relación de orden usual en los ordinales), existe otro modelo \( M' \) que es elementalmente equivalente a \( M \) (es decir, que las sentencias verdaderas en \( M \) son las mismas que las sentencias verdaderas en \( M' \)) en el que existe un automorfismo \( J: M\longrightarrow M' \) que no deja invariante a algún elemento de \( A \).

Aquí es importante que si partimos de un modelo \( M \) de ZC, no es necesario que sea un modelo natural, es decir, un modelo en el que la relación de pertenencia es la pertenencia usual, como ocurre en los modelos \( V_\lambda \), sino que la pertenencia puede interpretarse mediante una relación \( E\subset M\times M \) arbitraria. Pero, tanto si partimos de un modelo natural como si no, el modelo \( M' \) resultante no tiene por qué ser natural, sino que tendrá su propia relación de pertenencia \( E'\subset M'\times M' \).

Que \( J \) sea un automorfismo significa, en este caso, que es una biyección \( J: M'\longrightarrow M' \) con la propiedad de que \( \forall xy\in M'(x\,E\,y\leftrightarrow J(x)\,E'\,J(y)) \). Puesto que \( J \) respeta la pertenencia y también la igualdad (por ser biyectiva) y toda afirmación del lenguaje de ZC se construye a partir de fórmulas construidas por variables, pertenencias e igualdades, es fácil concluir que \( J \) respeta todas las fórmulas, es decir, que si \( \phi(x_1,\ldots, x_n) \) es una fórmula cualquiera, entonces

\( \forall x_1\cdots x_n\in M' (M'\vDash \phi(x_1,\ldots, x_n)\leftrightarrow M'\vDash \phi(J(x_1),\ldots, J(x_n)) \).

Conozco dos formas de probar este resultado: una requiere más familiaridad con la teoría de modelos, pues se basa en el teorema de compacidad y en la construcción de submodelos mediante funciones de Skolem; la otra es más conjuntista, pues en ella se obtiene el modelo como límite inductivo de un sistema de ultrapotencias del modelo de partida. Puestos a contar una de las dos, me parecería más sencillo explicar la segunda. Está expuesta en el artículo de Holmes "Strong Axioms of Infinity in NFU", que se encuentra fácilmente en internet. (Otra cosa es que, en general, y en este caso en particular, Holmes se explica como un libro cerrado.)

Aplicando este teorema tenemos un modelo \( (M,E) \), donde \( E\subset M\times M \), que cumple las mismas sentencias que cualquier \( V_\lambda \) prefijado (con la pertenencia usual), de modo que en particular es un modelo de ZC, pero en el que tenemos un automorfismo \( J: M\longrightarrow M \) tal que existe un \( \alpha\in M \) de modo que \( M\vDash \alpha \) es un ordinal infinito y \( J(\alpha)\neq \alpha \).

Conviene destacar que a la hora de entender "cómo puede ser que unos objetos cumplan los axiomas de NFA", la construcción de este modelo \( M \) (que estamos omitiendo) no arroja ninguna luz, sino que es un mero tecnicismo. Quiero decir que, al fin y al cabo, \( M \) sigue siendo un modelo de una teoría "normal", como es ZC (incluso podríamos suponer que es un modelo de ZFC si no nos importa postular su existencia, no demostrable en ZFC).  Es cierto que la existencia de \( J \) le introduce ciertas "rarezas" (lo convierte en un modelo no estándar), pero es el paso de \( M \) a un modelo de NFA, que vamos a detallar, el que muestra qué son realmente los conjuntos de NFA y por qué cumplen los axiomas que cumplen.

Como \( J \) conserva todas las fórmulas, se cumple también que \( M\vDash J(\alpha) \) es un ordinal infinito, y puesto que en \( M \) los ordinales están totalmente ordenados, tiene que ser o bien \( M\vDash \alpha<J(\alpha) \) o bien \( M\vDash J(\alpha)<\alpha \).

Si se da el primer caso, cambiamos \( J \) por \( J^{-1} \) y tenemos que se cumple el segundo. Así pues, no perdemos generalidad si suponemos que \( M\vDash J(\alpha)<\alpha \).



A partir de aquí podemos "desprendernos" de una buena parte de la teoría de modelos y los problemas conceptuales que puede presentar el tratar con modelos para alguien poco familiarizado con ellos mediante la estrategia siguiente:

Consideremos un lenguaje formal cuyos signos son los del lenguaje de ZFC más un signo de función \( j \), es decir, un signo tal que si \( t \) es un término del lenguaje, entonces \( j(t) \) también es un término. Consideremos la teoría axiomática T cuyos axiomas son los siguientes:

1) Todos los axiomas de ZC.
2) La afirmación que viene a decir que para cada ordinal \( \alpha \) existe el conjunto \( V_\alpha \) y que todo conjunto pertenece a un \( V_\alpha \) (esto es un teorema de ZFC, pero no de ZC).
3) \( \forall xy(j(x)=j(y)\leftrightarrow x = y) \)
4) \( \forall x\exists y\ x = j(y) \)  (Había una errata aquí.)
5) \( \forall xy(x\in y\leftrightarrow j(x)\in j(y)) \)
6) \( \exists \alpha\ (\alpha\text{ es un ordinal infinito}\land  j(\alpha)<\alpha) \)

Lo que prueban los resultados descritos en el apartado anterior es que si ZFC es consistente, entonces T también es consistente, pues el modelo \( M \) que hemos construido (o cuya existencia hemos afirmado que puede probarse en ZFC) cumple todos los axiomas de T sin más que interpretar el nuevo signo \( j \) como el automorfismo \( J \).

Por consiguiente, lo único que queda por hacer es construir un modelo de NFA partiendo de los axiomas de T. De este modo, si NFA fuera contradictorio, en T podríamos demostrar que no existen modelos de NFA, pero habremos demostrado que existe uno, luego tendremos que T es también contradictoria y, según hemos dicho, esto implica que ZFC es contradictorio.

Cabe señalar que la prueba en su conjunto es totalmente constructiva y finitista. Es decir, el argumento (debidamente detallado) proporciona un algoritmo por el que podríamos programar a un ordenador para que a partir de la demostración de una contradicción en NFA nos diera una demostración de una contradicción en ZFC, e incluso la demostración de una contradicción en la teoría MAC + consis MAC, que es mucho más débil que ZFC.

Completaremos la prueba la semana que viene. Soy consciente de que este mensaje puede resultar mucho más oscuro que cualquiera de los anteriores, así que si algún lector necesita aclaraciones adicionales sobre lo dicho aquí, que no dude en preguntar.
Título: Modelos de NFA (2)
Publicado por: Carlos Ivorra en 11 Marzo, 2012, 04:04 pm
Continuamos la demostración de que si ZFC es consistente también lo es NFA. Hasta ahora hemos probado (o hemos esbozado la prueba de) que si ZFC es consistente también lo es la teoría ZC más la existencia de un automorfismo \( j \) de la clase de todos los conjuntos que, para un cierto ordinal infinito \( \alpha \), se cumple que \( j(\alpha)<\alpha \).

Que \( j \) sea un automorfismo significa que cumple los axiomas

\( \forall xy(j(x)=j(y)\leftrightarrow x = y) \)

\( \forall x\exists y\ x = j(y) \)

\( \forall xy(j(x)\in j(y)\leftrightarrow x \in y) \)

Los dos primeros axiomas permiten definir \( j^{-1}(x) \) como el único conjunto \( y \) tal que \( x = j(y) \), y es fácil ver que los tres axiomas anteriores se cumplen también si cambiamos \( j \) por \( j^{-1} \).

Más en general, si \( \phi(x_1,\ldots, x_n) \) es cualquier fórmula del lenguaje de ZFC (es decir, en la que no aparezca \( j \)) se cumple

\( \forall x_1\cdots x_n(\phi(x_1,\ldots,x_n)\leftrightarrow \phi(j(x_1),\ldots, j(x_n)) \),

y lo mismo es válido para \( j^{-1} \) en lugar de \( j \).

Demostración
Supongamos que existe una fórmulapara la que el teorema no es cierto. Entonces podemos tomar una subfórmula \( \phi \)  de longitud mínima que lo incumpla, de modo que todas las subfórmulas de  \( \phi \) cumplen el teorema, pero  \( \phi \) lo incumple.

La fórmula \( \phi \) no puede ser de la forma \( x_i = x_j \) o \( x_i\in x_j \), pues éstas cumplen el teorema como consecuencia de los axiomas que estamos suponiendo.

Tampoco puede ser de la forma \( \lnot\psi \), pues, como \( \psi \) es una subfórmula de \( \phi \), tenemos que

\( \forall x_1\cdots x_n(\psi(x_1,\ldots,x_n)\leftrightarrow \psi(j(x_1),\ldots, j(x_n)) \),

y esto implica

\( \forall x_1\cdots x_n(\lnot\psi(x_1,\ldots,x_n)\leftrightarrow \lnot\psi(j(x_1),\ldots, j(x_n)) \).

Igualmente se razona que  \( \phi \) no puede ser de la forma  \( \psi\rightarrow \chi \),  \( \psi\lor\chi \) o  \( \psi\land \chi \), pues  las subfórmulas \( \psi \) y  \( \chi \) cumplirían el teorema y esto implica que  \( \phi \) también lo cumple.

Veamos el caso en que  \( \phi \) es  \( \exists x\psi(x,x_1,\ldots, x_n) \). Estamos suponiendo que la subfórmula  \( \psi \) cumple el teorema, es decir, que

\( \forall xx_1\cdots x_n(\psi(x,x_1,\ldots,x_n)\leftrightarrow \psi(j(x),j(x_1),\ldots, j(x_n)) \)

Entonces, si se cumple \( \phi(x_1,\ldotx, x_n) \), existe un conjunto \( x \) tal que \( \phi(x,x_1,\ldots, x_n) \), luego \( \psi(j(x),j(x_1),\ldots, j(x_n)) \), luego \( \exists x\psi(x,j(x_1),\ldots, j(x_n)) \), es decir, \( \phi(j(x_1),\ldots, j(x_n)) \).

Recíprocamente, si \( \phi(j(x_1),\ldots, j(x_n)) \) existe un \( x \) tal que \( \psi(x,j(x_1),\ldots, j(x_n) \), luego existe un \( y \) tal que \( x = j(y) \), de modo que \( \psi(j(y),j(x_1),\ldots, j(x_n) \), y por hipótesis de inducción \( \psi(y,x_1,\ldots, x_n)) \), luego \( \exists x\psi(x,x_1,\ldots, x_n) \), que es lo mismo que \( \phi(x_1,\ldots, x_n) \).

El caso en que \( \phi \) sea \( \forall x \psi(x,x_1,\ldots, x_n) \) se trata análogamente o bien se reduce a los anteriores mediante la equivalencia entre \( \phi \) y \( \lnot \exists x\lnot\psi \).

Esto prueba que el contraejemplo \( \phi \) no puede existir en ningún caso, luego el teorema es correcto.
[cerrar]

En otras palabras, si un conjunto \( x \) cumple cualquier propiedad expresable con una fórmula que depende de unos parámetros, entonces \( j(x) \) cumple la misma propiedad respecto de las imágenes por \( j \) de los parámetros. En particular, si \( \phi(x) \) es una fórmula sin parámetros, tenemos que todo conjunto cumple \( \phi(x)\leftrightarrow  \phi(j(x)) \).

Por ejemplo, si tomamos \( \phi(x) \equiv \forall y\ y\notin x \), tenemos que \( \phi(\emptyset) \), luego \( \phi(j(\emptyset) \), luego \( j(\emptyset) =\emptyset \).

Similarmente, si \( \phi(x,y) \equiv y \) es el ordinal siguiente a \( x \), entonces, para todo ordinal \( \delta \) se cumple \( \phi(\delta, \delta+1) \), luego \( \phi(j(\delta),j(\delta+1) \), luego \( j(\delta+1)=j(\delta)+1 \).

A partir de aquí fijamos un ordinal infinito \( \alpha \) tal que \( j(\alpha)<\alpha \) y vamos a construir un modelo de NFA. Su universo será \( M = V_\alpha \), es decir, los objetos de NFA (conjuntos y átomos) serán los elementos de \( V_\alpha \) y, de entre ellos llamaremos conjuntos a los elementos de \( C = V_{j(\alpha)+1} \).

Notemos que esto tiene sentido, pues, como \( j(\alpha)<\alpha \), tenemos que \( j(\alpha)+1\leq \alpha \), luego \( C=V_{j(\alpha)+1}\subset V_\alpha=M \).

En particular, los átomos de \( M \) serán los elementos de \( M\setminus C \).

Nota
Observemos que si fuera \( \alpha = j(\alpha)+1 \) tendríamos que \( M=C \) y no habría átomos, pero este caso no puede darse, pues, todo ordinal infinito \( \alpha \) puede expresarse de forma única como \( \alpha = \lambda+n \), donde \( \lambda \) es un ordinal límite y \( n \) un número natural, con lo que \( j(\alpha) = j(\lambda)+j(n) \) y, si se diera la igualdad anterior, sería \( \alpha = j(\lambda)+j(n)+1 \), donde \( j(\lambda) \) es un ordinal límite y \( j(n) \) es un número natural. Por la unicidad \( n = j(n)+1 \), pero esto no puede ser, porque \( n \) es par si y sólo si \( j(n) \) es par.

Esto no significa que no pueda construirse un modelo de NFA sin átomos por otro procedimiento, pero por éste no puede ser. Hasta ahora nadie ha sabido construir un modelo de NFA sin átomos a partir de un modelo de ZFC.
[cerrar]

Definimos ahora la relación de pertenencia entre los objetos de \( M \). Concretamente, definimos \( E\subset M\times M \) de modo que

\( x\,E\,y\leftrightarrow y\in C\land j(x)\in y \).

Notemos que, en principio, \( x\in V_\alpha \), \( y\in V_{j(\alpha)+1} \), luego \( y\subset V_{j(\alpha)} \) y \( j(x)\in V_{j(\alpha)} \).

Por último, como \( V_\alpha \) es infinito, existe una aplicación inyectiva (biyectiva si queremos) \( F: V_\alpha\times V_\alpha\longrightarrow V_\alpha \). Para cada par de objetos \( x,y\in M \), definimos su par ordenado (nivelado) como \( (x,y) = F(x,y) \), que es ciertamente otro objeto de \( M \).

Lo que hemos de probar a partir de aquí es que si llamamos "objetos" a los elementos de \( M \), llamamos "conjuntos" a los elementos de \( C \), llamamos "átomos" a los elementos de \( M\setminus C \), llamamos "par ordenado de componentes \( x,y \)" a \( F(x,y) \) y consideramos que la pertenencia entre dos objetos es la relación \( E \), entonces se cumplen los axiomas de NFA.

Notemos que el modelo \( M \) que acabamos de definir cumple el axioma de los átomos, pues (en la definición de \( E \)) hemos exigido como condición para que un objeto \( x \) pueda pertenecer a un objeto \( y \) que el segundo sea un conjunto. Así pues, los átomos no tienen elementos.

Axioma de extensionalidad: Tomamos dos conjuntos \( x, y\in C \) y suponemos que tienen los mismos elementos, es decir, que para todo objeto \( u\in M \) se cumple que \( u\,E\,x\leftrightarrow u\,E\,y \). Hemos de probar que \( x=y \).

La hipótesis es que, para todo \( u\in V_\alpha \), se cumple \( j(u)\in x\leftrightarrow j(u)\in y \).

Ahora bien, \( u\in V_\alpha \) equivale a \( j(u)\in j(V_\alpha) = V_{j(\alpha)} \), luego la hipótesis es también que para todo \( v\in V_{j(\alpha)} \) se cumple \( v\in x\leftrightarrow v\in y \), Como \( x,y\subset V_{j(\alpha)} \), esto implica que \( x=y \).

Axioma de los pares ordenados: El axioma de los pares ordenados se cumple trivialmente en \( M \), pues si \( x,y,u,v \) son cuatro objetos de \( M \) que cumplen \( (x,y) = (u,v) \), esto no significa sino que \( F(x,y) = F(u,v) \), pero \( F \) es inyectiva, luego \( x=u\land y=v \).

Axioma de formación de conjuntos: Ahora hemos de probar que las fórmulas estratificadas del lenguaje de NFA definen conjuntos. Toda fórmula estratificada es equivalente a una fórmula estratificada sin descriptores, así que podemos partir de una fórmula sin descriptores.

Más precisamente, toda fórmula estratificada es equivalente a otra fórmula estratificada cuyas subfórmulas son únicamente de uno de los tipos siguientes:

\( x = y,\qquad x\in y,\qquad x = (u,v) \).

Esbozo de prueba
Una subórmula general sería de la forma \( t_1=t_2 \) o \( t_1\in t_2 \), donde \( t_1 \) y \( t_2 \) son términos formados con pares ordenados y variables, pero ambos tipos de subfórmulas son equivalentes a fórmulas de la forma \( \existx xy(x=t_1\land y=t_2\land x=y) \) o bien \( \exists xy(x=t_1\land x = t_2\land x\in y) \), que claramente están estratificadas si lo están las fórmulas de partida.

A su vez, una fórmula de tipo \( x =(t_1,t_2) \) es equivalente a \( \exists uv(x = (u, v)\land u = t_1\land v=t_2) \), y aplicando transformaciones de este tipo un número finito de veces llegamos a una fórmula cuyas subfórmulas sean todas de los tipos indicados, sin perder la estratificación en el proceso.
[cerrar]

Si \( \phi(x_1,\ldots, x_n) \) es cualquier fórmula del lenguaje de NFA cuyas subfórmulas se restringan a los tipos indicados, podemos definir a partir de ella una fórmula \( \phi^1(x_1,\ldots, x_n,a,f) \) del lenguaje de ZFC más el signo j de modo que la fórmula \( \phi(x_1,\ldots, x_n) \) es verdadera en el modelo \( M \) si y sólo si \( \phi^1(x_1,\ldots, x_n, \alpha, F) \), donde \( F \) es la función que define los pares ordenados nivelados en \( M \).

En efecto, para construir \( \phi^1 \) basta sustituir cada subfórmula de tipo \( \text{cto}\,x \) por \( x\in V_{j(\alpha)+1} \), cada subfórmula de tipo \( x\in y \) por \( y\in V_{j(\alpha)+1}\land j(x)\in y \), cada subfórmula \( x = (u,v) \) por \( x = F(u,v) \) y cada cuantificador \( \forall x \) o \( \exists x \) por \( \forall x\in V_\alpha \) o \( \exists x\in V_\alpha \).

Supongamos ahora que la fórmula de partida admite una estratificación en la que cada variable \( x \) tiene asignado un tipo \( t_x \), y sea \( N \) un número natural mayor que todos los tipos asignados. En tal caso, en lugar de construir la fórmula \( \phi^1 \), podemos construir otra fórmula equivalente \( \phi^2 \) del modo siguiente:

En lugar de sustituir cada subfórmula \( \text{cto}\,x \) por \( x\in V_{j(\alpha)+1} \), la sustituimos por la subfórmula equivalente \( j^{N-t_x}(x)\in V_{j^{N-t_x+1}(\alpha)+1} \).

(Aquí \( j^3(x) = j(j(j(x))) \), etc.)

En lugar de dejar invariante cada subfórmula \( x=y \), la sustituimos por la subfórmula equivalente \( j^{N-t_x}(x)=j^{N-t_y}(y) \).

La fórmula es equivalente porque, como \( x=y \) es una subfórmula de \( \phi \), la definición de estratificación exige que \( t_x=t_y \), luego hemos aplicado \( j \) el mismo número de veces a ambos miembros.

En lugar de sustituir una subfórmula \( x\in y \) por \( y\in V_{j(\alpha)+1}\land j(x)\in y \), la sustituimos por la subfórmula equivalente \( j^{N-t_y}(y)\in V_{j^{N-t_y+1}(y)}\land j^{N-t_x}(x)\in j^{N-t_y}(y) \).

Nuevamente hemos usado que, por definición de estratificación, \( t_y = t_x+1 \), por lo que en la parte final hemos aplicado \( j \) el mismo número de veces a los dos miembros.

En lugar de sustituir \( x = (u,v) \) por \( x = F(u,v) \) la sustituimos por \( j^{N-t_x}(x) = j^{N-t_x}(F)(j^{N-t_u}(u),j^{N-t_v}(v)) \), donde la estratificación garantiza una vez más que \( t_x = t_u = t_v \), por lo que las fórmulas son ciertamente equivalentes.

De este modo, en la fórmula \( \phi^2 \) cada variable \( x \) distinta de la variable \( a \) que sustituimos por \( \alpha \) aparece en un término de la forma \( j^{N-t_x}(x) \). Además en \( \phi^2 \) pueden aparecer varios parámetros de la forma \( j^k(F) \), para distintos valores de \( k \).

En particular, cada variable que en \( \phi \) está ligada por un cuantificador \( \forall x \) aparece en \( \phi^2 \) en la forma \( \forall x\in V_\alpha\,\psi(j^{N-t_x}(x),\ldots) \), pero esto es equivalente a

\( \forall x\in V_{j^{N-t_x}(\alpha)}\,\psi (x,\ldots,) \), ya que cuando \( x \) recorre todos los elementos de \( V_\alpha \), tenemos que \( j^{N-t_x}(x) \) recorre todos los elementos de \( V_{j^{N-t_x}(\alpha)} \). (Lo mismo vale para cuantificadores existenciales.)

Esto significa que la fórmula \( \phi^2(x_1,\ldots, x_n,) \) es equivalente a una fórmula \( \phi^3(j^{N-t_{x_1}}(x_1),\ldots, j^{N-t_{x_n}}(x_n)) \), donde \( \phi^3(u_1,\ldots, u_n) \) es una fórmula del lenguaje de ZFC (sin el signo \( j \)) que, además de las variables \( u_1,\ldots, u_n \), tiene otras variables libres como parámetros que en  \( \phi^3(j^{N-t_{x_1}}(x_1),\ldots, j^{N-t_{x_n}}(x_n)) \) aparecen sustituidas por conjuntos de la forma \( V_{j^k(\alpha)} \) (los que acotan las variables ligadas por cuantificadores) o \( j^k(F) \).

En definitiva, dada una fórmula estratificada \( \phi(x_1,\ldots, x_n) \) del lenguaje de NFA, hemos encontrado una fórmula \( \phi^3(u_1,\ldots, u_n,\ldots) \) del lenguaje de ZFC tal que \( \phi(x_1,\ldots, x_n) \) es verdadera en \( M \) si y sólo si \( \phi^3(j^{N-t_{x_1}}(x_1),\ldots, j^{N-t_{x_n}}(x_n),\ldots) \), donde los puntos suspensivos representan conjuntos de la forma \( V_{j^k(\alpha)} \) o \( j^k(F) \).

A partir de aquí consideramos una instancia concreta del axioma de formación de conjuntos, es decir, dada una fórmula estratificada \( \phi(x,x_1,\ldots, x_n) \), queremos probar que, dados \( x_1,\ldots, x_n\in M \), existe un conjunto \( A\in C \) tal que para todo objeto \( x\in M \) se cumple que \( x\,E\,A \) si y sólo si la fórmula \( \phi(x,x_1,\ldots, x_n) \) es verdadera en \( M \). Equivalentemente, buscamos un \( A\in V_{j(\alpha)+1} \) tal que:

\( \forall x\in V_\alpha(j(x)\in A\leftrightarrow \phi^3(j^{N-t_x}(x),j^{N-t_{x_1}}(x_1),\ldots, j^{N-t_{x_n}}(x_n))) \).

Ahora bien, cuando \( x \) recorre \( V_\alpha \) tenemos que \( j(x) \) recorre \( V_{j(\alpha)} \), luego la equivalencia anterior es equivalente a

\( \forall x\in V_{j(\alpha)}(x\in A\leftrightarrow \phi^3(j^{N-t_x-1}(x),j^{N-t_{x_1}}(x_1),\ldots, j^{N-t_{x_n}}(x_n))) \).

El \( A \) que buscamos tiene que ser un elemento de \( V_{j(\alpha)+1} \), es decir, un subconjunto de \( V_{j(\alpha)} \), para cualquier \( A\subset V_{j(\alpha)} \) la equivalencia anterior equivale a

\( \forall x(x\in A\leftrightarrow x\in V_{j(\alpha)}\land \phi^3(j^{N-t_x-1}(x),j^{N-t_{x_1}}(x_1),\ldots, j^{N-t_{x_n}}(x_n))) \).

Una vez más, cuando un conjunto \( x \) recorre \( V_{j(\alpha)} \), el término \( j^{N-t_x-1}(x) \) recorre \( V_{j^{N-t_x}(\alpha)} \), luego la condición anterior equivale a

\( \forall x(x\in A\leftrightarrow x\in V_{j^{N-t_x}(\alpha)}\land \phi^3(x,j^{N-t_{x_1}}(x_1),\ldots, j^{N-t_{x_n}}(x_n))) \).

Así pues, basta definir

\( A = \{x\in V_{j^{N-t_x}(\alpha)}\mid  \phi^3(x,j^{N-t_{x_1}}(x_1),\ldots, j^{N-t_{x_n}}(x_n))\} \),

que existe por el axioma de especificación de ZC. Esto termina la prueba de que el modelo \( M \) cumple el axioma de formación de conjuntos y, por consiguiente, es un modelo de NFA.

La semana que viene demostraremos que \( M \) cumple también el axioma de elección. De momento termino contestando una cuestión que planteaba Sailor Starruler, que conjeturaba que en NFA los pares ordenados de átomos deben ser átomos y los de conjuntos conjuntos.

Acabamos de ver que podemos construir un modelo de NFA en el que los pares ordenados nivelados se correspondan con cualquier función inyectiva \( F:M\times M\longrightarrow M \) prefijada. Si tomamos \( F \) biyectiva tenemos un modelo de NFA en el que todos los objetos son pares ordenados. Si, más concretamente, tomamos \( F \) como unión de una biyección \( C\times C\longrightarrow C \) y una biyección \( (M\setminus C)\times (M\setminus C)\longrightarrow M\setminus C \), obtenemos un modelo de NFA en el que los pares ordenados de átomos son átomos y los de conjuntos son conjuntos, mientras que si partimos de una biyección \( F: M\times M\longrightarrow M\setminus C \) obtenemos un modelo de NFA en el que todos los pares ordenados son átomos. Lo que no podemos conseguir es un modelo de NFA en el que todos los pares ordenados sean conjuntos, pues \( |C|<|M| \), luego no existen inyecciones \( M\times M\longrightarrow C \). Por otra parte, también podemos diseñar una aplicación inyectiva \( F:M\times M\longrightarrow M \) que asigne algunos pares de átomos a conjuntos y otros a átomos, y algunos pares de conjuntos a átomos y otros a conjuntos.

Así pues, todas las posibilidades sobre la "naturaleza" de los pares ordenados nivelados son consistentes con los axiomas de NFA excepto que todos sean conjuntos.
Título: Modelos de NFA (3)
Publicado por: Carlos Ivorra en 25 Marzo, 2012, 03:37 pm
En este mensaje terminamos la descripción del modelo de NFA que hemos construido. Resumo lo que tenemos hasta ahora:

1) Estamos trabajando en ZFC más la existencia de un automorfismo j que cumple \( j(\alpha)<\alpha \) para cierto ordinal infinito \( \alpha \).

2) Fijado un ordinal en estas condiciones, el conjunto de los objetos de nuestro modelo de NFA es \( M = V_\alpha \)

3) De entre ellos, serán conjuntos los elementos de \( C = V_{j(\alpha)+1} \) y los restantes serán átomos.

4) Definimos la relación de pertenencia en \( M \) como la dada por

\( x\,E\,y\leftrightarrow y\in C\land j(x)\in y \).

5) Fijamos una aplicación inyectiva \( F: M\times M\longrightarrow M \) y definimos el par ordenado determinado por dos objetos de \( M \) como \( (x,y)^M = F(x,y) \).

Hemos demostrado que si tomamos como objetos los elementos de \( M \) y definimos los conjuntos, la pertenencia y los pares ordenados como acabamos de hacer, se cumplen todos los axiomas de NFA. Falta probar que también se cumple el axioma de elección.

Para ello observamos que si \( x,y\in C \) son dos \( \text{conjuntos}^M \) cualesquiera (el superíndice indica que son conjuntos respecto al modelo \( M \), pues, por ejemplo, si \( x\in M\setminus C \), entonces \( x \) es un conjunto, pero no es un \( \text{conjunto}^M \), sino un  \( \text{\'atomo}^M \)) , entonces \( x\subset^M y \) equivale a \( x\subset y \).

Demostración
La fórmula \( x\subset^M y \) significa que \( x \) e \( y \) satisfacen la definición de inclusión en NFA, lo cual equivale a que sean conjuntos (que lo son, porque así lo estamos suponiendo) y además cumplan

\( \forall u\in M(u\,E\,x\rightarrow u\,E\, y) \),

Pero esto equivale a \( \forall u\in M(j(u)\in x\rightarrow j(u)\in y) \), o también a \( \forall u\in M(u\in j^{-1}(x)\rightarrow u\in j^{-1}(y)) \), es decir, a \( j^{-1}(x)\subset j^{-1}(y) \) (porque \( x, y\subset M \)) y, como \( j \) es un automorfismo, esto equivale a \( x\subset y \).
[cerrar]

Más en general, si \( y\in C \) y \( x\subset y \), entonces automáticamente \( x\in C \) y \( x\subset^M y \).

Demostración
Tenemos que \( y\in V_{j(\alpha)+1} \), luego \( x\subset y\subset V_{j(\alpha)} \), luego \( x\in \mathcal PV_{j(\alpha)} = V_{j(\alpha)+1}= C \).
[cerrar]

Por otra parte, si \( x,y\in C \), se cumple que \( (x\times y)^M = j(F)[x\times y] \).

Demostración
Tenemos que \( u\in (x\times y)^M\leftrightarrow j(j^{-1}(u))\in (x\times y)^M\leftrightarrow j^{-1}(u)\,E\,(x\times y)^M \)\( \leftrightarrow (j^{-1}(u)\in x\times y)^M \)

Esto último equivale a que \( j^{-1}(u) \) sea un par ordenado en \( M \) con primera componente en \( x \) y segunda componente en \( y \), es decir a que

\( \exists vw\in M(v\,E\,x\land w\,E\, y\land j^{-1}(u) = (v,w)^M) \), que a su vez equivale a \( \exists vw\in M(j(v)\in x\land j(w)\in y\land j^{-1}(u) = F(v,w)) \), o también a \( \exists vw\in M(v\in j^{-1}(x)\land w\in j^{-1}(y)\land j^{-1}(u) = F(v,w)) \). Esto significa que \( j^{-1}(u)\in F[j^{-1}(x)\times j^{-1}(y)] \) y, como \( j \) es un automorfismo, es equivalente a \( u\in j(F)[x\times y] \).
[cerrar]

Por lo tanto, si \( x,y\in C \) y \( R\subset x\times y \), tenemos que \( j(F)[R]\subset j(F)[x\times y] \), luego, llamando \( R^*=j(F)[R] \), tenemos que \( R^*\subset (x\times y)^M \). De este modo, a cada relación \( R \) entre dos conjuntos \( x \) e \( y \) podemos asociarle una \( \text{relaci\'on}^M \) \( R^* \) tal que, para todo \( u,v\in M \), se cumple

\( (u\,R^*\,v)^M\leftrightarrow (u,v)^M\,E\, R^*\leftrightarrow j(F(u,v)) = j(F)(j(u),j(v))\in j(F)(R) \) \( (j(u),j(v))\in R\leftrightarrow j(u)\,R\,j(v) \).

Ahora es fácil probar que si \( R \) es un buen orden en un conjunto \( x\in C \), entonces (\( R^* \) es un buen orden en \( x)^M \).

Demostración
Las propiedades de las relaciones de orden se traducen fácilmente a \( M \). Veamos, por ejemplo, la propiedad simétrica: si \( (u\,R^*\,v)^M\land (v\,R^*\,u)^M \), entonces \( j(u)\,R\,j(v)\land j(v)\,R\,j(u) \), luego \( j(u) = j(v) \) y, por consiguiente, \( u=v \).

Supongamos ahora que \( y\in C \) cumple \( (y\subset x\land y\neq \emptyset)^M \). Hemos de probar que \( y \) tiene un mínimo elemento para la relación \( R^* \). La hipótesis equivale a que \( y\subset x\land y\neq \emptyset \), por lo que \( y \) tiene un mínimo elemento para \( R \), es decir, existe \( u\in y \) tal que \( \forall v\in y\ u\,R\,v \).

Por lo tanto, si \( (v\in y)^M \), es decir, si \( j(v)\in y \), tenemos que \( j(j^{-1}(u))\,R\,j(v) \), que equivale a \( (j^{-1}(u)\,R^*,v)^M \), y esto prueba que \( j^{-1}(u) \) es el mínimo de \( y \) en \( M \).
[cerrar]

Ahora es inmediato que \( M \) cumple el axioma de elección, pues cumple que todo conjunto puede ser bien ordenado. En efecto, dado \( y\in C \), podemos tomar un buen orden \( R \) en \( y \), y acabamos de probar que \( (R^* \) es un buen orden en \( y)^M \).

También es inmediato probar que si \( x,y\in C \) y \( f: x\longrightarrow y \) (inyectiva, suprayectiva, biyectiva) entonces \( (f^*: x\longrightarrow y)^M \) (inyectiva, suprayectiva, biyectiva) y se cumple que \( (f^*(u))^M = j^{-1}(f)(u) \). Además, toda aplicación en \( M \) es de esta forma.

Demostración
Si \( (g: x\longrightarrow y)^M \), entonces \( g\subset (x\times y)^M = j(F)[x\times y] \), luego \( f = j(F)^{-1}[g]\subset x\times y \) y se cumple que \( g = j(F)[f]=f^* \). El hecho de que \( g \) sea una aplicación (inyectiva, suprayectiva, biyectiva) en \( M \) se traduce fácilmente en que \( f \) también lo es (fuera de \( M \)).
[cerrar]

Tanto en \( M \) como fuera de \( M \) tenemos que un conjunto \( x \) es infinito si y sólo si existe \( f: x\longrightarrow x \) inyectiva y no suprayectiva, luego podemos concluir que, para todo  \( x\in C \), se cumple que \( x \) es infinito en \( M \) si y sólo si es infinito, luego \( x \) es finito en \( M \) si y sólo si es finito.

Por otro lado, \( (|x| = |y|)^M \) si y sólo si existe una biyección \( f: x\longrightarrow y \) (en M o, equivalentemente, fuera de M) si y sólo si \( |x| = |y| \).

Dado \( (n\in \mathbb{N})^M \), tomemos un conjunto \( u\,E\,n \), es decir, \( j(u)\in n \). Entonces \( u \) es finito en \( M \), luego es finito, y existe un número natural \( \nu\in \omega \) (el conjunto de los ordinales finitos de von Neumann) tal que \( |u|=\nu = |\nu| \), luego \( (|u| = |\nu|)^M \), luego \( \nu\in n \).

En definitiva, todo número natural en M contiene un número natural (ordinal de von Neumann), que claramente es único (pues si contuviera dos, serían equipotentes en M, luego serían equipotentes). Recíprocamente, todo número natural \( \nu \) es finito, luego finito en M, luego pertenece a un número natural en M, es decir \( j(\nu) \) pertenece a un número natural en M. Tenemos así una biyección \( N: \omega\longrightarrow \mathbb{N}^M \) que a cada número natural \( \nu \) le asigna el único número natural \( n \) en \( M \) que cumple \( j(\nu)\in n \).

Veamos ahora que si \( u\in M \) entonces \( \{u\}^M = \{j(u)\} \).

Demostración
\( v\in \{u\}^M\leftrightarrow j(j^{-1}(v))\in \{u\}^M\leftrightarrow j^{-1}(v)\,E\,\{u\}^M\leftrightarrow j^{-1}(v)=u\leftrightarrow v = j(u) \)
[cerrar]

Por otro lado, si \( x\in C \), se cumple que \( (\mathcal P_1x)^M =j(\mathcal P_1x) \).

Demostración
\( u\in (\mathcal P_1x)^M\leftrightarrow j(j^{-1}(u))\in (\mathcal P_1x)^M\leftrightarrow (j^{-1}(u)\,E\,(\mathcal P_1x))^M \) \( \leftrightarrow \exists v(v\,E\,x\land j^{-1}(u) = (\{v\}^M))\leftrightarrow \exists v(j(v)\in x\land j^{-1}(u) = \{j(v)\}) \) Esto equivale a que \( j^{-1}(u)\in \mathcal P_1x \), luego a que \( u\in j(\mathcal P_1x) \).
[cerrar]

Ahora es inmediato que si \( (n\in \mathbb{N})^M \) es un número natural que se corresponde con \( \nu\in \omega \), entonces \( (Tn)^M \) se corresponde con \( j(\nu) \).

Demostración
Tenemos que \( \nu\,E\,n \), luego \( (\mathcal P_1\nu)^M\,E\, (Tn)^M \), pero \( (\mathcal P_1\nu)^M = j(\mathcal P_1\nu) \) y \( |\mathcal P_1\nu| = |\nu| \), luego \( |j(\mathcal P_1\nu)| = |j(\nu)| \), luego \( (|j(\nu)| = |\mathcal P_1\nu|)^M \), luego \( j(\nu)\,E\, (Tn)^M \), y esto significa que \( j(\nu) = N((Tn)^M) \).
[cerrar]


Así pues, el axioma de cómputo, que en una de sus formas equivalentes afirma que \( \forall n\in \mathbb{N}\ T(n)=n \), se cumple en \( M \) si y sólo si \( \forall \nu\in \omega\ j(\nu) = \nu \), es decir, si y sólo si \( j \) fija a todos los números naturales.

No hemos visto los detalles de la construcción del modelo de ZC dotado del automorfismo \( J \) que garantiza la consistencia de la existencia del automorfismo \( j \) con el que estamos trabajando, pero en ella podemos exigir que \( J \) tenga la propiedad de dejar fijos a todos los números naturales. Cuidando de que esto sea así, llegamos a que el modelo \( M \) que estamos considerando es un modelo de NFA + AC, luego esta teoría es consistente si ZFC es consistente.
Título: Re: NFA
Publicado por: specu en 13 Mayo, 2012, 04:30 pm
Muy interesante esta serie de publicaciones. Por otra parte, la lectura de Quine me resultó siempre muy agradable tanto por la claridad habitual en sus escritos como por su interés por los temas propios de la filosofía (es muy de lamentar que muchos de sus textos no se consigan en castellano).

En este caso, envío este mensaje para mencionar sólo, por si resulta de utilidad, que en la entrada http://rinconmatematico.com/foros/index.php/topic,53163.msg211927.html#msg211927
en la definición de inclusión, me parece que falta el cuantificador universal para la variable «u», pudiendo confundir al lector.


Saludos
Título: Re: NFA
Publicado por: Carlos Ivorra en 13 Mayo, 2012, 04:55 pm
En este caso, envío este mensaje para mencionar sólo, por si resulta de utilidad, que en la entrada http://rinconmatematico.com/foros/index.php/topic,53163.msg211927.html#msg211927
en la definición de inclusión, me parece que falta el cuantificador universal para la variable «u», pudiendo confundir al lector.

Ya he corregido la errata. Gracias por el aviso.
Título: Re: NFA
Publicado por: Carlos Ivorra en 22 Junio, 2012, 12:12 pm
Por si a alguien le puede interesar, acabo de publicar en mi página web

http://www.uv.es/~ivorra/Libros/Libros.htm (http://www.uv.es/~ivorra/Libros/Libros.htm)

Un libro titulado Teorías de Conjuntos cuyos tres últimos capítulos tratan sobre NFA y desarrollan el contenido de este hilo, con pruebas completas de todos los resultados discutidos aquí, y muchos más.
Título: Re: Número 1. (2012) - 2. NFA
Publicado por: inttegrattor en 26 Noviembre, 2014, 08:01 am
Les estaré muy agradecido si mantienen sus publicaciones en este foro, es realmente EMOCIONANTE leer las discusiones y aportes, se aprende bastante. ¡FELICITACIONES! :)